Derm ROSH
A 19-year-old woman is found to have polycystic ovarian syndrome resulting in increased levels of serum androgens. Which of the following dermatologic conditions is most likely to occur in this patient? AAcne vulgaris BHyperhidrosis CRosacea DTinea infection
Correct Answer ( A ) Explanation: Acne vulgaris is a common dermatologic condition characterized by a papular or pustular eruption involving the chest, face, and back. Etiology is not precisely defined, however, plugged follicles, increased sebum production, and inflammation are thought to promote acne. The presence of Propionibacterium acnes (also called Cutibacterium acnes) is suspected to have a pathogenetic role in the development of acne vulgaris. Adolescents between the ages of 10 to 15 years old are most likely to present with acne, and the condition tends to persist into adulthood for five to 10 years. Patients with family members who experience acne vulgaris are more likely to develop the condition themselves. If both of a child's parents have acne, there is a 75% chance that the child will also have acne. Patients with androgen excess (e.g., congenital adrenal hyperplasia, polycystic ovarian syndrome) are at an increased risk for developing acne vulgaris. Patients with moderate or severe inflammatory acne through perimenarcheal years or those with persistent acne that is poorly responsive to antibiotic therapy should be evaluated for these conditions by testing serum levels of testosterone, follicle stimulating hormone, luteinizing hormone, and dehydroepiandrosterone sulfate. Acne skin lesions are classified as either inflammatory or noninflammatory. Papules, pustules, and nodules (cysts) are inflammatory manifestations, while noninflammatory findings include open comedones (blackheads) and closed comedones (whiteheads). Fungal and bacterial cultures can be used to differentiate between acne and infectious folliculitis. Initial treatment of acne vulgaris involves proper facial hygiene with twice daily washing with mild soap and tepid water. Topical medical therapies are first-line treatment for acne vulgaris and include prescription and over-the-counter medications that contain sulfur, salicylic acid, resorcinol, and benzoyl peroxide. Topical retinoids (e.g., tretinoin, adapalene, tazarotene) are comedolytic medications that are first-line agents to treat these lesions and can result in irritation, flaking, and dryness of the skin. Topical antimicrobial medications (e.g., clindamycin, erythromycin, dapsone) are administered to control inflammatory lesions by decreasing the presence of P. acnes. Azelaic acid is effective in the treatment of both inflammatory and noninflammatory acne. If topical treatment is ineffective, oral antibiotics (e.g., azithromycin, tetracycline) can be administered. This treatment strategy requires weeks to demonstrate efficacy and compliance for months to achieve maximal benefit. Photodynamic therapy involves the application of a photosensitizing drug with subsequent exposure to a laser or blue light source. Patients with nodulocystic acne are less likely to benefit from the previously described regimens and can be administered oral isotretinoin if significant response to other therapies is not achieved after three months. Markedly severe cases of acne vulgaris may require oral prednisone to decrease inflammation. Female patients can be started on oral contraceptives. Those who are unable to take an oral contraceptive or who have relapsed after a course of oral retinoid therapy can be administered spironolactone. Tazarotene and oral isotretinoin are contraindicated in pregnant patients as they are pregnancy category X medications. The safest drugs in this demographic are azelaic acid, topical erythromycin or clindamycin, and oral erythromycin or azithromycin. Dermatologic surgeons can treat scarring and pigmentation changes with excision, laser therapy, dermabrasion, chemical peels, and injection with fillers (e.g., collagen, fat). Patients may develop significant psychosocial complications secondary to acne vulgaris, such as depression, anxiety, and social withdrawal. Hyperhidrosis (B) is a dermatologic condition characterized by excessive sweat production that can involve localized or widespread areas of skin. Underlying conditions include infection, vasomotor conditions (Reynaud phenomenon), neurologic disease (Parkinson disease, postencephalitis), and neoplasia (lymphoma). Metabolic and endocrine dysfunction may also cause hyperhidrosis with implicated conditions including hyperthyroidism, diabetes mellitus, obesity, and carcinoid syndrome. While patients with polycystic ovarian syndrome often present with obesity and insulin resistance, the hyperandrogenic state of the condition predisposes patients to developing acne vulgaris. Rosacea (C) presents with redness, telangiectasia, flushing, blushing, and papules and pustules. It is most common in patients older than 30 years of age and has an unknown cause. Patients with polycystic ovarian syndrome are not more likely to develop this condition than the general population. Hyperandrogenism does not predispose a patient to tinea infection (D). This fungal infection has a variety of subtypes that are named based on the involved area, such as tinea pedis that affects the foot while tinea capitis affects the scalp. Tinea infections commonly present as a flat annular lesion with an elevated scaly border.
A 75-year-old farmer with pale skin presents to the clinic with several erythematous and scaly individual papular lesions on his balding scalp. The lesions are less than 1 centimeter, are not ulcerated, have not bled, and are growing slowly. There are also similar lesions on his dorsal forearms and hands. Which of the following is the most likely diagnosis? AActinic keratosis BMelanoma CSeborrheic keratosis DSquamous cell carcinoma
Correct Answer ( A ) Explanation: Actinic keratoses are common cutaneous lesions that result from the proliferation of atypical epidermal keratinocytes. Risk factors for the development of actinic keratoses include chronic sun exposure, history of sunburns, fair skin, advancing age, and male sex. Actinic keratoses have the potential to progress to squamous cell carcinoma. The risk that an individual lesion will progress to squamous cell carcinoma is low, but about 60% of squamous cell carcinomas begin as actinic keratoses. Actinic keratoses most commonly present as scaly, erythematous macules or papules on sites of chronic sun exposure. They are often described as feeling like sandpaper. Lesions may also be pigmented, nonerythematous, or free of scale. The typical locations are the scalp, face, lateral and posterior neck, dorsal forearms, and dorsal hands. The lesions are usually a few millimeters to 2 centimeters in diameter. Actinic keratosis is often diagnosed based on visual and tactile clinical examination. Biopsy is indicated if the diagnosis is uncertain, and biopsies are often performed to distinguish between actinic keratosis and squamous cell carcinoma. Indications for biopsy include lesions larger than 1 centimeter in size, indurated or ulcerated lesions, bleeding lesions, rapidly growing or recurring lesions, and lesions that fail to respond to appropriate therapy. There should be a low threshold to consider the possibility of squamous cell carcinoma in immunocompromised patients. The recommended treatment for isolated actinic keratosis is liquid nitrogen cryotherapy. Shave removal (biopsy) is an alternative treatment modality that is especially useful in suspicious lesions that require biopsy. Field treatment (allows treatment of a larger area) with topical fluorouracil, topical imiquimod, or photodynamic therapy is recommended for patients with multiple lesions on the face or scalp. Combination therapy, including field treatment and liquid nitrogen cryotherapy, is recommended for patients with multiple lesions that are hyperkeratotic. Sun avoidance is recommended to avoid the development of actinic keratosis and is most important during the peak hours in the spring and summer. Protective clothing and use of broad-spectrum sunscreens with a sun protective factor of 30 or higher are recommended while in the sun to limit ultraviolet exposure. Sunscreen should be applied 15 to 30 minutes prior to sun exposure to allow a protective film on the skin to form, and it should be reapplied every two hours. Melanoma (B) classically presents as a hyperpigmented lesion. The most common type is superficial spreading. Melanoma is often asymmetric, with irregular borders, variation in color, diameter > 6 millimeters, and evolving lesions. The diagnosis is made with biopsy. The description of the lesions in this vignette is more consistent with actinic keratosis. Seborrheic keratosis (C) is the most common benign skin tumor. These lesions are small, raised, and well-demarcated with a stuck-on appearance. The risk factors, lesion description, and location (only sun-exposed areas) in this patient are most consistent with actinic keratosis. Squamous cell carcinoma (D) is often preceded by actinic keratosis. These lesions are ultimately distinguished by biopsy. However, the patient in this vignette has no findings that raise concern for squamous cell carcinoma, such as lesions larger than 1 centimeter in size, ulceration, bleeding, rapid growth, rapid recurrence, or failure to respond to treatment.
An 8-year-old boy presents to the clinic with multiple flesh-colored, dome-shaped papules on his trunk and axilla. The lesions have central umbilication. Which of the following is the best recommended treatment? AObservation BTopical ketoconazole CTopical mupirocin DTopical permethrin
Correct Answer ( A ) Explanation: Molluscum contagiosum is a poxvirus that causes localized skin infections. Molluscum contagiosum occurs most often in children. It is transmitted by direct skin contact or fomites. Molluscum contagiosum most commonly presents as single or multiple small, flesh-colored, dome-shaped papules with central umbilication. The lesions are typically 2-5 millimeters in diameter with a shiny surface. The most common locations include the trunk, axilla, popliteal fossa, antecubital fossa, and crural folds. The lesions may be pruritic. Furthermore, the lesions can become inflamed, which is actually a sign of impending resolution and does not necessarily suggest secondary infection. Immunocompromised patients may experience persistent and widespread disease. The diagnosis of molluscum contagiosum is made based on clinical findings, however, biopsy can confirm the diagnosis when necessary. Molluscum contagiosum is self-limited in immunocompetent individuals, therefore, it is a reasonable management option to observe and not treat. If treatment is desired, then cryotherapy, curettage, cantharidin, or podophyllotoxin may be used. Cantharidin cannot be used in the genital area. Cryotherapy can result in prominent hypopigmentation in patients with dark skin. Children with molluscum contagiosum should not be excluded from daycare or school, however, lesions should be covered with clothing or a bandage to reduce the risk of transmission to others. Topical ketoconazole (B) is used to treat fungal infections, such as candidiasis, dermatophyte infections, and seborrheic dermatitis. Topical mupirocin (C) is a topical antibiotic used in the treatment of mild paronychia and impetigo. Topical permethrin (D) is a pediculicide used in the treatment of head lice and scabies.
Which of the following describes part of the pathogenesis of acanthosis nigricans? AHypersensitivity reaction BInsulin resistanceCorrect Answer CProliferation of melanocytes DSuperficial bacterial infection
Correct Answer ( B ) Explanation: Acanthosis nigricans is a common and benign disorder that typically presents with hyperpigmented plaques on the skin. There are a variety of endocrine (diabetes mellitus, polycystic ovarian syndrome, Cushing syndrome, acromegaly), metabolic, genetic, and malignant disorders that may contribute to the development of acanthosis nigricans, and an evaluation of the underlying cause should always be performed. Obesity is the most common cause of this disorder. Drugs are an additional cause of acanthosis nigricans. Insulin resistance plays a role in many cases of acanthosis nigricans that are linked to obesity and diabetes, which includes most cases of acanthosis nigricans. This correlation suggests that hyperinsulinemia plays a role in the development of acanthosis nigricans. The classic presentation is thickened hyperpigmented plaques that are grey to brown in color. The back of the neck, sides of the neck, and axillae are the most common sites of involvement. Other intertriginous areas, such as the anogenital region, inframammary region, and abdominal, antecubital, and inguinal folds, are less commonly affected. Severe cases may involve mucosal sites or other non intertriginous areas. Acanthosis nigricans typically occurs in a symmetric distribution and is usually asymptomatic. However, lesions that occur in skin folds may become macerated and inflamed, leading to a malodorous smell. Acanthosis nigricans can usually be diagnosed clinically. Skin biopsy can be done in rare cases where the diagnosis is uncertain. Patients with acanthosis nigricans should be evaluated for the possibility of diabetes mellitus. The possibility of an occult malignancy (gastric cancers most commonly) should be considered in older, nonobese adults with new-onset acanthosis nigricans without another identifiable cause. However, there is no clear evidence regarding the appropriate cancer evaluation in those patients. The primary indication for treatment of acanthosis nigricans is cosmetic concern because most often it is a benign condition that is typically asymptomatic. Treatment of the underlying cause is the preferred management, such as weight loss or improving glycemic control. Furthermore, if acanthosis nigricans is drug-induced (systemic glucocorticoids, injected insulin, oral contraceptives, testosterone), then discontinuing the causative medication should be considered. Acanthosis nigricans tends to improve with treatment in cases due to obesity, drugs, or malignancy. Cases due to insulin resistance are less likely to improve. Topical retinoids, such as topical tretinoin, and topical vitamin D analogs, such as calcipotriol, are the treatment of choice when improvement is unsatisfactory or when the underlying cause cannot be treated. Excessive scrubbing of the affected skin during bathing is not recommended. Acanthosis nigricans is a chronic disorder that persists in the absence of removal of the underlying cause or successful skin-directed therapy. Hypersensitivity reactions (A) occur when the immune system has a harmful effect. Hypersensitivity reactions include allergic and autoimmune conditions. Acanthosis nigricans is not known to be a hypersensitivity reaction. Proliferation of melanocytes (C) describes the pathogenesis of a mole. Superficial bacterial infection (D) is the cause of infections, such as cellulitis, erysipelas, and impetigo.
Which of the following is an appropriate second-line therapy for acne vulgaris? AIsotretinoin BOral doxycycline COral trimethoprim-sulfamethoxazole DTopical benzoyl peroxide
Correct Answer ( B ) Explanation: Acne vulgaris is the most common cutaneous disorder affecting adolescents and young adults, and the prevalence of acne vulgaris is estimated to be 35-90%. The pathophysiology of acne can be divided into four main factors: follicular hyperkeratinization, increased sebum production, Cutibacterium acnes within the follicle, and inflammation. Other factors that may play a role in acne genesis include diet, specifically high intake of dairy products, family history, stress, obesity, and insulin resistance. The prognosis of acne is typically very good, and most acne resolves in the third decade of life. For patients with chronic inflammatory or severe nodulocystic acne, there is an increased risk of scarring or postinflammatory hyperpigmentation, which may be difficult to treat. Acne vulgaris typically affects those areas of the body that have the largest hormonally responsive sebaceous glands, including the face, neck, chest, upper back, and upper arms. Mild acne typically involves a few comedones and papulopustules. There is typically no scarring or postinflammatory hyperpigmentation. Moderate acne involves comedones, inflammatory papules, and pustules. There are typically more lesions involved than mild acne, and scarring and postinflammatory hyperpigmentation are possible. Severe acne involves comedones, inflammatory lesions, and large, cystic nodules greater than 5 mm. Significant erythema and tenderness are often present. Scarring and postinflammatory hyperpigmentation are very likely. Acne vulgaris is a clinical diagnosis. For patients in whom polycystic ovarian syndrome is a suspected underlying cause of acne vulgaris, serum total and free testosterone, dehydroepiandrosterone sulfate, luteinizing hormone, and follicle-stimulating hormone values may be measured. Additionally, if insulin resistance is suspected, serum glucose and insulin levels may be measured. The treatment for acne vulgaris depends on the level of severity of acne involvement. For most patients, first-line therapy with topical benzoyl peroxide, a topical antibiotic such as clindamycin, or a topical tretinoin may be effective. For patients with moderate to severe acne, or patients who have not found success with first-line therapies, initiating an oral antibiotic as a second-line therapy may be beneficial. Oral doxycycline or azithromycin is preferred. Antibiotics in the tetracycline class are most frequently used and appear to have both antibacterial and anti-inflammatory properties. Topical medications may continue to be an effective adjunct to oral antibiotic therapy and are often continued. For patients with severe, nodulocystic acne, or for patients who have not found success with multiple courses of oral antibiotics, oral isotretinoin is an available treatment option. It is also useful for treating scarring and post-inflammatory hyperpigmentation resulting from severe acne vulgaris. The risk for adverse effects, including teratogenicity, precludes the use of isotretinoin as routine acne therapy. In all cases of acne vulgaris, washing with a gentle cleanser and using a noncomedogenic moisturizer are important for skin maintenance. Isotretinoin (A) is reserved for severe nodulocystic or recalcitrant acne. It may also help reduce postinflammatory hyperpigmentation or scarring as a result of chronic inflammatory acne. It is considered a third-line therapy. The risk for adverse effects, including teratogenicity, precludes the use of this drug as routine acne therapy. Oral trimethoprim-sulfamethoxazole (C) is an effective treatment for severe acne, however, its use is limited by the potential for bone marrow suppression and for the development of toxic epidermal necrolysis. Therefore, this antibiotic is reserved for patients who cannot tolerate tetracyclines or who have treatment-resistant acne. Topical benzoyl peroxide (D) is an effective first-line therapy for the treatment of acne vulgaris. It has both antibacterial and comedolytic properties and is available in both prescription and nonprescription strengths.
A 20-year-old woman presents to the clinic with a rash consisting of symmetric and distinct target lesions. She is diagnosed with erythema multiforme. What is the most common etiology of this condition? ABeta-lactam antibiotics BHerpes simplex virus CMycoplasma pneumoniae infections DSulfonamides
Correct Answer ( B ) Explanation: Erythema multiforme is an uncommon and immune-mediated disorder that presents with cutaneous and sometimes mucosal lesions. The pathogenesis involves a hypersensitivity reaction that can be triggered by infections or medications. Infections are the most common cause of erythema multiforme and account for 90% of cases. Herpes simplex virus is the most common etiologic agent, and most cases are associated with herpes labialis (cold sore on the lip). Erythema multiforme most commonly affects young adults but can affect patients of any age. Medications are an uncommon cause of erythema multiforme and account for fewer than 10% of cases. Erythema multiforme can be classified as major or minor depending on the presence of mucosal involvement. Erythema multiforme major has cutaneous and mucosal involvement, whereas erythema multiforme minor only has cutaneous involvement. The clinical presentation of erythema multiforme is characterized by the appearance of distinctive target-like lesions on the skin. The target lesion is described as an erythematous papule with surrounding pallor and an outside erythematous halo. Typically, a few to hundreds of symmetric and similarly appearing lesions erupt within 24 hours and then progress over three to four days. Cutaneous lesions often begin on extensor surfaces (elbows and knees) but can spread centripetally. Lesions are usually asymptomatic but some patients describe pruritus or burning. These lesions may be accompanied by erosions or bullae. Erythema multiforme major may have prodromal symptoms, such as fever, chills, and arthralgias, whereas erythema multiforme minor does not have prodromal symptoms. Mucous membrane involvement typically occurs with cutaneous involvement, but rarely does mucosal involvement occur alone. The oral mucosa is most commonly affected, but the ocular or genital mucosa may also be affected. The lesions or erythema multiforme typically resolve within two weeks in erythema multiforme minor and within four to six weeks in erythema multiforme major. Other symptoms may include symptoms of the underlying etiologic infection, such as herpes labialis. These skin lesions do not scar, but postinflammatory hyperpigmentation may occur. A minority of patients with erythema multiforme have recurrent symptoms and experience several episodes per year. Erythema multiforme is typically diagnosed clinically. Patients should be asked about symptoms and signs of recent infections, such as herpes simplex virus infections, Mycoplasma pneumoniae infection, and recent new medications. However, in cases where the diagnosis is unclear, biopsy can be performed to confirm the diagnosis. Laboratory findings in individuals with erythema multiforme are nonspecific and may include leukocytosis, elevated erythrocyte sedimentation rate, or elevated liver enzymes (aspartate aminotransferase and alanine aminotransferase). The management of erythema multiforme includes glucocorticoids. Most individuals with mild or moderate disease (limited mucosal involvement) are treated with topical glucocorticoids, such as triamcinolone acetonide. Patients with severe disease that involves the mucosal surfaces may be treated with systemic steroids, such as prednisone, for two to four weeks. Patients with itching can be treated with antihistamines, such as cetirizine. Patients with ocular involvement should be referred to an ophthalmologist. Complications of ocular involvement may include conjunctival scarring, keratitis, or visual impairment. Treatment of the underlying infectious etiology often is not recommended in first-time cases of erythema multiforme because the underlying infection has often resolved by the time the rash begins. Treatment of herpes simplex virus-induced erythema multiforme with oral antivirals, such as acyclovir, does not alter the course of erythema multiforme. The inciting agent (e.g., infectious etiologic agent or medication) should be identified and eliminated in patients with recurrent erythema multiforme. Patients with herpes simplex virus-induced or idiopathic erythema multiforme that recurs at least six times per year or who have less frequent but disabling recurrent episodes should be treated with continuous antiviral treatment, such as continuous acyclovir. Patients with severe and recurrent disease who fail to respond to continuous systemic antiviral treatment should be treated with azathioprine, mycophenolate, or dapsone. Beta-lactam antibiotics (A) consist of penicillins and cephalosporins. Medications cause fewer than 10% of cases of erythema multiforme. Mycoplasma pneumoniae infections (C) are a cause of atypical community-acquired pneumonia. Mycoplasma pneumoniae infections can be associated with erythema multiforme, especially in children, but are not as common as an etiologic agent as herpes simplex virus. Sulfonamides (D) include trimethoprim-sulfamethoxazole. Sulfonamides are a possible cause of erythema multiforme, but medications cause fewer than 10% of cases of erythema multiforme.
A 60-year-old woman presents to the clinic with a unilateral vesicular rash on an erythematous base for the past three days. She says that the rash was preceded by a burning pain in the same distribution. Which of the following is the most likely diagnosis? AAllergic contact dermatitis BHerpes zoster CImpetigo DVaricella
Correct Answer ( B ) Explanation: Herpes zoster is caused by a reactivation of latent varicella zoster virus in the sensory ganglia. Varicella zoster virus establishes latency in dorsal root and cranial nerve ganglia after primary infection and can spread to the skin of that dorsal root ganglion via the peripheral nerves. Herpes zoster is most common in individuals over age 50 and immunocompromised individuals are at increased risk. The classic presentation of herpes zoster is a dermatomal vesicular rash and acute neuritis, which usually precedes the rash. Acute neuritis typically manifests as pain, which may be described as burning, sharp, stabbing, or tingling. Acute neuritis occurs prior to the development of the rash in 75% of cases. The rash classically presents as grouped vesicles on an erythematous base in a dermatomal distribution and does not cross the midline. The thoracic and lumbar dermatomes are most commonly involved. Involvement of the trigeminal nerve can cause sight-threatening ophthalmic involvement. Patients with ophthalmic involvement should see an ophthalmologist. Most cases involve one dermatome, although involvement of multiple dermatomes is possible. The rash typically begins as erythematous papules, which progress to vesicles and bullae. Subsequently, within three or four days, the vesicles often become pustular. In immunocompetent hosts, the lesions typically crust within seven to 10 days and are no longer considered contagious. Systemic symptoms, such as fever, malaise, and headache, are present in 20% of cases. Scarring or alterations in pigmentation (hypopigmentation or hyperpigmentation) may persist for months after the lesions have resolved. In immunocompromised patients, the rash of herpes zoster may have lesions in a generalized distribution. The diagnosis of herpes zoster is made clinically by the presence of pain in a dermatomal distribution that precedes the development of a rash. However, diagnostic testing with polymerase chain reaction may be obtained in atypical presentations. The treatment of herpes zoster consists of antiviral medications and pain control. Antiviral therapy is recommended for patients with uncomplicated herpes zoster who present within 72-hours of the onset of lesions. Patients who present more than 72 hours after lesions begin should be treated with antivirals if new lesions are still appearing at the time of presentation. Patients who have lesions that have already crusted are unlikely to benefit from antiviral therapy. However, immunocompromised patients should be treated with antivirals if they present within seven days of lesions appearing. The nucleoside analogue antivirals consisting of acyclovir, valacyclovir, and famciclovir are the recommended class of antiviral medications. The recommended medication for pain control varies based on the severity of pain. Patients with mild pain can be treated with acetaminophen or nonsteroidal anti-inflammatory drugs, such as ibuprofen. Patients with moderate or severe pain may require opioids, such as hydrocodone or oxycodone. Common medications used for adjuvant therapy include corticosteroids, gabapentin, and tricyclic antidepressants. However, these medications are not recommended for uncomplicated cases because clinical benefit has not been established. Secondary bacterial infection is a possible complication of herpes zoster, and it should be treated with antibiotics that cover staphylococcal and streptococcal organisms. Other complications of herpes zoster include Ramsay Hunt syndrome and herpes zoster ophthalmicus. Herpes zoster ophthalmicus is a sight-threatening condition that has been linked to varicella zoster virus reactivation within the trigeminal ganglion. Ramsay Hunt syndrome is the major otologic complication of varicella zoster virus. The classic manifestations of Ramsay Hunt syndrome include facial paralysis, ear pain, and vesicles in the auditory canal or auricle. The management of Ramsay Hunt syndrome consists of valacyclovir and prednisone. Neurologic complications of herpes zoster include meningitis, encephalitis, and myelitis. Intravenous acyclovir should be administered to patients with these neurologic complications. Postherpetic neuralgia is another complication of herpes zoster and is diagnosed clinically when pain persists beyond four months in the same distribution as the preceding documented episode of acute herpes zoster infection. The recommended treatment of postherpetic neuralgia is tricyclic antidepressants, such as amitriptyline. Patients with herpes zoster can transmit varicella zoster virus to individuals who have not had varicella and have not received the varicella vaccine. Transmission of varicella zoster virus occurs by direct contact or by aerosolization of virus from skin lesions. Patients with localized zoster are not infectious before vesicles appear and are no longer infectious when the lesions have crusted. Therefore, until the rash has crusted, patients should be advised to keep the rash covered and to wash their hands often to prevent the spread of the virus to others. Furthermore, patients with varicella zoster virus should avoid contact with pregnant women who have never had varicella or the varicella vaccine, premature or low birth weight infants, and immunocompromised individuals. Zoster vaccination is recommended in immunocompetent individuals who are 50 years of age or older. Allergic contact dermatitis (A) presents with erythematous and indurated scaly plaques. Vesiculation and bullae may be seen in severe cases. However, allergic contact dermatitis is typically associated with an exposure, such as poison ivy, and is marked by itching rather than burning pain. Furthermore, allergic contact dermatitis is not associated with a dermatomal distribution. Impetigo (C) is a superficial bacterial infection most often caused by Staphylococcus aureus. The classic progression of impetigo lesions may resemble herpes zoster because the lesions are initially papular, then vesicular, followed by pustular. The lesions of impetigo classically form golden-colored crusts. Impetigo is most common in children 2 to 5 years old. Impetigo is not preceded by pain and does not classically occur in a dermatomal distribution. Varicella (D) is another clinically-distinct form of varicella zoster virus disease. Varicella is also called chickenpox. Varicella is a mild illness in most healthy children, however, it is more severe in adults and immunocompromised individuals. Transmission of varicella occurs by direct cutaneous contact with vesicle fluid from skin lesions or by aerosolized droplets from nasopharyngeal secretions. Varicella presents as a diffuse vesicular rash. Prodromal symptoms may include fever, malaise, and pharyngitis. Lesions progress from papules, to vesicles, to pustules, and then to crusted papules. Lesions in different stages is a classic finding of varicella. The face, trunk, and extremities are common areas of involvement. The varicella vaccine was introduced as a routine childhood vaccination in 1995. The presentation of varicella is diffuse, whereas varicella zoster virus causes dermatomal manifestations.
A 35-year-old man presents to the office complaining of redness to his forehead and cheeks for the past few months. He has tried over-the-counter topical preparations, which have not helped. Upon physical examination, you note mild erythema to the cheeks, forehead, and nose with the presence of multiple telangiectasias, skin thickening, and dryness. He states that he experienced flushing to the face as a child that has progressively gotten worse. Which of the following is the most likely diagnosis? AAcne vulgaris BRosaceaCorrect Answer CSeborrheic dermatitis DSystemic lupus erythematosus
Correct Answer ( B ) Explanation: Rosacea is a common skin condition that has a variety of clinical signs and symptoms, including erythema, telangiectasia, skin coarseness, and formation of inflammatory papules and pustules resembling acne. Rosacea is defined by persistent erythema to the central portion of the face that lasts for at least three months. Criteria for diagnosis include the presence of flushing, papules, pustules, and telangiectasias. Other characteristics of rosacea include burning and stinging, edema, plaques, skin dryness, ocular involvement, and skin thickening and irregular surface nodulataries. It is common in fair-skinned persons of European and Celtic descent. The etiology of rosacea is unknown, although several factors may contribute to its development, such as vasculature (increased blood flow to the blood vessels of the face and increased numbers of blood vessels closer to the surface of the face), exposure to harsh climates (e.g., excess wind and ultraviolet light), chemical exposure, and ingestion of certain drugs (e.g., amiodarone, topical and nasal steroids, high doses of vitamins B6 and B12). Demodex species, which are mites that are normally found in human hair follicles, may play a role in rosacea, but this theory is controversial since these mites are found on many healthy individuals.Traditionally, spicy foods, alcohol, and hot beverages are thought to trigger flushing in patients with rosacea. Patients with rosacea likely have a long history of facial flushing dating back to childhood or adolescence. As adults, flushing may increase with exposure to certain triggers, such as foods, heat, emotions, and other causes of rapid temperature changes. Symptoms are usually intermittent at first but can lead to permanent skin flushing and development of telangiectasia. Rosacea has a spectrum of signs and symptoms, and patients may present in any stage of disease development. Some patients present with erythema and telangiectasia over the cheeks and forehead. Inflammatory papules and pustules resembling acne may be seen on the nose, forehead, and cheeks. Patients may develop thickened and disfigured noses (rhinophyma) in severe cases, which may occur as an isolated finding without other findings. Rarely, there may be inflammatory changes to the neck and upper chest as well. Unlike acne, patients will report dryness and peeling of the skin instead of skin greasiness. There is also an absence of comedones, which distinguishes rosacea from acne. Scarring is an uncommon occurrence. Patients may also have ocular involvement, including lymphedema to the periorbital areas, conjunctival injection, and rarely chalazion and episcleritis. Diagnosis is made clinically. A skin biopsy may be necessary to exclude other etiologies that may resemble rosacea including connective tissue diseases (e.g., lupus erythematosus, dermatomyositis), photosensitivity, polycythemia vera, long-term topical steroid use, and contact dermatitis. Rosacea fulminans is a rare complication characterized by the formation of nodules and abscesses with sinus tract formation accompanied by systemic signs, such as low-grade fever, elevated erythrocyte sedimentation rate, and an elevated white blood cell count. Before initiating treatment, patients should be advised to avoid any known exacerbating factors, such as hot or cold temperatures, wind exposure, caffeine, exercise, alcohol, hot drinks, and spicy foods, as well as stressors that may elicit strong emotions, topical products that may irritate the skin, and medications that cause flushing. Patients should also be advised to use sunscreen on a daily basis. Vascular lasers are the mainstay of rosacea therapy especially when the predominant symptom is erythema, since it reduces the number of vessels in the surrounding tissues. Nonablative laser is effective in treating rosacea but is very expensive and is often not covered by insurance. Cosmetic treatment of rhinophyma may be achieved by dermabrasion, laser peels, and surgical shaving. Pharmacotherapy is also effective for the treatment of this condition with topical metronidazole commonly used as a first-line agent. Topical erythromycin and clindamycin are other effective formulations. Other common medications include topical azelaic acid, sulfacetamide, and topical acne medications (e.g., benzoyl peroxide). Topical and oral antibiotics (e.g., metronidazole, doxycycline, clarithromycin) are also effective, used alone and in combination, especially for rosacea in the oral and ocular regions for which they are considered first-line. Retinoids can also be used for treatment, including topical tretinoin and oral isotretinoin. Immunosuppressants can also be used, including tacrolimus ointment, which reduces itching and inflammation. Corticosteroids are contraindicated except as a short course in rosacea fulminans, which is treated with high doses of prednisolone followed by oral isotretinoin. Antihypertensive agents are also used, including potassium-sparing agents (e.g., spironolactone, amiloride, triamterene) and topical alpha-agonists (e.g., topical oxymetazoline and topical brimonidine). Acne vulgaris (A) is characterized by the presence of open or closed comedones and inflammatory papules, pustules, and nodules. It is not associated with telangiectasias or thickening of the skin and is usually associated with greasy, not dry, skin. Seborrheic dermatitis (C) is characterized by mild, patchy scaling that can develop into widespread thick adherent crusts that may present over red and inflamed skin. The lesions develop intermittently and are usually seen in oily or hairy areas of the head and neck. They are not associated with telangiectasias or flushing. Malar rashes seen in systemic lupus erythematosus (D) can be mistaken for rosacea, but they are commonly found in a butterfly pattern over the cheeks and nasal bridge, sparing the nasolabial folds. The above patient also has lesions to his forehead and does not present with any other symptoms that may accompany systemic lupus erythematosus.
What finding differentiates rosacea from lupus erythematosus? AAbsence of Demodex folliculorum BExacerbation with heatCorrect Answer CImprovement with sun exposure DPresence of a malar rash
Correct Answer ( B ) Explanation: Rosacea is a common, chronic, relapsing, and remitting inflammatory skin disorder. It is most common in skin types I and II, and the prevalence of rosacea in these populations ranges from 1-10%. The pathogenesis of rosacea is not well understood but is thought to involve abnormalities in innate immunity, inflammatory reactions to cutaneous microorganisms such as the Demodex mite, ultraviolet damage, or vascular dysfunction. Risk factors for rosacea include people of Celtic and Northern European origin, sun exposure, and advancing age. Rosacea is a chronic relapsing condition for which there is no cure. There are four distinct subtypes of rosacea: erythematotelangiectatic, papulopustular, phymatous, and ocular rosacea. Rosacea is a complex disease process with multiple clinical manifestations that may fit different subtypes of the disease. Patients with rosacea may exhibit any of the following symptoms in isolation or in association with each other: centrofacial erythema with flushing or telangiectasias, noncomedonal papules or pustules, burning or stinging of the skin, cutaneous edema, or xerosis. Symptoms are often exacerbated with heat, sunlight, spicy foods, and alcohol. Phymatous changes may also develop and are most commonly seen in men. Phymatous rosacea presents as hypertrophic tissue manifesting as thickened, sebaceous skin with irregular contours. Phymatous changes are most often seen on the nose (rhinophyma) but may also affect the chin and glabella. Additionally, ocular symptoms are present in nearly 50% of patients with rosacea. These symptoms include burning or stinging of the eyes, light sensitivity, or a foreign body sensation. Patients may also develop chalazia, conjunctivitis, crusting on eyelashes, or lid margin irregularity. The diagnosis of rosacea is typically made based on reported symptoms and physical exam findings. Skin biopsies are rarely indicated since cutaneous histopathologic findings in rosacea are nonspecific. Skin biopsies may be performed to rule out other disorders. Treatment for rosacea should be focused on reducing associated symptoms. Topical therapies include metronidazole, azelaic acid, topical ivermectin, or sulfacetamide-sulfur. Systemic agents, such as oral tetracyclines, are typically used in patients who fail to improve with topical agents alone or who present with numerous inflammatory lesions. Patient education should be focused on avoiding the triggers of rosacea, which include extremes of temperature, sunlight, spicy foods, alcohol, exercise, and acute physiological stressors. Demodex folliculorum is a microscopic mite that lives and feeds on human skin. While Demodex folliculorum is often found in patients with rosacea, Demodex mites can be found on normal skin, and additionally, the absence of Demodex folliculorum (A) does not necessarily rule out rosacea. Therefore, it is not the best option in the case presented in the vignette above. Patients with cutaneous disorders such as psoriasis often see improvement with sun exposure (C). Sun exposure typically worsens rosacea and cutaneous lupus erythematosus. The presence of a malar rash (D) is common in lupus erythematosus. While rosacea can affect the malar area of the face, it most commonly involves the nose, chin, cheeks, and forehead. Additionally, rosacea does not spare the nasolabial folds.
Which of the following clinical manifestations is most supportive of erythema infectiosum? AFlu-like prodrome followed by a generalized vesicular rash BKoplik spots CMild febrile illness followed by a malar rash with a slapped-cheek appearance DPastia lines
Correct Answer ( C ) Explanation: Erythema infectiosum, also known as fifth disease, is a mild febrile illness caused by parvovirus B19. Erythema infectiosum commonly occurs as an outbreak among school-aged children, but it can occur in adults also. Transmission occurs by respiratory droplets, however, patients are no longer infectious by the time the rash develops. It is important for pregnant women to avoid contact with known outbreaks or potentially infected individuals. Erythema infectiosum begins with a nonspecific flu-like prodrome that includes fever, rhinorrhea, headache, malaise, nausea, and diarrhea. These nonspecific symptoms are caused by the onset of viremia. A classic erythematous malar rash occurs two to five days later. There is usually associated circumoral pallor, which gives the rash its classic slapped-cheek appearance. The facial rash may be followed by a reticulated or lacelike rash on the trunk. In immunocompetent children with classic symptoms, the presumptive diagnosis can be made based on clinical features alone without laboratory confirmation. Most patients feel well by the time the rash appears because viremia has resolved. The rash of erythema infectiosum usually resolves without treatment within a couple of weeks. There are no specific indicated therapies. Children may remain at school since the infectious stage has resolved by the time the rash occurs. In addition to erythema infectiosum, parvovirus B19 may cause arthropathy in older children and adults and can cause aplastic crisis in patients with sickle cell disease. Flu-like prodrome followed by a generalized vesicular rash (A) is consistent with varicella, also known as chicken pox. The prodrome consists of fever, malaise, myalgias, and anorexia. The prodrome of varicella is typically followed by a generalized vesicular rash within 24 hours. The rash usually begins on the trunk and spreads to the extremities. Koplik spots (B) are a characteristic finding of measles. Koplik spots are white spots occurring on an erythematous background on the buccal mucosa. Koplik spots are considered pathognomonic in measles and are often part of the prodrome that develops before the exanthematous rash. Pastia lines (D) are seen in patients with scarlet fever. Scarlet fever is a complication that can occur after untreated streptococcal pharyngitis. The rash presents as diffuse erythema that blanches and has overlying 1-2 mm papules, which give the rash a sandpaper quality. Other common features of scarlet fever include circumoral pallor, strawberry tongue, and Pastia lines. Pastia lines are linear petechial lines, which occur in the antecubital fossae or axillary folds.
A man presents to the clinic with rapid onset of painful and red swelling of the lateral nail fold of one of his fingers. There is no associated abscess or involvement of the pad of the finger. What is the best antibiotic treatment if coverage for methicillin-resistant Staphylococcus aureus is needed? AAmoxicillin BCephalexin CDicloxacillin DDoxycycline
Correct Answer ( D ) Explanation: Acute paronychia is an acute inflammatory process that involves the proximal and lateral nail folds and has been present for less than six weeks. It may occur with or without abscess formation. The etiologic pathogen is most commonly an organism present in the skin flora, such as Staphylococcus aureus or Streptococcus pyogenes. Acute paronychia occurs when these organisms are inoculated by disruption of the nail fold barrier. Oral flora may cause acute paronychia in individuals who bite their fingernails. Oral flora includes Eikenella corrodens, Fusobacterium, Peptostreptococcus, Prevotella, and Porphyromonas spp. Risk factors for acute paronychia include recent manicures, nail-biting, thumbsucking, and picking at a hangnail. Acute paronychia of the toes usually occurs in association with ingrown toenails. Acute paronychia classically presents with a rapid onset of painful erythema and swelling of the proximal and lateral nail folds. It usually occurs two to five days following minor local trauma or exposure to one of the risk factors. A superficial abscess is often present with acute paronychia. The infection of acute paronychia may occasionally extend along the proximal nail fold to the opposite side of the nail, which is called runaround infection. Complications of acute paronychia include spread of infection to the nail bed or pulp space, which is called felon. Felon is a bacterial infection of the fingertip pulp that requires prompt surgical treatment with incision and drainage to prevent the development of osteomyelitis, permanent nail deformities, and ischemic necrosis of the fingertip. Felon typically presents with severe pain, swelling, and erythema in the pad of the fingertip. The diagnosis of acute paronychia is usually straightforward and made clinically. Laboratory tests are usually not necessary. However, in patients with severe infection and abscess, cultures should be obtained to guide antimicrobial therapy. The treatment of acute paronychia includes local skin-care measures, topical or oral antibiotics, and surgical modalities depending on the severity of inflammation and presence or absence of abscess. Individuals with acute paronychia without abscess are typically treated with topical antibiotics and warm water or antiseptic soaks, such as with chlorhexidine or povidone and iodine, multiple times per day. The topical antistaphylococcal antibiotic should be applied after each warm soak. Topical antistaphylococcal antibiotics include triple antibiotic ointment and mupirocin. Each warm soak should last 10 to 15 minutes. Patients with more severe inflammation may require empiric oral antibiotic therapy with an antistaphylococcal agent, such as cephalexin or dicloxacillin. Patients with risk factors for methicillin-resistant Staphylococcus aureus should be treated with empiric oral therapy that covers methicillin-resistant Staphylococcus aureus, such as clindamycin, doxycycline, or trimethoprim-sulfamethoxazole. Patients with acute paronychia with abscess require incision and drainage. Incision and drainage are often performed by inserting a number 11 surgical blade under the affected cuticle margin. Appropriate local anesthesia can be obtained by a digital block. Partial nail plate removal is usually performed for the treatment of paronychia associated with ingrown toenails. Warm soaks are important after incision and drainage of acute paronychia to facilitate drainage of the abscess. Incision and drainage alone (without antibiotics) are adequate for most patients with acute paronychia with abscess formation. However, patients with severe cases or those who are immunosuppressed can be treated with empiric antistaphylococcal oral antibiotics. A pus culture is recommended for all drained paronychia. Amoxicillin (A) is a broad-spectrum penicillin that does not cover methicillin-resistant Staphylococcus aureus. Cephalexin (B) is a first-generation cephalosporin antibiotic that provides coverage against methicillin-sensitive Staphylococcus aureus. It does not cover methicillin-resistant Staphylococcus aureus. Dicloxacillin (C) is a penicillin antibiotic that provides coverage against methicillin-sensitive Staphylococcus aureus. It does not cover methicillin-resistant Staphylococcus aureus.
A patient presents to the urgent care complaining of the new-onset, swollen, pruritic rash shown above. The rash began as a few vesicles that appeared after gardening yesterday. Which of the following is the most likely diagnosis? ACellulitis BContact dermatitis CEczema DPsoriasis
Explanation: Contact dermatitis is a dermatologic condition caused by contact with an allergen. It is a delayed-type hypersensitivity reaction that results in eczematous dermatitis. Prior to skin outbreak, sensitization to the allergen is required. Common causes of allergic contact dermatitis include metals (e.g., nickel, chromate); rubber additives; preservatives or other additives in skin lotions, creams, or sunscreens; fragrances; dyes; nail polish additives; and topical medications. Poison ivy, sumac, and oak are the most common causes of allergic contact dermatitis in the United States. Clinical manifestations of contact dermatitis usually develop 14-21 days after exposure. If the patient has been previously sensitized to the particular allergen, inflammation typically occurs 12-48 hours after contact. Skin findings include vesicles, edema, and redness while the patient often complains of extreme pruritus and swelling. The rash of contact dermatitis originates at the area of direct exposure but recurrent contact may cause the rash to spread beyond these boundaries. Contact dermatitis caused by plants presents with linear streaks of vesicles and can progress to bullae. The most common sites of contact dermatitis are the hands, forearms, and face. Patch testing is performed by exposing localized areas of the skin to a broad panel of screening allergens, including occupational allergens and personal care products. This testing is used to evaluate patients with persistent or recurrent dermatitis despite appropriate treatment. The technique is conducted in a series of three visits with allergens applied in appropriate concentration in the first visit and removal, reading, and grading of the results in the second visit. The third visit should be scheduled to obtain a delayed reading to accurately identify all skin reactions. Primary treatment of contact dermatitis includes removing and avoiding known allergenic substances. Symptomatic relief from itching can be achieved with oatmeal baths and cool compress application. Topical treatment involves a corticosteroid ointment twice daily for two to three weeks. Potency of the corticosteroid depends on the body site being treated with low potency corticosteroids (e.g., alclometasone dipropionate) for the face; medium potency corticosteroids (e.g., flurandrenolide, triamcinolone acetonide) for the arms, legs, and trunk; and high potency formulations (e.g., betamethasone valerate, desoximetasone, clobetasol propionate) for the hands and feet. Ointments are preferred to cream formulations as the latter may have allergenic additives. Patients should be advised to discontinue all moisturizers, lotions, and topical medications during treatment. Plain petrolatum may be applied if needed for dryness. Severe or generalized allergic contact dermatitis can be treated with a three week tapering course of oral corticosteroids. Topical tacrolimus or pimecrolimus can be used as alternative topical medications for chronic or treatment-resistant contact dermatitis. Contact dermatitis can persist for up to three weeks after one exposure. Cellulitis (A) may present with induration, swelling, and erythema similar to the presentation of contact dermatitis but is generally painful and does not cause pruritus. The linear pattern of vesicles shown in the picture above makes the diagnosis of contact dermatitis secondary to plant contact more likely. The rash of contact dermatitis often mimics the skin findings of eczema (C), however, the patient in the vignette above displays a linear distribution that is characteristic of contact dermatitis rather than atopic dermatitis (eczema). The distribution of the rash can help distinguish these two conditions, with atopic dermatitis most commonly affecting generalized areas while contact dermatitis is localized to the site of direct exposure. Psoriasis (D) presents with scaly, lichenified plaques with associated pruritus. These plaques commonly manifest on the extensor surfaces of adults including the knees, elbows, as well as the gluteal cleft and the scalp. The clinical findings demonstrated in the vignette above do not indicate a psoriatic lesion.
A 75-year-old woman presents to the clinic with tense, fluid-filled blisters on the flexural surface of her arms and on her trunk. She states the blisters itch. Biopsy is obtained and shows antibodies attacking the epithelial cells of the basement membrane. What is the most likely diagnosis? ABullous pemphigoid BPemphigoid gestationis CPemphigus DUrticaria
Correct Answer ( A ) Explanation: Bullous pemphigoid is an autoimmune blistering disease that most commonly occurs in adults over the age of 60, and the risk increases with age. It is equally common in men and women. The pathogenesis of bullous pemphigoid involves antibodies attacking the basement membrane below the epidermis, which leads to subepidermal blistering. It is unknown what causes individuals to be affected by bullous pemphigoid. The classic clinical presentation of bullous pemphigoid is tense (not easily ruptured), fluid-filled bullae on the skin that are extremely pruritic. These bullae eventually rupture and result in moist erosions with crusting. Common locations for cutaneous involvement include the flexural surfaces, trunk, and axillary and inguinal folds. Some individuals have prodromal symptoms that occur weeks or months prior to the onset of bullous pemphigoid. These prodromal findings may include pruritus, papules, urticaria-like lesions, and eczematous lesions. Mucosal involvement occurs in 10-30% of patients with bullous pemphigoid, and oral erosions are the most common mucosal presentation. Patients suspected of having bullous pemphigoid should have skin or mucous membrane biopsies performed. Direct immunofluorescence staining of a biopsy highlights antibodies along the basement membrane that lies between the epidermis and dermis and is considered the gold standard for diagnosis. The management of bullous pemphigoid depends on the amount of skin surface area involved. High-potency topical corticosteroids, such as clobetasol propionate, are recommended for patients with limited disease. Systemic corticosteroids, such as prednisone, are also highly effective. Corticosteroid-sparing immunosuppressants may be used in patients who cannot tolerate corticosteroids. Furthermore, many patients initially treated with systemic corticosteroids are transitioned to other immunosuppressive agents due to the duration of treatment required with bullous pemphigoid since bullous pemphigoid often persists for years. Immunosuppressants commonly used in the treatment of bullous pemphigoid include methotrexate, cyclosporine, and azathioprine. Complications of bullous pemphigoid include staphylococcal or streptococcal skin and soft tissue infections. Antibiotics are used if secondary bacterial infections are suspected. Bullous pemphigoid often requires treatment for years, although symptoms typically show significant improvement within months. The level of BP180 antibodies in serum usually correlates with the clinical activity of bullous pemphigoid. Therefore, these antibodies are often used to monitor response to treatment. Pemphigoid gestationis (B) is an autoimmune subepidermal blistering disorder that occurs during pregnancy or the postpartum period. The typical presentation is intensely pruritic papules, urticarial plaques, vesicles, and bullae that begin on the abdomen prior to spreading to other areas. The face and mucous membranes are typically spared. The patient in this vignette was past child-bearing years. Pemphigus (C) represents a group of autoimmune blistering disorders. However, the blisters in pemphigus are flaccid in nature, which contrasts with the tense bullae seen in bullous pemphigoid. Urticaria (D) can be due to a variety of causes, including medications, foods, insect stings, or infections. Patients typically present with well-circumscribed plaques that may have central pallor. The lesions are usually transient, appear quickly, and resolve within 24 hours. Blistering lesions are atypical for urticaria.
A 22-year-old woman presents to the emergency department complaining of a burn to her right lower arm. She states she accidentally spilled a pot of boiling water on her arm while cooking. On physical exam, there is a large blister with surrounding erythema approximately 4 centimeters in diameter on the volar aspect of the patient's lower right arm. The blister is easily unroofed when palpated and reveals mottled white and red tissue that does not blanch with pressure. Which of the following best describes this patient's burn? ADeep partial-thicknessC BFull-thickness CSuperficial DSuperficial partial-thickness
Correct Answer ( A ) Explanation: Burns are thermal injuries to the epidermis and can extend to the dermis and other deeper structures, depending on their severity. Treatment is based on burn depth and percentage of body surface area affected, necessitating accurate measurement and classification of burn size and depth. New nomenclature has replaced the classification of burns as first, second, or third degree. Currently, burns can be described as superficial, superficial partial-thickness, deep partial-thickness, full-thickness, and fourth-degree. Superficial burns involve only the epidermis. They present as painful areas of erythema that blanch when pressure is applied and do not blister. Superficial burns heal without scarring and can be treated with aloe, ice application, and by keeping the area clean. Superficial partial-thickness burns involve the epidermis and some areas of the dermis. They present as painful areas of erythema that blanch with pressure and blister. Treatment of superficial partial-thickness burns involves debriding the wound, applying silver sulfadiazine or other antimicrobial creams, and keeping the wound covered with bandages that will protect the wound while also promoting re-epithelialization. Superficial partial-thickness burns typically heal without scarring. Deep partial-thickness burns involve the epidermis and the dermis. They blister, do not blanch, and involve loss of hair follicles. The area will appear mottled white and red, and there may be hypoesthesia. Full-thickness burns are similar to deep partial-thickness wounds except that they extend to all layers of the dermis and do not blister. Their appearance can be waxy and white, gray and leathery, or charred and black. Hypoesthesia or complete anesthesia is noted, and the lesions do not blanch. Deep partial-thickness and full-thickness burns produce extensive scarring and should be treated with excision and grafting. Fourth-degree burns describe full-thickness burns that extend beyond the dermis to deeper structures, such as subcutaneous fat, muscle, or bone, and are often life-threatening. Determining the extent of body surface area burned is important when performing a physical exam on a burn patient, as this also determines treatment. Because they are self-limited, superficial burns are excluded regardless of their size. The rule of nines is often used to estimate the percentage of body surface area burned. Another classification system for body surface area involves the Lund-Browder chart, which takes into account the patient's age. If the patient has burns to greater than 10% of their body surface area, or if the burns involve the face, hands, genitalia, feet, perineum, or major joints, the patient should be referred to a burn treatment center. Full-thickness burns (B) appear similar to this patient's burn but they do not blister. Superficial burns (C) are erythematous, do not blister, and will blanch when pressure is applied to them. Superficial partial-thickness burns (D) blanch with pressure. They are erythematous, not mottled, and often weeping.
What etiologic agent is the most common cause of hand, foot, and mouth disease? ACoxsackievirus BEchovirus CEnterovirus DHerpes simplex virus 1
Correct Answer ( A ) Explanation: Hand, foot, and mouth disease is a self-limited, acute viral exanthem that causes vesicular eruption that usually presents in the mouth (oral enanthem) and can involve the hands, feet, buttocks, and genitals. Most cases of hand, foot, and mouth disease are caused by coxsackievirus A type 16, although there are multiple associated strains as well. Enterovirus 71 has also been known to cause some outbreaks of hand, foot, and mouth disease with neurologic involvement. Infection is transmitted fecal-orally or from contact with skin lesions and oral secretions. Epidemics of hand, foot, and mouth disease occur commonly in the summer to early fall months, with risk factors for transmission including attendance at child care centers, contact with affected individuals, and large family numbers. Children younger than 10 years of age are most commonly affected with hand, foot, and mouth disease, and the disease generally lasts for about seven to 10 days, though the patient may continue to shed the virus through the stool for several weeks. Patients will present with a sore throat, sore mouth, and malaise. Rarely, patients with hand, foot, and mouth disease caused by enterovirus 71 will have vomiting. Initially, patients will have macular lesions on the buccal mucosa, tongue, and hard palate that develop into vesicles that erode and are surrounded by an erythematous halo. These same lesions may also develop on the palms of the hands and soles of the feet, as well as the buttocks, and genitalia. Patients may also have a fever lasting about 24-48 hours. Very rarely, patients may develop aseptic meningitis with certain coxsackievirus strains as well as with enterovirus 71 infection. Other neurologic complications in cases of hand, foot, and mouth disease caused by enterovirus 71 include encephalitis, encephalomyelitis, acute cerebellar ataxia, acute transverse myelitis, Guillain-Barré syndrome, and benign intracranial hypertension. Diagnosis is made clinically. There may be a benefit to isolating and identifying the causative virus (e.g., differentiating coxsackievirus from enterovirus 71) to determine the patient's prognosis but is otherwise unnecessary. Since there is no antiviral agent specific to hand, foot, and mouth disease, treatment is supportive. Patients are encouraged to have adequate fluid intake since dehydration is the most common complication of hand, foot, and mouth disease. If a patient is unable to have adequate oral intake, intravenous hydration may be necessary. Pain can be treated with over-the-counter medications, such as oral acetaminophen or ibuprofen, numbing mouthwashes, and oral sprays. Fever may be treated with antipyretics. Patients with central nervous system manifestations of hand, foot, and mouth disease may require hospitalization. The prognosis of hand, foot, and mouth disease is excellent, and patients are expected to fully recover. Certain echovirus (B) strains can cause herpangina, which is characterized by papules or vesicles that ulcerate and form on an erythematous base that can affect the tonsils, soft palate, and uvula. It generally affects the posterior oral cavity. Enterovirus (C) 71 is a rare strain, not the most common strain of hand, foot, and mouth disease, and is associated with neurologic complications. Herpes simplex virus 1 (D) causes herpetic gingivostomatitis, which is characterized by vesicular eruptions to the mouth, lips, gingiva, and buccal mucosa that can ulcerate and cause moderate to severe pain. It generally affects the anterior oral cavity.
A 45-year-old man presents to the emergency department with complaints of a skin problem on his right foot. There is no history of injury, however, he was camping two days ago and wonders if it could be a spider bite or poison ivy. He has a low-grade fever, but his vital signs are otherwise stable. His exam is notable for right foot dorsal edema and expanding erythema that is warm and tender to palpation and extends from the foot to the ankle and lower leg. The affected area is about 8 cm in diameter but has poorly defined borders. He has full range of motion in the joints of his lower extremity, and there is focal tenderness only over the area of swelling and erythema. He has a few scattered superficial abrasions on both legs. His left leg has a normal exam. Which of the following is the most likely diagnosis? AAcute gout BCellulitis CContact dermatitis DSpider bite
Correct Answer ( B ) Explanation: Cellulitis is a common skin and soft tissue infection that involves the deeper dermis and subcutaneous fat. Skin and soft tissue infections that may appear similar include erysipelas, which affects the upper dermis and lymphatics, and abscess, which is a collection of pus within the dermis or subcutaneous fat. Risk factors for cellulitis include skin barrier disruption, skin inflammation, obesity, immunosuppression, edema due to lymphatic or venous insufficiency, and coexisting skin infections. The most common pathogens associated with cellulitis are beta-hemolytic streptococci and Staphylococcus aureus. Classical findings in cases of cellulitis include spreading skin erythema, edema, and warmth with poorly demarcated borders. Cellulitis is almost always unilateral and most commonly affects the lower extremities. Affected skin may have petechiae, vesicles, bullae, or edema around hair follicles creating an orange peel (peau d'orange) texture. Patients may also present with fever and other systemic symptoms of infection, such as chills, malaise, and headache. Regional lymph nodes may also be tender and enlarged. Diagnosis is clinical, and laboratory testing and imaging are not indicated for uncomplicated cases. In patients with severe, recurrent, or toxic systemic symptoms, underlying causes of cellulitis, such as cellulitis due to unusual (Clostridia and other anaerobes) or resistant organisms, or comorbidities, such as osteomyelitis, septic arthritis, septic bursitis, and immunosuppression, should be considered. The differential diagnosis associated with cellulitis is broad and includes infectious as well as noninfectious causes. History and exam are key components to narrowing the diagnosis. Treatment of cellulitis includes antibiotic treatment, such as cephalexin, and adjunctive recommendations, such as elevating the affected area and using warm compresses, with close monitoring for clinical improvement. If resistant bacteria are suspected due to abscess or failure of prior treatment, clindamycin, doxycycline or trimethoprim-sulfamethoxazole are good oral options to cover methicillin-resistant Staphylococcus aureus. Providers should consult local antibiograms regarding the most relevant antibiotic treatment options for the organisms causing cellulitis. Patients with underlying comorbidities, which can lead to recurrence or complication, should have these issues addressed as well. With appropriate treatment, cellulitis should show improvement within 48 hours of therapy and resolve without complications after 5-10 days (dependent on the underlying status of the patient, body site, and extent of involvement) of treatment. Patients who do not improve or have high fevers, increasing involvement including streaking cellulitis, or cellulitis overlying joints should be evaluated in a hospital setting and may require admission for intravenous antibiotics. Acute gout (A) presents similarly to cellulitis, in that patients will report severe pain, warmth, edema, and swelling. However, gout will present with these findings over a single joint, most commonly the first metatarsophalangeal joint. Patients with gout will likely have a prior history of similar arthritic attacks and tophi on exam. Contact dermatitis (C) will present as pruritic lesions. While they may be erythematous and edematous, the lesions are also likely to have associated oozing vesicles and bullae. There may also be clues related to the shape and size of the lesion, as the dermatitis is usually limited to the area of direct contact with a trigger. Spider bite (D), like many insect bites, triggers an inflammatory reaction. Usually the reaction can be noted within minutes of the bite and is likely to be pruritic in addition to presenting with local edema and erythema. In some cases, there may be a delayed skin reaction.
Which cutaneous distribution of lesions differentiates pityriasis rosea from other dermatoses? AAcral distribution BChristmas tree pattern distribution CExtensor surface distribution DZosteriform distribution
Correct Answer ( B ) Explanation: Pityriasis rosea is an acute, self-limited, papulosquamous, erythematous skin eruption that first appears on the trunk and spreads to the proximal extremities. Pityriasis rosea is often considered a viral exanthem (possibly human herpesvirus or influenza) since it typically appears after a prodrome of symptoms and appears in clusters. It has not been shown to be associated with bacterial or fungal organisms. Pityriasis is typically found in children and young adults and may also be seen in patients with decreased immunity, such as pregnant women and transplant recipients. Pityriasis rosea is a self-limiting disease, and the rash will typically resolve within several months. It is not a scarring disease, however, patients of deeper skin pigmentation may experience postinflammatory hyperpigmentation. The prodrome of symptoms prior to the cutaneous outbreak of pityriasis rosea is vague but may include fatigue, fever, headache, sore throat, joint pain, nausea, or irritability. Pityriasis rosea often has a duration of six to eight weeks. It evolves rapidly, usually beginning with a "herald patch," a 2-5 cm single, round, sharply demarcated, pink- or salmon-colored lesion on the chest, neck, or back. The primary plaque is seen on the skin in 50-90% of cases one week or more before the onset of the eruption of smaller lesions. Soon after onset, the herald patch becomes slightly scaly and begins to clear centrally, producing a collarette of scale on the periphery of the lesion. The secondary, more widespread eruption occurs 2 to 21 days later. These lesions are similar to the herald patch but are much smaller and greater in number. They occur in crops following the cleavage lines of the skin. On the back, this eruption produces a "Christmas tree" pattern. The lesions are rarely symptomatic, although some patients may complain of mild pruritus that is worse when the body overheats, such as during exercise or a shower. The diagnosis of pityriasis rosea is primarily clinical. A herald patch without secondary eruption may be difficult to differentiate from tinea corporis, so a potassium hydroxide examination of scales for dermatophytes may be helpful. Since cutaneous manifestations of the secondary eruption may be difficult to differentiate from syphilis, serologic testing may be indicated to rule out syphilis as the causative factor. A skin biopsy is rarely indicated but may be performed to help confirm the diagnosis. The treatment for pityriasis rosea is purely symptomatic. The disease is self-limiting and will typically resolve without treatment within two to three months. There is a low likelihood of transmission, and it does not recur in most patients. Symptomatic therapy should focus on increasing patient comfort. Topical steroids and antihistamines may be considered to improve pruritus. There is some evidence that oral acyclovir may accelerate clinical improvement, although efficacy data is limited. An acral distribution (A) refers to the area on the bottom of the feet or palms. Very few dermatoses involve acral distribution. Hand, foot, and mouth disease is one example of acral distribution of cutaneous lesions. Extensor surface distribution (C) is most commonly associated with psoriasis or dermatitis herpetiformis and is not often associated with pityriasis rosea. Zosteriform distribution (D) describes a unilateral, streak-like pattern following the skin dermatomes. This cutaneous distribution is most commonly seen with herpes zoster. It is rarely a distribution associated with pityriasis rosea.
A 2-year-old boy presents to the clinic with superficial ulcers on his tongue and buccal mucosa. There are also erythematous macular lesions on his palms and soles. His mom states he has had a subjective low-grade fever but no other symptoms. He is tolerating oral fluids. Which of the following is the recommended management for the most likely diagnosis? AAcyclovir BAmoxicillin CIbuprofen DTopical lidocaine
Correct Answer ( C ) Explanation: Hand, foot, and mouth disease is a viral illness most often caused by the coxsackie virus, which is an enterovirus. Hand, foot, and mouth disease occurs most often in children younger than 5-7 years but may occur in any age group. Hand, foot, and mouth disease occurs most often during the summer and early fall. Transmission occurs primarily by the fecal-oral route but may also occur by oral or respiratory secretions and through vesicular fluid. Hand hygiene is very important, because the virus may be shed in the stool for weeks and fomite transmission is possible. The most common symptoms include mouth or throat pain, low-grade fever, and cutaneous manifestations. There often is not a prodrome. Oral lesions are typically painful and most commonly occur on the tongue or buccal mucosa. Skin lesions are usually nonpruritic and nontender and may present as macular, maculopapular, or vesicular lesions. Lesions occur most often on the hands, feet, buttocks, or extremities. Lesions from hand, foot, and mouth disease usually resolve spontaneously within one week. Complications, including meningitis and encephalitis, are rare. The diagnosis of hand, foot, and mouth disease is made clinically based on the appearance and location of the lesions. The differential diagnosis of hand, foot, and mouth disease includes herpangina, aphthous ulcers, and herpes simplex gingivostomatitis. The management consists primarily of supportive care and patient education. Ibuprofen or acetaminophen may be used to treat pain and fever. Topical lidocaine is not recommended due to unproven benefit and possible systemic toxicity. There are no antiviral medications that can be used to treat hand, foot, and mouth disease. It is important for children with hand, foot, and mouth disease to stay hydrated. Children who cannot take in any oral fluids may need to be admitted to the hospital. Acyclovir (A) is an antiviral medication. Enteroviruses, such as the coxsackie virus, do not have enzymes that acyclovir acts on and are, therefore, ineffective. Acyclovir may be used in the treatment of gingivostomatitis caused by the herpes virus. The lesions on the hands and feet of this patient make hand, foot, and mouth disease the most likely diagnosis. Amoxicillin (B) is a broad-spectrum beta-lactam antibiotic. Amoxicillin is often used in the treatment of streptococcal pharyngitis. Topical lidocaine (D) could be used to alleviate pain associated with oral lesions but is not recommended in hand, foot, and mouth disease due to unproven benefit and known risks of systemic toxicity.
Which of the following is the causative organism for pityriasis versicolor? AAspergillus BCandida CCorynebacterium DMalassezia
Correct Answer ( D ) Explanation: Malassezia is a lipid-dependent yeast that is the causative organism for pityriasis versicolor. Also known as tinea versicolor, pityriasis versicolor is a superficial, dermatologic fungal infection that is found more commonly in tropical climates but can occur anywhere in the world. Adolescents and young adults are affected more commonly than older adults, young children, or infants. Risk factors for developing pityriasis versicolor include hot and humid weather, the use of topical skin oils, and hyperhidrosis. There have been studies that show there may be a genetic predisposition to developing the condition. Patients will typically present with skin color changes. These changes result in either hyper- or hypopigmented macules on the upper trunk, shoulders, and proximal upper extremities. Diagnosis is typically clinical but can be confirmed with a potassium hydroxide (KOH) preparation showing hyphae and yeast cells. First-line therapy consists of topical preparations containing an azole antifungal or selenium sulfide. Oral azole antifungal therapy is reserved for those patients with widespread symptoms or those resistant to topical treatments. Prevention may be accomplished through the use of topical selenium sulfide or ketoconazole prior to warmer months when symptoms may become more prevalent. Aspergillus (A) is an opportunistic fungal infection that can have pulmonary or dermatologic manifestations but is not a causative organism for pityriasis versicolor. Candida (B) can cause yeast dermatitis. Corynebacterium (C) is the cause of erythrasma, which is typically found in intertriginous areas.
Which of the following medications is first-line treatment for pediculosis capitis? AOral ivermectin BTopical lindane CTopical mupirocin DTopical permethrin
Correct Answer ( D ) Explanation: Pediculosis capitis is a common condition that is caused by Pediculus humanus capitis (the head louse) infestation of the hair and scalp. Pediculosis capitis occurs worldwide and in individuals of all socioeconomic backgrounds. However, children are affected most frequently. African American children are affected far less frequently than Caucasian children, which is thought to be due to lice being less able to grasp the hair type. Pruritus occurs as an allergic reaction to lice saliva injected during feeding. The life span of the female louse is approximately one month. Each day, the female louse lays about seven to 10 eggs that are firmly cemented to the base of a host hair. The eggs (commonly called nits) are oval capsules that hatch in eight days and release nymphs (immature louse). The nymphs require eight days to mature and become adult lice. The hatched eggs (nits) are typically more visible than unhatched eggs. Adult head lice are gray-white in color and 2 to 3 millimeters in length. Adult lice feed on the scalp and adjacent areas of the face and neck. Adult lice can survive up to 55 hours without a host, but it is thought that they typically dehydrate and become nonviable long before that. Direct contact with the head of an infested person is the primary mode of transmission. Lice are not transmitted via jumping, flying, or using pets as vectors. It is controversial whether lice can be spread by contaminated inanimate objects, such as hats, towels, or sheets. Lice presents with itching of the scalp. Symptoms may be delayed four to six weeks in patients without prior sensitization. Other findings include the presence of lice, nits, and excoriations on the scalp and postauricular skin. Complications may include secondary bacterial infection, which is usually caused by Staphylococcus aureus. The diagnosis should be suspected in patients with scalp pruritus, especially children. However, the diagnosis is confirmed by visualization of live lice. Systematically combing wet or dry hair with a fine-toothed nit comb is a more sensitive measure for detecting active louse than visual inspection of the hair and scalp alone. Nits are often easier to identify than nymphs or adult louse. However, finding nits without lice does not confirm active infection, and nits often persist after successful therapy. Nits within 0.25 inches from the scalp are likely viable, however, nymphs that are further away are less likely to be viable. Topical pediculicides are the most common initial treatment. Manual removal of lice (with wet combing) is sometimes used as an alternative to topical pediculicide therapy. Oral therapy is occasionally required for refractory cases. It is important to confirm the diagnosis by detecting living lice prior to treatment. Acceptable topical pediculicides include pyrethroids (permethrin or pyrethrins), malathion, benzyl alcohol, spinosad, and topical ivermectin. Topical pediculicides should be rinsed in a sink rather than a shower or bath to limit skin exposure. The most common causes of treatment failure are lack of adherence to treatment and continued contact with other infested individuals. Oral ivermectin is a treatment option for patients who fail topical therapy. Children should not be excluded from school based on the presence of live lice or nits. However, affected children should avoid direct head contact with other individuals and be treated promptly. Household members of an infested individual should be examined and treated if infested (defined as live lice or nits within 1 centimeter of the scalp). Washing clothing and linen used by the infested person during the two days prior to therapy in hot water or drying the items on a high-heat dryer cycle is recommended. Vacuuming the furniture and carpet on which the infested person sat or laid down is also recommended. Oral ivermectin (A) is an oral pediculicide. It works by interfering with neurotransmission in arthropods and helminths. It is considered second-line treatment for pediculosis capitis for individuals who fail first-line topical pediculicides. Oral ivermectin is not recommended in pregnant women or children weighing less than 15 kilograms. Topical lindane (B) is not recommended as first-line treatment for pediculosis capitis because of safety concerns (there have been case reports of seizures and death). Topical mupirocin (C) is a topical antibiotic commonly used in the treatment of impetigo. It is not used for pediculosis capitis.
A 32-year-old woman presents with concern for a new rash on both of her elbows. The lesions are not painful or itchy. She is otherwise well with no other new symptoms or recent illnesses. On exam, well-demarcated erythematous plaques with silvery scales are noted, which are concerning for psoriasis. Which of the following is likely the first step in the pathogenesis of this disease? AKeratinocytes that produce an inflammatory response BProduction of cytokines CUp-regulation and differentiation of T cells DUp-regulation of dendritic cells
Correct Answer ( D ) Explanation: Psoriasis is a common but complex chronic inflammatory disease whose pathogenesis is being slowly elucidated through an increased understanding of the immune response. The dermatologic findings of psoriasis are characterized by well-defined erythematous plaques with silvery scales, classically located on extensor surfaces. The scalp and fingernails are also frequently affected. About one-third of patients with psoriasis are also affected by psoriatic arthritis. Additionally, psoriatic patients are also at increased risk for a number of comorbidities including cardiovascular disease, diabetes, eye disorders, hypertension, inflammatory bowel disease, and metabolic syndrome. These comorbidities are believed to be associated with psoriasis due to a number of factors including chronic inflammation, systemic treatments for psoriasis, and lifestyle. The immune cascade of psoriasis is thought to begin in susceptible individuals due to some type of antigenic trigger (such as a medication, infection, or trauma, as well as lifestyle factors like alcohol and cigarette smoking), which causes up-regulation of dendritic and other innate immune cells to produce proinflammatory cytokines. These proinflammatory cytokines in turn lead to up-regulation of myeloid dendritic cells in the skin to further produce cytokines. One of the most notable of these is interleukin-23, which stimulates the activation and differentiation of T cells. The T cells then produce additional cytokines including interleukin-17A and interleukin-22, which promote keratinocyte up-regulation and growth. It is the keratinocytes that are thought to sustain the positive feedback loop of the inflammatory response seen in psoriasis as they produce cytokines and chemokines. As understanding of the pathophysiology of psoriasis develops, targeted treatments to combat the immune response associated with the disease are also being developed. For example, vitamin D and retinoids are able to disrupt keratinocyte production of cytokines and chemokines. Medications like cyclosporine inhibit global T cell responses, inhibiting their production of cytokines. And biologic agents including tumor necrosis factor (TNF)-alpha inhibitors, may block the activation of T cells and induce T cell death. Psoriatic patients usually start treatment with a combination of topical and systemic therapies such as topical steroids, calcineurin inhibitors, vitamin D and vitamin A analogues, and methotrexate, advancing treatment to biologic agents as needed. Outpatient treatment is usually sufficient but patients who develop pustular disease and psoriatic arthritis require more aggressive treatments and sometimes even hospitalization. Most patients will respond well to treatment and as a result, serious complications of psoriasis are now uncommon. Keratinocytes produce an inflammatory response (A) by producing cytokines and chemokines. These actions contribute to a positive feedback loop that sustains the inflammatory response seen in psoriasis. Production of cytokines (B) is accomplished by activated dendritic cells. These cytokines then trigger the inflammatory response, which leads to the sequelae associated with psoriasis. Up-regulation and differentiation of T cells (C) are stimulated by cytokines, especially interleukin-23. These cytokines must first be produced by myeloid dendritic cells.
A 30-year-old woman presents to the clinic with an erythematous lesion on her trunk. The lesion has central clearing with an advancing peripheral border. It is erythematous, scaling, and pruritic. Potassium hydroxide preparation shows segmented hyphae. Which of the following is the most likely diagnosis? APityriasis rosea BPsoriasis CSecondary syphilis DTinea corporis
Correct Answer ( D ) Explanation: Tinea corporis is a cutaneous dermatophyte infection that occurs on sites other than the feet, groin, face, or hand. The most common microbial cause is Trichophyton rubrum. Transmission of infection may occur by direct skin contact with an infected individual or animal, by transmission from other sites of infection (scalp, feet, etc.), or by contact with fomites. Tinea corporis presents as an erythematous, circular or oval, and scaling patch or plaque that spreads centrifugally. Central clearing typically occurs while an active and advancing raised border remains. The advancing border with central clearing results in an annular plaque from which the disease derives its other name, "ringworm." Extensive tinea corporis should raise concern for an immunocompromising condition, such as human immunodeficiency virus. The diagnosis of tinea corporis is confirmed by potassium hydroxide preparation, which shows segmented hyphae characteristic of dermatophyte infections. The skin scrapings used for the potassium hydroxide preparation should be taken from the active border of a plaque. There are a variety of dermatologic conditions that present similarly to tinea corporis. Clinical features that should cause consideration of alternative diagnoses include extensive skin involvement, absence of scale, failure to respond to antifungal therapy, and a negative potassium hydroxide preparation. Tinea corporis is treated with topical antifungals, such as azoles, ciclopirox, and tolnaftate. Topical nystatin is not effective for dermatophyte infections. Topical antifungal treatment is generally administered once or twice per day for one to three weeks until symptom resolution occurs. Systemic therapy is an alternative for patients with extensive skin involvement and patients who fail topical therapy. Oral antifungal agents include terbinafine, itraconazole, griseofulvin, and fluconazole. In addition to tinea corporis, dermatophyte infections can manifest as tinea pedis (dermatophyte infections of the foot), tinea cruris (dermatophyte infection involving the crural fold), tinea capitis (dermatophyte infection of the scalp hair), and tinea barbae (dermatophyte infection involving the beard hair). Tinea capitis must be treated with systemic antifungal agents. Pityriasis rosea (A) usually initially presents with a herald patch. The herald patch is a single round or oval patch that is pink or salmon-colored and typically occurs on the chest, back, or neck. The herald patch is usually sharply delineated and 2-5 centimeters in diameter. The herald patch is then followed by smaller but similar appearing lesions in a Christmas tree distribution (along the lines of cleavage of the skin). The herald patch presentation is very similar to tinea corporis. However, the vignette states that the potassium hydroxide preparation revealed segmented hyphae, which confirms the diagnosis of tinea corporis. Psoriasis (B) most commonly presents as erythematous plaques with an overlying silvery scale on extensor surfaces. Psoriasis may also be associated with nail changes and psoriatic arthritis. Psoriasis lacks central clearing and an advancing border. Secondary syphilis (C) can manifest as a diffuse and symmetric maculopapular eruption involving the entire trunk and extremities, including the palms and soles. If syphilis is suspected, then the diagnosis must be confirmed with serologic testing.
What percentage of the total body surface area must be involved to diagnose toxic epidermal necrolysis? A10% B15% C20% D30%
Correct Answer ( D ) Explanation: Toxic epidermal necrolysis is a life-threatening mucocutaneous disease most commonly precipitated by medication use. The most frequently implicated drugs include sulfonamides (e.g., sulfamethoxazole, sulfasalazine), anticonvulsants (e.g., lamotrigine, phenytoin), nonsteroidal anti-inflammatory drugs (e.g., piroxicam, tenoxicam), and allopurinol. Patients with HIV infection, autoimmune conditions (e.g., systemic lupus erythematosus), malignancy, and genetic factors are at an increased risk for developing this condition. Recent immunization with diphtheria-pertussis-tetanus, measles, poliomyelitis, and influenza vaccinations have also been associated with the development of toxic epidermal necrolysis. Additionally, it may manifest due to infection with agents such as cytomegalovirus, Mycoplasma species, Streptococcus species, syphilis, and tuberculosis, among others. While the exact pathogenesis of toxic epidermal necrolysis is unknown, the marked skin sloughing is suspected to be caused by an increase in keratinocyte apoptosis, which is mediated by the cell-surface death receptor (Fas) and its ligand (FasL). The mortality rate for this condition ranges from 30-50% and is nearly 100% in patients with toxic epidermal necrolysis from acute graft-versus-host-disease. The condition is characterized by widespread blistering and sloughing of the skin and mucous membranes. The rash begins with diffusely red, "sunburn-like" tender skin with scattered target lesions and bullae. Bullae coalesce as the disease progresses, and widespread skin sloughing results from full-thickness epidermal necrosis and detachment. Toxic epidermal necrolysis exhibits Nikolsky sign, which is epidermal detachment secondary to gentle lateral pressure. Toxic epidermal necrolysis and Stevens-Johnson syndrome are differentiated by the percent of total body surface area involved with the former including over 30% and the latter including less than 10%. These conditions overlap in patients with skin detachment of 10-30% of total body surface area. Prior to skin manifestations, patients may develop a high fever with constitutional symptoms, such as malaise, cough, and abdominal pain. Multiple organ systems may be affected, with pulmonary manifestations including pneumonitis, bronchitis, and bronchiolitis obliterans. Seizures and coma may also be present, and renal failure is common. Due to the widespread breakdown of the protective skin barrier, patients are at increased risk for sepsis and electrolyte and fluid imbalances. Laboratory testing should aim to identify these complications using complete blood count with differential, complete metabolic panel, erythrocyte sedimentation rate, and C-reactive protein. Chest radiograph should be performed in all patients to rule out pulmonary complications. Skin biopsy can distinguish between toxic epidermal necrolysis and staphylococcal scalded skin syndrome based on the thickness and severity of epidermal damage. Direct immunofluorescence can aid in ruling out autoimmune blistering conditions, such as paraneoplastic pemphigus. Patients with widespread blistering, neutropenia, impaired renal function, multiple medications, and those of old age have a worse prognosis. Treatment goals include pain control using non-opioid analgesics, such as ibuprofen or acetaminophen. Opioid analgesics are indicated for more severe pain control. Sources of infection should be identified and treated. Suspected offending medications should be withdrawn, and patients require meticulous wound care. Patients with severe involvement should be treated in a burn center. If administered early, intravenous immunoglobulin G may improve survival. Systemic corticosteroids (prednisone) may halt progression of the condition but increase the risk for infection. Complications include massive loss of fluids with resultant electrolyte imbalance and hypovolemic shock with renal failure. Bacteremia and insulin resistance can occur as well as multiple organ dysfunction syndrome and a hypercatabolic state. Patients with total body surface area involvement of less than 10% (A) are diagnosed with Stevens-Johnson syndrome. This condition is three times more prevalent than toxic epidermal necrolysis. Total body surface area involvement of 15% (B) or 20% (C) falls within the overlapping diagnostic criteria for Stevens-Johnson syndrome and toxic epidermal necrolysis. Patients in this classification have a worse mortality rate than patients with less than 10% total body surface area involvement but have a better mortality rate than patients diagnosed with toxic epidermal necrolysis.
Which of the following is the best diagnosis of the condition pictured above? (flip) AAtopic dermatitis BPsoriasis CSeborrheic dermatitis DTinea capitis
Explanation: Seborrheic dermatitis is a common, chronic inflammatory papulosquamous disease that can affect all age groups. Malassezia yeast is the suspected cause. Infantile seborrheic dermatitis, often called cradle cap, most commonly occurs between 3 weeks and 12 months of age and affects 10% of children within the first month of life. It presents as a yellow, greasy adherent scale on the vertex of the scalp with minimal underlying erythema. The lesion may become thick and adherent as scale accumulates. Seborrheic dermatitis can affect the diaper and axillary areas of infants and may present with secondary bacterial or candidal infection. In older children and adults, seborrheic dermatitis presents as greasy, moist, transparent to yellow papules with scaling with red patches and plaques. The scalp margins, central face, and presternal areas are the most common areas involved due to the maximal concentration of sebaceous glands in these locations. This condition also characteristically affects the eyebrows, base of the eyelashes, nasolabial folds, paranasal skin, and external ear canals. Fungal culture and potassium hydroxide scrapings can aid in ruling out a tinea infection that presents with similar scale. Skin biopsy is not required in the diagnosis of seborrheic dermatitis. Infantile seborrheic dermatitis is self-limited with treatment involving gentle removal of scale using warm mineral or olive oil and a low-strength corticosteroid (e.g., hydrocortisone, desonide). Severe, treatment-resistant infantile cases should be referred to a dermatologist for further evaluation and treatment. Adult cases present with a chronic course of seasonal remissions and exacerbations with flares in the condition precipitated by stress, fatigue, and climate changes. Facial seborrheic dermatitis can be treated with topical antifungals (e.g., ketoconazole cream or foam, ciclopirox gel) or daily facial washing with antidandruff shampoo or soaps. Scalp involvement can also be treated with antidandruff shampoos with formulations including ketoconazole, ciclopirox, coal tar, salicylic acid, selenium sulfide, and zinc pyrithione. Periodic twice daily application of low-dose corticosteroids for several days can control flares. Alternative treatments include promiseb cream, tacrolimus 0.1% ointment, or pimecrolimus 1% cream. Thick, adherent scalp lesions can be treated with overnight application of 10% liquor carbonis detergens in a cream solution or fluocinolone scalp oil. Oral antifungal medications that target Malassezia (such as ketoconazole, fluconazole, or itraconazole) are indicated with severe disease. Secondary bacterial infection should be treated with antistaphylococcal antibiotics (dicloxacillin, cephalexin). Involvement at the base of the eyelashes can be treated with lid massage and frequent washing with antidandruff shampoos. Patients with neurologic disease or HIV infection are more resistant to treatment and commonly have more severe manifestations of the condition. Atopic dermatitis (A), also called eczema, is a common disorder characterized by dry skin and severe pruritus. Flexor regions are most commonly affected with skin manifestations including erythema, papulation, oozing and crusting, excoriation, and lichenification. Psoriasis (B) presents with sharply demarcated, erythematous lesions with silvery white scales. This condition most commonly affects the extensor areas but has a variety of presentations that can present elsewhere. Psoriasis also presents with characteristic nail changes, arthritis, and a positive family history. Tinea capitis (D) typically presents with annular plaques with surrounding scale and alopecia and most commonly affects children. Black dots within regions of alopecia represent broken hairs. The picture above demonstrates a yellow, greasy, scaled lesion that is most indicative of infantile seborrheic dermatitis.
Which of the following is the greatest risk factor for the development of actinic keratosis? AAge over 40 years BFemale sex CHispanic race DResidence in northern climates
Correct Answer ( A ) Explanation: Actinic keratosis results from atypical epidermal proliferation of keratinocytes secondary to prolonged ultraviolet exposure. Risk factors for the development of actinic keratosis include prolonged sun exposure, fair skin type, and age over 40 years. Men are more commonly affected than women, and bald men are especially at risk for the development of lesions on the sun-exposed scalp. Actinic keratosis is more common in communities where fair-skinned individuals live nearer to the equator and in sunny climates, such as Australia. Actinic keratoses can present as erythematous macules, papules, or plaques and have scaly surfaces with irregular borders. Lesions may be of any size or shape and single or multiple. The differential diagnosis of actinic keratosis includes squamous cell carcinoma, psoriasis, solar lentigo, basal cell carcinoma, or inflamed seborrheic keratosis. Actinic keratoses are considered premalignant lesions as they have the potential to develop into cutaneous squamous cell carcinoma. Diagnosis of actinic keratosis is clinical, and treatment involves application of liquid nitrogen, topical dermabrasive agents, photodynamic therapy, or surgical excision. Punch or shave biopsy can provide a definitive diagnosis if needed. Prevention of actinic keratosis involves avoiding excess ultraviolet light exposure and applying sunscreen. Female sex (B) is not an independent risk factor for the development of actinic keratosis. Hispanic race (C) is not considered an independent risk factor for actinic keratosis, although fair-skinned Hispanic patients are at risk for developing ultraviolet damage and actinic keratosis. Residence in northern climates (D) is not a risk factor for actinic keratosis. Actinic keratosis is more common in geographic areas with more sun exposure. Patients in all regions should be encouraged to use sunscreen when outside and avoid indoor tanning booths.
Which of the following is the most common type of skin cancer in the United States? ABasal cell carcinoma BKaposi sarcoma CMalignant melanoma DSquamous cell carcinoma
Correct Answer ( A ) Explanation: Basal cell carcinoma accounts for the majority of cases of skin cancer in the United States. Risk factors for the development of basal cell carcinoma include advanced age, male sex, fair skin type, exposure to ultraviolet radiation, and immunosuppression. Of these, ultraviolet radiation exposure is the single most important modifiable risk factor. Basal cell carcinoma is a slow-growing tumor of epithelial origin that has a low rate of metastasis. However, it is considered malignant due to the potential for local invasion and destruction of surrounding structures. The classic appearance of basal cell carcinoma is a pearly, tan, or pink nodule with telangiectasias and central ulceration. These lesions are described as nodular basal cell carcinomas. Other types include superficial and morpheaform. Superficial basal cell carcinomas are erythematous macules with raised borders, and morpheaform basal cell carcinomas appear as atrophic, indurated papules or plaques. Definitive diagnosis of basal cell carcinoma is with biopsy, and treatment is surgical excision with clear margins greater than 2 mm. Topical cytotoxic creams are sometimes employed on small lesions in noncritical areas. Radiation therapy can be used on patients who are poor surgical candidates. Kaposi sarcoma (B) is an uncommon skin cancer that is more prevalent in immunocompromised individuals. The appearance of Kaposi sarcoma is a violaceous, indurated macule, papule, or plaque. Lesions may be singular or multiple and can extend to local lymph nodes. Those with cutaneous Kaposi sarcoma should be evaluated for visceral involvement, as lesions can grow in the gastrointestinal and pulmonary tract. Treatment is with radiation, diminishing doses of immunosuppressant therapy, or optimizing highly-active antiretroviral therapy. Malignant melanoma (C) is not the most common skin cancer in the United States, although it is the most common cause of skin cancer deaths. Malignant melanoma appears as a hyperpigmented macule or nodule with irregular borders. This melanoma may arise de novo or may grow at the site of a longstanding nevus. Malignant melanoma metastasizes quickly, so early intervention with crucial. Survival is highly dependent on tumor stage at diagnosis. Those with superficial spreading melanoma staged as IA have an excellent prognosis. Treatment of early melanoma involves excision with Mohs surgery to ensure wide, clear margins. Treatment of advanced melanoma includes surgery and adjuvant chemotherapy. Squamous cell carcinoma (D) is the second most common skin cancer in the United States. A typical squamous cell carcinoma is an erythematous, scaly papule or plaque on a sun-exposed surface. Diagnosis is with biopsy, and treatment involves excision with wide borders, as these lesions can metastasize.
What skin condition known as "the itch that rashes" involves lichenified plaques in a flexural distribution, especially of the antecubital and popliteal fossae, volar aspect of the wrists, ankles, and neck? AAtopic dermatitis BPsoriasis vulgaris CScabies DSeborrheic dermatitis
Correct Answer ( A ) Explanation: Commonly known as "the itch that rashes," atopic dermatitis is a chronic, pruritic, inflammatory skin disease that occurs most frequently in children but may also affect adults. It is a common skin condition, affecting approximately 5-20% of people. Many different factors are involved in the pathogenesis of atopic dermatitis including a weakened skin barrier, genetics, and immune dysregulation. Atopic dermatitis is often associated with an elevated serum level of immunoglobulin E and a personal or family history of atopy, a group of disorders that includes eczema, asthma, and allergic rhinitis. There is no cure for atopic dermatitis, but most patients will see natural improvement in their symptoms in their 30s. The clinical presentation of atopic dermatitis is highly variable. Most cases involve cutaneous xerosis and pruritus. Typically, there are lichenified plaques in a flexural distribution, especially of the antecubital and popliteal fossae, volar aspect of the wrists, ankles, and neck. Acute eczema is often characterized by intensely pruritic, erythematous papules and vesicles with exudation and crusting. Alternatively, subacute or chronic lesions present as dry, scaly, or excoriated erythematous papules. Lichenification may also be present in chronic atopic dermatitis. Patients may also present with a variety of cutaneous findings called "atopic stigmata," which include centrofacial pallor, dermographism, keratosis pilaris, palmar hyperlinearity, pityriasis alba, periorbital darkening, and Dennie-Morgan infraorbital folds. The diagnosis of atopic dermatitis is primarily clinical. In order to make the diagnosis, at least three of the following criteria should be met: evidence of pruritic skin; involvement of the antecubital fossae, popliteal fossae, neck, areas around the eyes, or anterior ankles; a history of asthma or hay fever; onset before 2 years of age; or visible dermatitis involving the flexural surfaces. Skin biopsy and laboratory testing are not used routinely for diagnosing atopic dermatitis, although they may be helpful in ruling out other cutaneous disorders. Treating atopic dermatitis is a multistep process. The initial focus should be on eliminating exacerbating factors, which most commonly involve environmental allergens, food allergens, and contact allergens. Once the exacerbating factors have been removed, maintaining skin hydration should become the next focus. Topical emollients applied twice a day may help reduce the severity of atopic dermatitis and result in fewer flares. Improving bathing practices by using a warm soaking bath and mild or soap-free cleansers may improve atopic dermatitis as well. When needed, oral antihistamines may be used to help alleviate pruritus. Topical steroids should be avoided in children and should only be used if the benefits outweigh the risks. There are a variety of comorbidities that may be associated with atopic dermatitis, all of which should be reviewed with the patient. These include allergic rhinitis, asthma, food allergies, ichthyosis vulgaris, eye disease, obesity, cardiovascular disease, and anemia. Psoriasis vulgaris (B) typically presents as well-demarcated, lichenified plaques with a silvery scale. They are most common on extensor surfaces, differentiating psoriasis vulgaris from atopic dermatitis, which is commonly found in a flexural distribution. Scabies (C) is a common skin condition caused by the mite Sarcoptes scabiei. It typically presents as an intensely pruritic eruption of multiple small, erythematous, excoriated papules. The distribution is characteristic and involves the sides and webs of the fingers, wrists, axillae, areolae, and genitalia, differentiating it from atopic dermatitis. Seborrheic dermatitis (D) is most commonly found on the scalp but can also be found in the eyebrows, melolabial folds, and chest. It is rarely found on flexural surfaces, which differentiates it from atopic dermatitis.
A 64-year-old woman presents to the clinic with a well-demarcated and raised rash on her face. The rash is erythematous, tender, and warm. She states that she developed fever and chills a couple of hours before the rash. She is diagnosed with erysipelas. What is the most common etiologic pathogen? AGroup A Streptococcus BPasteurella multocida CPseudomonas aeruginosa DStaphylococcus aureus
Correct Answer ( A ) Explanation: Erysipelas and cellulitis are infections of the skin and soft tissues. Erysipelas involves the superficial lymphatics and upper dermis, and cellulitis involves the deeper dermis and subcutaneous fat. Erysipelas develops because of bacterial entry due to breaches in the skin barrier. Erysipelas occurs most commonly in young children and older adults. Most cases of erysipelas are caused by beta-hemolytic streptococci (groups A, B, C, G, and F). The most common cause is group A Streptococcus (Streptococcus pyogenes). Staphylococcus aureus is a less common but notable cause. Erysipelas presents as areas of skin erythema, edema, and warmth. There is typically a clear demarcation between involved and uninvolved skin, and the involved skin is often raised. Erysipelas is almost always unilateral, and the lower extremities are the most common site of involvement. Bilateral involvement in cellulitis or erysipelas is rare and should prompt consideration of other causes. Lymphangitis and involvement of regional lymph nodes are common findings in erysipelas. Erysipelas is always nonpurulent, whereas cellulitis may present with or without purulence. Erysipelas generally begins with a sudden onset, which contrasts the typical indolent onset of cellulitis over a couple of days. Systemic manifestations, including fever, chills, severe malaise, and headache, are common in erysipelas and may precede the onset of cutaneous manifestations by minutes to hours. Involvement of the ear (Milian ear sign) can also help distinguish between cellulitis and erysipelas because the ear does not contain deeper dermis tissue. Laboratory manifestations are nonspecific and may include leukocytosis, elevated C-reactive protein, and elevated erythrocyte sedimentation rate. However, laboratory testing is not required for patients with uncomplicated infection in the absence of comorbidities or complications. The diagnosis of erysipelas is usually made based on clinical manifestations. Empiric antibiotic treatment directed at beta-hemolytic streptococci is the recommended treatment for erysipelas. Patients with systemic manifestations, such as fever and chills, should be treated with parenteral antibiotics. Appropriate choices of antibiotics include cefazolin, ceftriaxone, or flucloxacillin. Cefazolin has coverage against streptococci and methicillin-sensitive Staphylococcus aureus, which is beneficial in cases where erysipelas and cellulitis are difficult to distinguish. Ceftriaxone is convenient because of the once-daily dosing. Patients with mild infection or patients who improve following initial treatment with parenteral antibiotic therapy may be treated with oral penicillin or amoxicillin. Patients with beta-lactam allergies can be treated with cephalexin (if the patient can tolerate cephalosporins), clindamycin, or trimethoprim-sulfamethoxazole. The duration of therapy varies between five and 14 days based on clinical response. Elevation of the affected area is also recommended. Pasteurella multocida (B) is a common etiologic agent of soft tissue infections due to dog or cat bites. Pasteurella species can be isolated from the wound of 50% of dog bites and 75% of cat bites. However, the patient described in this vignette does not have a history of a cat or dog bite. Pseudomonas aeruginosa (C) occurs more commonly in skin and soft tissue infections of immunocompromised patients. Other conditions commonly associated with Pseudomonas aeruginosa include skin and soft tissue infections of individuals with hot tub exposure, ecthyma gangrenosum, burn wound infections, infection following nail puncture, and green nail syndrome. Antibiotics options for skin and soft tissue infections due to Pseudomonas aeruginosa include antipseudomonal beta-lactams (cefepime or ceftazidime), carbapenems, and fluoroquinolones. Staphylococcus aureus (D) is a notable etiologic pathogen of erysipelas. However, beta-hemolytic streptococci is the most common cause.
An 80-year-old man presents to the clinic with a well-demarcated and nonpurulent rash that is raised from the surrounding skin and located in a butterfly distribution on his bilateral cheeks. The rash is erythematous, edematous, and warm to touch. He is febrile with a temperature of 101.6°F. He is diagnosed with erysipelas. Which of the following findings suggests erysipelas rather than cellulitis? AClear demarcation between the involved and uninvolved skin BLymphadenopathy CThe presence of purulent drainage DWarmth of the skin
Correct Answer ( A ) Explanation: Erysipelas is a type of bacterial infection of the skin and soft tissue. It involves the superficial lymphatics and upper dermis, whereas cellulitis involves the deeper dermis and subcutaneous fat. Skin infections often develop because of bacterial entry due to breaches in the skin barrier. Erysipelas occurs most commonly in young children and older adults. Most cases of erysipelas are caused by beta-hemolytic streptococci (groups A, B, C, G, and F). The most common is group A Streptococcus (Streptococcus pyogenes), and Staphylococcus aureus is a less common but notable cause. Erysipelas presents as areas of skin erythema, edema, and warmth. There is typically clear demarcation between involved and uninvolved skin, and the involved skin is often raised. The presentation is often abrupt and frequently involves systemic symptoms, such as fever, chills, malaise, and headache. Bilateral involvement in cellulitis or erysipelas is rare and should prompt consideration of other causes. However, one exception to this presentation is involvement of the bilateral cheeks ("butterfly" pattern) in erysipelas. The lower extremity is the most common site of involvement. Lymphangitis and involvement of regional lymph nodes are common in erysipelas. Erysipelas is always nonpurulent, whereas cellulitis may present with or without purulence. Manifestations of erysipelas are nonspecific and may include leukocytosis, elevated C-reactive protein, and elevated erythrocyte sedimentation rate. However, laboratory testing is not required for patients with uncomplicated infection in the absence of comorbidities or complications. The diagnosis of erysipelas is usually made based on clinical manifestations. Empiric antibiotic treatment directed at beta-hemolytic streptococci is the recommended treatment for erysipelas. Patients with systemic manifestations, such as fever and chills, should be treated with parenteral antibiotics. Appropriate choices of parenteral antibiotics include cefazolin, ceftriaxone, or flucloxacillin. Cefazolin has coverage against streptococci and methicillin-sensitive Staphylococcus aureus, which is beneficial in cases where erysipelas and cellulitis are difficult to distinguish. Ceftriaxone is convenient because of the once-daily dosing, and it provides coverage against group A Streptococcus and some cases of methicillin-sensitive Staphylococcus aureus. Patients with mild infection or patients who show improvement following initial treatment with parenteral antibiotic therapy may be treated with oral penicillin or amoxicillin. Patients with beta-lactam allergies can be treated with cephalexin (if the patient can tolerate cephalosporins), clindamycin, or trimethoprim-sulfamethoxazole. The duration of therapy varies between five and 14 days based on clinical response. Elevation of the affected area is also recommended in the treatment of erysipelas and nonpurulent cellulitis. Lymphadenopathy (B) may be present in either cellulitis or erysipelas and cannot be used to distinguish these two conditions. The presence of purulent drainage (C) suggests cellulitis rather than erysipelas, which is always nonpurulent. Warmth of the skin (D) occurs with cellulitis or erysipelas and cannot be used to distinguish the two conditions.
Which of the following is a risk factor for developing lichen simplex chronicus? ADepression BFemale sex CGenetic predisposition DHepatitis C infection
Correct Answer ( A ) Explanation: Lichen simplex chronicus is a diagnosis consisting of chronically pruritic skin. The pathophysiology is not well understood, however, it is related to consistent scratching or rubbing of the skin in an area that leads to fibrosis of the skin. These areas become excoriated and lichenified. There is an "itch-scratch" cycle created where itching leads to more scratching and more scratching causes cutaneous trauma that leads to more itching. Risk factors for developing the condition include depression and anxiety, liver disease, or atopic dermatitis. Patients often present with thickened, lichenified skin in areas that are easily reached such as the scrotum or vulva, lateral or posterior neck, or dorsal foot. Skin biopsies can aid in ruling out other causes of pruritus but will not help with diagnosing lichen simplex chronicus. Management includes identifying and curbing the compulsion to scratch. Treatment can include topical corticosteroids, antihistamines, or antidepressants. Female sex (B) is not associated with a greater risk of lichen simplex chronicus but has been associated with a greater risk of lichen planopilaris. Genetic predisposition (C) puts patients at risk for a variety of skin disorders including lichen planus, but is not associated with lichen simplex chronicus. Hepatitis C infection (D) is thought to have an association with lichen planus but this is uncertain.
Which of the following statements is true regarding a measles exanthem? AIt appears approximately two to four days after the onset of fever BIt begins on the wrists and ankles CIt consists of blanching lesions in later stages of the exanthem DIt consists of widespread Koplik spots
Correct Answer ( A ) Explanation: Measles is a viral illness associated with a prodrome of fever, cough, and conjunctivitis that is followed by a characteristic exanthem. The virus is highly contagious and can be spread through fomites or person-to-person contact. Patients can be contagious for up to five days before symptoms even begin. The incubation period from exposure to symptom development is approximately 6 to 21 days. Patients then begin to experience fever, malaise, cough, coryza, and conjunctivitis. This usually lasts two to four days before the onset of the rash. Koplik spots, the enanthem associated with measles, are found just before the rash develops on the buccal mucosa. These are described as whitish or bluish elevated lesions that are 1-3 mm in diameter. These begin to slough off when the exanthem appears. The exanthem associated with measles typically begins on the face or behind the ears and spread in a cephalocaudal pattern. It blanches early on but in later stages will not blanch secondary to petechiae. The rash usually lasts five to seven days and will fade and desquamate as it clears. Clinical improvement of the virus begins a few days after the rash appears. Diagnosis is clinical, however, it can be confirmed with a positive serum anti-measles immunoglobulin M (IgM) approximately three days after the exanthem appears. Complications from measles can include gastrointestinal manifestations, secondary infections, pneumonia, and encephalitis. Treatment is supportive. There is an available vaccination for measles that is typically given to children between the ages of 12 and 15 months. The exanthem associated with Rocky Mountain Spotted Fever begins on the wrists and ankles (B), however, the rash associated with measles typically begins on the face. Blanching lesions (C) are present early on in the course of the exanthem. In later stages of the exanthem, the lesions will not blanch. Koplik spots (D) are an enanthem, not an exanthem. These are found localized to the buccal mucosa and occasionally other areas of the oropharynx.
A 20-year-old man presents to the emergency department with sudden onset of itchy erythematous papules and plaques one hour after eating crab. He is diagnosed with urticaria. Which of the following is a classic finding of urticaria? ACentral pallor BGolden-colored crusting CLesions in a Christmas tree distribution DTarget-shaped lesions
Correct Answer ( A ) Explanation: Urticaria (hives) is a common disorder with a lifetime prevalence of approximately 20% in the general population. It may be accompanied by angioedema, which is swelling of the dermis and subcutaneous tissues. The pathogenesis of urticaria involves the activation of mast cells in the upper dermis leading to the release of histamine and other vasoactive mediators, which cause itching and swelling. Common causes include allergic reactions (immunoglobulin E mediated hypersensitivity reactions) to medications (penicillins, cephalosporins, morphine, codeine, dextromethorphan, sulfa drugs, diuretics, aspirin, and nonsteroidal anti-inflammatory drugs), foods (milk, eggs, peanuts, soy, wheat, fish, shellfish, and tomatoes), insect stings and bites, reactions to medications that cause nonallergic mast cell activations (e.g., narcotics), infections (viral, bacterial, or parasitic), and ingestion of nonsteroidal anti-inflammatory drugs. In addition, environmental causes include cold, heat, and exercise-induced urticaria. In rare cases, urticaria may be due to an underlying malignancy. Urticaria caused by an allergic reaction typically occurs within minutes to two hours of exposure to the allergen, which is commonly a food, medication, insect bite or sting, or latex exposure. Allergic reactions may be limited to the skin or part of a systemic reaction with possible anaphylaxis. Urticaria is often classified according to its chronicity. Acute urticaria is defined by the presence of urticaria for less than six weeks. Urticaria is considered chronic when symptoms are present on most days of the week for six weeks or longer. Acute and chronic urticaria present with the same lesions and therefore are indistinguishable at initial presentation. Individuals with new-onset urticaria may have an identifiable trigger, such as medications, food ingestions, insect bites, or infections. However, in many cases, there is no identifiable trigger. Urticaria presents as circumscribed and erythematous raised lesions that often have central pallor. The shape of urticaria may be round, oval, or serpiginous, and the size varies from less than 1 centimeter to several centimeters. Urticaria is associated with intense pruritus that often seems worse at night. Individual lesions are often transient, appearing and enlarging over the course of minutes to hours and then disappearing within 24 hours. Lesions often coalesce as they enlarge. Urticaria can affect any location on the body but areas compressed by clothing, such as waistbands, and areas where skin rubs together may have more severe urticaria. Urticaria is diagnosed clinically by taking a detailed history and by the presence of characteristic skin lesions. Laboratory studies are typically normal in patients without any history or exam finding to suggest an underlying disease process. Serum tests for allergen-specific immunoglobulin E antibodies can be performed to confirm an allergy in patients with a suspected allergen trigger. The treatment of new-onset urticaria is focused on the short-term relief of pruritus. Two-thirds of cases of new-onset urticaria will be self-limited and resolve spontaneously. However, pharmacologic therapy can help hasten the resolution of symptoms. Second-generation H1 antihistamines, such as cetirizine, are typically used to treat urticaria. There is no data suggesting that one second-generation antihistamine is more effective than another. First-generation H1 antihistamines, such as diphenhydramine or hydroxyzine, are often helpful at bedtime because of the sedating effect. H2 antihistamines are also often used in the treatment of acute urticaria. There is one study that supports H2 antihistamines, such as ranitidine or famotidine, improving urticaria resolution in patients presenting to the emergency department with acute urticaria. Glucocorticoids are typically not indicated in cases with acute urticaria without angioedema. However, in cases where urticaria persists for a couple of days, a brief course (less than one week) of systemic glucocorticoids, such as prednisone, can be used. Antihistamines should be continued until several days after the resolution of the urticaria. Golden-colored crusting (B) is associated with impetigo, which is a skin infection seen most often in children 2-5 years old. Impetigo is usually caused by Staphylococcus aureus. It is treated with topical (mupirocin) or systemic antibiotics (cephalexin or dicloxacillin) depending on the extent of skin involved. Lesions in a Christmas tree distribution (C) is associated with pityriasis rosea. This condition is self-limited and can last up to eight to 10 weeks. The treatment is aimed at controlling pruritus with topical corticosteroids and antihistamines. Target-shaped lesions (D) is characteristic of erythema multiforme, which is usually caused by an infection (most commonly herpes simplex virus). The erythema migrans lesion of Lyme disease may also be characterized as target-shaped.
A 65-year-old woman with a history of stasis dermatitis presents to the emergency department with a progressive rash on her left lower leg. She also complains of subjective fever and chills. The rash is red, swollen, warm to touch, and has some purulent exudate. Vital signs are T 100.6°F, HR 70, BP 116/76 mm Hg, RR 16, and oxygen saturation is 99% on room air. Which of the following antibiotics is the most appropriate choice for this patient? AIntravenous cefazolin BIntravenous vancomycin COral cephalexin DOral trimethoprim-sulfamethoxazole
Correct Answer ( B ) Explanation: Cellulitis is a skin and soft tissue infection that manifests as an area of skin erythema, edema, and warmth. It develops due to bacterial entry through breaches in the skin and involves the deep dermis and subcutaneous fat. Cellulitis is most common in middle-aged and older adults. Skin barrier disruption often predisposes to cellulitis. It may be due to trauma (abrasion, penetrating wound, pressure ulcer, venous leg ulcer, insect bite, or injection drug use), skin inflammation (eczema or radiation therapy), breaks in the skin between the toes, or pre-existing skin infection (tinea pedis, impetigo, and varicella). Additional associated conditions include edema due to impaired lymphatic drainage, edema due to venous insufficiency, obesity, and immunosuppression. The most common microbial cause of cellulitis is group A Streptococcus (also called Streptococcus pyogenes). Staphylococcus aureus is a notable but less common cause. The clinical manifestations of cellulitis consist of areas of skin erythema, edema, and warmth. Systemic manifestations, such as fever and chills, may also occur. The lower extremities are the most common site of involvement, and cellulitis is nearly always unilateral. Cellulitis may occur with or without associated purulence or abscess. The course of cellulitis is often gradual with symptoms occurring over a few days. Cellulitis may also cause lymphangitis and enlargement of regional lymph nodes. Vesicles, bullae, ecchymoses, and petechiae may also be observed. Laboratory findings of cellulitis are nonspecific and may include leukocytosis and elevated erythrocyte sedimentation rate or C-reactive protein. Complications of cellulitis include bacteremia, osteomyelitis, endocarditis, metastatic infection, and sepsis. The diagnosis of cellulitis is based on clinical manifestations. Cultures of debrided material and blood cultures are warranted in cases with severe local infection, systemic signs of infection, failure of initial antibiotic therapy, extremes of age, and in patients with special exposures, such as water-associated injury and bites. Cellulitis is treated with antibiotics. Patients with mild infection can be treated with an oral antibiotic regimen. Parenteral antibiotics are indicated in some cases. Patients with systemic manifestations (fever, chills, etc.), rapid progression of symptoms, inability to tolerate oral medications, or failure of 48 hours of oral antibiotics should be treated with parenteral antibiotics. The antibiotic regimen should cover group A Streptococcus and methicillin-susceptible Staphylococcus aureus. Methicillin-resistant Staphylococcus aureus coverage should also be included in patients with a purulent infection, prior episode of methicillin-susceptible Staphylococcus aureus, recent hospitalization, residence in a long-term care facility, hemodialysis, or human immunodeficiency virus infection. Cephalexin is a common oral antibiotic regimen for patients with nonpurulent cellulitis, and cefazolin is a common parenteral agent used for patients with nonpurulent cellulitis. Incision and drainage should be performed in patients with a drainable abscess. Patients with purulent infection who can be treated orally are typically treated with clindamycin, trimethoprim-sulfamethoxazole, or doxycycline. These agents each cover methicillin-susceptible Staphylococcus aureus. However, amoxicillin or cephalexin must be added if doxycycline is used because doxycycline does not cover group A Streptococcus. If parenteral therapy is indicated for a purulent infection, then vancomycin is typically used. Patients initially treated with parenteral therapy can transition to oral therapy once systemic symptoms are absent and there are signs of clinical improvement. Intravenous cefazolin (A) is used when parenteral therapy is indicated but the cellulitis is nonpurulent. Intravenous cefazolin provides coverage against group A Streptococcus but does not cover methicillin-resistant Staphylococcus aureus. Since this patient had systemic symptoms and purulent cellulitis, both intravenous therapy and methicillin-resistant Staphylococcus aureus coverage are indicated. Oral cephalexin (C) is a first-generation cephalosporin that provides coverage against group A Streptococcus but not methicillin-resistant Staphylococcus aureus. Oral trimethoprim-sulfamethoxazole (D) is an oral antibiotic that provides coverage against methicillin-resistant Staphylococcus aureus. However, this patient requires parenteral therapy.
A 55-year-old man presents to the office with complaints of right lower leg pain and erythema for the past three days. Physical exam reveals erythema without well-defined borders, as well as edema, warmth, and tenderness of the right lower leg. When questioned, the patient states he believes he had a cut on his leg a few days before this rash began. Vitals are within normal limits, and the patient is otherwise healthy. What would be the best next step for management of this patient? AHospital admission with initiation of piperacillin-tazobactam BOral cephalexin COral trimethoprim-sulfamethoxazole DTopical trimethoprim-sulfamethoxazole
Correct Answer ( B ) Explanation: Cellulitis is defined as an area of skin that is erythematous, edematous, and warm, resulting from bacterial infiltration into the dermal and subcutaneous tissues. Cellulitis most often occurs in adults of middle to older age. Cellulitis is most commonly caused by beta-hemolytic streptococci, particularly group A Streptococcus or Streptococcus pyogenes. Staphylococcus aureus is a less common cause. Gram-negative aerobic bacilli infection is possible but rare. A skin abscess may also present with cellulitis. There are a number of potentially predisposing factors, which include penetration or disruption of the skin, inflammatory skin conditions such as eczema, lymphedema, venous insufficiency, obesity, immunosuppression, and preexisting skin infections, including fungal. The diagnosis is clinical, but particular attention should be paid to the presence of systemic complications. Risk factors for methicillin resistance include recent hospitalization, prior methicillin-resistant S. aureus (MRSA) infection or known colonization, residence at long-term care facility, recent surgery, hemodialysis, HIV infection, and proximity to an indwelling medical device. Laboratory testing is not required if uncomplicated and without comorbidities such as diabetes, immunosuppression, etc. Physical exam will show a spreading erythematous area without well-defined borders. It may also be edematous, tender, and warm to the touch. A full history should be obtained to determine if patient is at higher risk for complications. Systemic signs and symptoms on history and physical exam include sustained tachycardia, tachypnea, pyrexia, hypotension, nausea, and vomiting. Treatment for uncomplicated cellulitis in a healthy individual consists of oral cephalexin, but intravenous cefazolin is warranted if comorbidities or systemic complications are present. If methicillin resistance is suspected, either oral trimethoprim-sulfamethoxazole or amoxicillin plus doxycycline should be initiated. Intravenous therapy in the presence of suspected methicillin resistance necessitates vancomycin or daptomycin. Clinical improvement in uncomplicated cellulitis in a healthy individual is usually seen within 24 to 48 hours after initiation of an appropriate oral agent. Patients should be instructed to return to clinic if no improvement is seen within five days. Hospital admission with initiation of piperacillin-tazobactam (A) is not indicated at this time since he does not appear to have systemic complications and this patient is otherwise healthy. Oral trimethoprim-sulfamethoxazole (C) is not a first-line agent in the presence of methicillin susceptibility. It is indicated as a first-line choice along with either doxycycline or clindamycin in cases where patients have a history or risk factors of MRSA infections and in cases where patients fail antibiotics that do not cover MRSA. Topical trimethoprim-sulfamethoxazole (D) is not indicated for cellulitis.
A 23-year-old woman presents with a new pruritic genital lesion for three days. She reports a painful mass in her right groin as well. Physical exam reveals a small cluster of 2-3 mm flesh-colored, cauliflower-like lesions on the right labia majora and right inguinal lymphadenopathy. Which aspect of the patient's presentation indicates that this lesion is likely caused by human papillomavirus as opposed to herpes simplex virus? AA new genital lesion BCauliflower-like projections CInguinal lymphadenopathy DPruritus
Correct Answer ( B ) Explanation: Human papillomavirus is a virus that affects the basal keratinocytes of the epidermis, producing epithelial tumors of the skin and mucous membranes. It is transmitted through contact with infected skin or mucosa. Human papillomavirus infections are divided into three categories: anogenital or mucosal, nongenital cutaneous, and epidermodysplasia verruciformis. There are over 40 mucosal genotypes that can infect the genital tract. Human papillomavirus types 6 and 11 cause the majority of genital warts, and types 16 and 18 cause the majority of related neoplasms. Human papillomavirus is responsible for a majority of anogenital and oropharyngeal carcinomas and is the leading cause of cervical cancer. The risk of developing human papillomavirus infection is directly related to number of sexual partners. Presently, there is no cure for human papillomavirus infections. Human papillomavirus may present clinically in various ways. Infection with types 16 and 18 typically presents with no clinical signs or symptoms and is often only identified by Pap smear. Types 6 and 11, which cause the majority of genital warts, often present with a new genital lesion and are often pruritic and enlarging. They may be found in groups or as single lesions and often have cauliflower-like projections. Inguinal lymphadenopathy may also be present. Contrary to genital warts caused by human papillomavirus, lesions associated with herpes simplex virus do not have cauliflower-like projections. They typically present as new painful or pruritic erythematous vesicles or ulcers on the genital area. They may present in groups or as single lesions. Lymphadenopathy may also be present. Human papillomavirus is typically diagnosed by Pap smear or shave biopsy of a suspicious lesion. The evaluation of patients with human papillomavirus should include testing for other sexually transmitted infections and concomitant internal involvement, including screening for HIV, hepatitis B and C, gonorrhea, chlamydia, trichomonas, and syphilis. Although there is no cure for human papillomavirus, in patients with a healthy immune system, the viral load may decrease to undetectable amounts. Prevention of human papillomavirus is best achieved with sexual abstinence. Safe sexual practices should be implemented, including limiting the number of sexual partners and using proper protective barrier methods. Vaccinations against human papillomavirus are available for both boys and girls. The quadrivalent or 9-valent vaccine available for boys and girls protects against types 6, 11, 16, and 18. Routine vaccination is recommended at 11 years of age but can be administered as early as 9 years of age. For adolescents and adults who have not been previously vaccinated or who have not completed the vaccine series, catch-up vaccination is recommended. A new genital lesion (A) can be suspicious for a number of benign and malignant gynecologic disorders including human papillomavirus and herpes simplex virus. For example, acquired angiokeratomas of the vulva are common, benign, often "new genital lesions" that appear as asymptomatic 1-2 mm violaceous macules or papules. Contrarily, vulvar squamous cell carcinoma may also present as a "new genital lesion" but typically involves vulvar pruritus, pain, or bleeding and is a malignant process that requires intervention. Inguinal lymphadenopathy (C) is a physical exam finding that is not specific to either human papillomavirus or herpes simplex virus, although it
A 65-year-old woman is hospitalized and develops a pressure-induced skin and soft tissue injury. Physical examination reveals exposed dermis and a red and moist wound bed. There is no visible adipose tissue. What is the appropriate staging of this pressure injury? AStage 1 BStage 2C CStage 3 DStage 4
Correct Answer ( B ) Explanation: Pressure-induced skin and soft tissue injuries are localized areas of damage to the skin over a bony prominence because of pressure. Patients commonly develop these injuries during extended hospital stays or in other situations where the patient is immobile. The superficial skin is less susceptible to pressure-induced damage than deeper tissues, and therefore, the external appearance may underestimate the amount of damage. The National Pressure Ulcer Advisory Panel uses a staging system to describe the severity of ulcers. Stage 1 is characterized by intact skin with a localized area of nonblanchable erythema that does not dissipate once pressure is removed. Stage 2 is characterized by partial-thickness loss of skin with exposed dermis. The wound bed is viable, pink or red, moist, and may present as intact or with a ruptured fluid-filled blister. Adipose fat and deeper tissues are not visible in stage 2 pressure ulcers. Stage 3 is characterized by full-thickness loss of skin with visible adipose tissue. In addition, granulation tissue and rolled wound edges (epibole) are often present. Fascia, muscle, tendon, ligament, cartilage, and bones are not exposed. Stage 4 is characterized by full-thickness skin and tissue loss with exposed fascia, muscle, tendon, ligament, cartilage, or bone in the ulcer. The depth varies by anatomical location. An unstageable pressure injury occurs when the extent of full-thickness skin and tissue loss cannot be confirmed because it is obscured by slough or eschar. In these cases, if slough or eschar is removed, then a stage 3 or stage 4 pressure ulcer will be revealed. A suspected deep tissue pressure injury refers to lesions with intact overlying skin but suspected deep tissue injury. Deep tissue injury may be suspected due to a localized area of skin with persistent nonblanchable deep red, maroon, or purple discoloration or epidermal separation revealing a blood-filled blister. Pain and temperature change often precede skin color changes. It is not recommended to change the stage as healing occurs. The depth of stage 3 and stage 4 pressure ulcers varies by anatomic location. For example, the bridge of the nose has minimal subcutaneous tissue, therefore, an individual could have a shallow stage 3 ulcer on the bridge of the nose, whereas the gluteal region is more likely to have deep stage 3 pressure ulcers. The differential diagnosis of pressure ulcers includes ulcers due to diabetes, arterial insufficiency, and venous insufficiency. Ulcers due to these conditions are not considered to be pressure ulcers. However, these conditions can predispose to pressure ulcers, and ulcers are often multifactorial. The diagnosis of pressure ulcers is made clinically. The management of pressure-induced skin and soft tissue injury includes reducing or eliminating contributing factors by providing pressure redistribution and support surfaces. Patients should be positioned and repositioned at least every two hours to relieve tissue pressure. Local wound care may include debridement for patients with necrotic tissue (usually stage 3 and stage 4 ulcers). Pain control is also important. Ischemic, infection, and break down of local tissue can contribute to pain. Acetaminophen or nonsteroidal anti-inflammatory drugs (NSAIDs), such as ibuprofen, may be used for mild pain. Opioid analgesics may be used for moderate or severe pain. Topical local anesthetics, such as lidocaine, have shown some benefit in small trials but should only be used as adjunct treatment for pain control. All open ulcers are colonized with bacteria. However, only ulcers with symptoms or signs of infection should be treated with antibiotics and wound culture. Patients with deep wounds should be evaluated for the presence of osteomyelitis. Optimizing nutritious intake is also important for wound healing of stage 3 and 4 pressure ulcers. Adjunctive therapies with possible benefit include negative pressure wound therapy, hyperbaric oxygen therapy, and electrical stimulation. The best environment for pressure skin and soft tissue injury is a moist (not wet or dry) environment with dry surrounding intact skin. Stage 1 (A) is incorrect because the skin is not intact. Stage 1 is characterized by intact skin with a nonblanchable area of erythema. Stage 3 (C) is incorrect because there is no visible subcutaneous tissue. Stage 4 (D) is incorrect because there is no visible subcutaneous tissue, fascia, muscle, bone, tendon, ligament, or cartilage.
A 60-year-old man presents with two asymptomatic, well-demarcated, hyperpigmented, and round lesions on his back. The lesions have a verrucous and stuck-on appearance. The lesions have been there for several years and are not changing. He states that the lesions are not bothersome, but he came in because he wants to make sure they are not cancer. What is the recommended management for this condition? ACryotherapy BNo treatment is necessary CShave excision DTriamcinolone acetonide
Correct Answer ( B ) Explanation: Seborrheic keratoses are common benign epidermal tumors that consist of a proliferation of immature keratinocytes. They occur most often after the age of 50 but can occur in young adulthood. There is a genetic component to the etiology. The pathogenesis is not completely understood. It is unknown if human papillomavirus or ultraviolet radiation exposure plays a role. Seborrheic keratoses present as well-demarcated, round or oval lesions with a verrucous surface. They are often described as having a stuck-on or warty appearance. Seborrheic keratoses are most often asymptomatic but chronic irritation (due to friction) may occasionally cause pain, pruritus, or bleeding. The number of lesions may vary from isolated to hundreds, and the lesions are most often located on the trunk, face, and upper extremities. Seborrheic keratosis lesions are typically brown or black. The diagnosis is usually made clinically. Biopsy is necessary if the diagnosis is uncertain and there is concern for malignancy. Seborrheic keratoses typically show a well-demarcated proliferation of keratinocytes with small keratin-filled cysts on histology. Treatment is generally not required for seborrheic keratosis because they are benign and slow-growing lesions. However, lesions that are symptomatic or cause cosmetic concern can be removed. Common treatments include cryotherapy (most common), shave excision, and electrodesiccation. Postinflammatory hyperpigmentation following cryotherapy can occur and is especially common in African American patients. Cryotherapy (A) and shave excision (C) are treatment options for seborrheic keratoses in patients who have symptomatic or bothersome lesions or lesions with concern for malignancy. The patient in this vignette has classic seborrheic keratosis and no findings concerning for malignancy. Therefore, no treatment is the best management. Triamcinolone acetonide (D) is a topical glucocorticoid. Topical glucocorticoids are often used to treat dermatologic conditions related to hypersensitivity reactions or autoimmune causes. They have no role in the treatment of seborrheic keratoses.
Which of the following is true regarding the treatment of superficial partial-thickness burns? AIce should be used initially to cool the wound BRuptured blisters should be debrided CTopical antibiotics are indicated DTopical steroids are indicated
Correct Answer ( B ) Explanation: The proper classification of a burn is crucial to ensuring the best treatment. Burns are classified according to their depth and size. Burn size can be estimated using the rule of nines. It is important to distinguish superficial burns from partial-thickness burns and to accurately assess the burn size. Superficial burns involve the epidermis layer only and present as painful, dry, red burns that are blanchable with pressure. The healing time for superficial burns is three to six days. Superficial partial-thickness burns (epidermis plus superficial dermis) present as painful to temperature and air and cause blisters. These burns are moist, red, weeping, and blanchable with pressure. The healing time for superficial partial-thickness burns is seven to 21 days. Deep partial-thickness burns (epidermis to deep dermis) present with blisters but do not blanch with pressure. These burns may appear wet or waxy dry, have variable color (cheesy white to red), and are not painful and patients can only sense pressure. Deep partial-thickness burns require more than 21 days to heal and often require surgical intervention. Full-thickness burns vary in color from waxy white to leathery gray to charred and black. These are dry and inelastic and do not blanch with pressure. Sensation is intact to deep pressure only and these rarely heal without surgery. Deeper injury (i.e. fourth-degree) burns involve the fascia or muscle. Sensation is intact to only deep pressure and these never heal without surgery. Superficial burns were previously called first degree burns, partial thickness burns were previously called second degree burns, and full thickness burns were previously called third degree burns. Burns are also classified dichotomously as minor or major according to the American Burn Association. The classification is based on the depth and size of the burn, anatomic location of the burn, and age of the patient. The initial treatment of minor thermal injuries consists mainly of cooling (with room temperature tap water or cool, saline-soaked gauze), simple gentle cleansing with mild soap and water, and appropriate dressing. Cooling helps to provide pain relief and limit tissue injury. However, water should not be applied for more than five minutes to avoid macerating the wound, and ice should not be used to cool minor thermal injuries. Towels or wet gauze can be used as an alternative to soaking the burn in water and can be applied for up to 30 minutes. Pain management and tetanus prophylaxis are also important. Pain management of small burns can be accomplished with acetaminophen and nonsteroidal anti-inflammatory drugs, such as ibuprofen, alone or in combination with opioids, such as oxycodone. Elevation of extremity burns above the level of the heart can reduce pain and swelling. Patients with larger or recently sustained burns may experience significant pain requiring intravenous opioids, such as morphine or hydromorphone. Burn wounds should be cleaned. The recommended cleaning agents are mild soap and tap water or chlorhexidine wash. Skin disinfectants, such as povidone-iodine, should not be used because these cleaners can inhibit the healing process. Patients should wash their wounds daily during dressing changes. Sloughed or necrotic skin, including ruptured blisters, should be debrided before applying a dressing because these skin remnants may increase the risk of infection and limit the contact of topical antimicrobial agents to the burn wound. Blisters often develop with superficial or deep partial-thickness burns. Ruptured blisters should be debrided. However, needle aspiration of intact blisters is not recommended due to increased risk of infection. The management of clean and intact blisters is controversial. Infection is a potential complication of burns. Superficial burns, such as sunburns, and superficial partial-thickness burns rarely develop infections and do not require a topical antimicrobial agent. Application of nonperfumed moisturizer cream is typically all that is required for superficial burns. Topical antibiotics should only be applied to deep partial-thickness or full thickness burns. Systemic prophylactic antibiotics are not indicated to prevent infection with any acute burn. Some clinicians use aloe vera or a basic topical antibiotic, such as bacitracin, on superficial burns. These treatments are inexpensive, and aloe vera provides some antibacterial activity, but there is no clear evidence demonstrating improved outcomes. Silver sulfadiazine is commonly used but is controversial and may increase the duration of wound healing and cause increased pain. There is no role for topical steroids in the initial treatment of minor burns because they may increase the risk of infection and impair healing. Wound dressing should be applied after washing with mild soap and water for minor burns treated in the clinic or emergency department. The dressing should include a nonadherent gauze (xeroform) placed over the burn, a second layer of fluffed dry gauze, and an outer layer of an elastic gauze roll, such as kerlix. Each individual toe or finger should be wrapped separately to prevent maceration. Superficial burns do not require a dressing. Recommendations for the frequency of dressing changes ranges from twice daily to weekly. Severe burns can cause the loss of significant fluids. The Parkland formula is used to estimate the amount of fluids that should be replaced. The Parkland formula recommends 4 milliliters per kilogram per percentage of total body surface area involved given intravenously during the first 24 hours. In the Parkland formula, 50% of the fluids are given in the first eight hours and the other 50% is given over the next 16 hours. Patients with major burns defined according to the American Burn Association should be transferred to burn centers and may require surgery and skin grafts. Burns to the hands and feet are at increased risk of loss of function and should have a lower threshold for transfer. Ice should be used initially to cool the wound (A) is incorrect because direct application of ice can increase pain and burn depth. Topical antibiotics are indicated (C) is incorrect because superficial partial-thickness burns are rarely infected. These burns can be treated with nonperfumed moisturizer or aloe vera. Topical antibiotics, such as bacitracin, may be used but are not indicated. Topical steroids are indicated (D) is incorrect. Topical steroids may increase the risk of infection or impair healing.
A 35-year-old man who is positive for human immunodeficiency virus with a CD4 count of 99/mm³ presents to his primary care provider for follow-up. He has recently started antiretroviral therapy and has noticed a painless skin rash that has gotten significantly worse. On physical exam, which of the following presentations of rash would be most likely identified? ABlanching erythematous "brick-red" maculopapular rash on the head and neck BCoarse, sandpaper-like, erythematous, blanching rash that is diffuse CPurple-brown, nonpruritic, nonpainful papules on the face and lower extremities DVesicular lesions distributed along a dermatome
Correct Answer ( C ) Explanation: AIDS related Kaposi sarcoma is a vascular tumor that presents as either a cutaneous lesion or a mucous membrane lesion. Cutaneous lesions appear as red, pink, purple, or brown lesions due to their vascularity. They may also have a yellow, perilesional halo. These papular or nodular lesions can be found anywhere in the body but are most commonly seen on the face, lower extremities, oral mucosa, and genitalia. They can develop on the soles of the feet. They range in size from several millimeters to several centimeters in diameter and may grow slowly or rapidly. They are painless and are not pruritic. Nodular forms may bleed easily. Due to vascular obstruction, there may be lymphedema that is out of proportion given the size of the lesion. There are four forms of Kaposi sarcoma. All of them require previous infection from human herpesvirus 8, also known as Kaposi sarcoma-associated herpes virus. The most common form is seen in patients with acquired immune deficiency syndrome (AIDS) and it is considered an acquired immune deficiency syndrome-defining condition, as it occurs in patients with a low CD4 count (< 100/mm³). The other forms of Kaposi sarcoma are endemic or African, organ transplant-associated, and classic, which affects Mediterranean and Jewish men. Initial diagnosis can be made based on clinical presentation, but biopsy of the lesion or polymerase chain reaction of the lesion for human herpesvirus 8 is required for definitive diagnosis. The condition is staged based on its progression. Stage I, the maculonodular stage, and Stage II, the infiltrative stage, have slower rates of progression. Stage III, the florid stage, and Stage IV, the disseminated stage, are rapidly progressive and have increased incidence of gastrointestinal and visceral involvement and impairment. Kaposi sarcoma does not have a cure. Palliative treatments are useful in decreasing the severity and progression of the condition. Antiretroviral therapy is used to elevate the CD4 count. Initially, the medication can cause an increase in the severity of Kaposi sarcoma, but as the CD4 count rises, both the severity of the condition as well as its progression will decrease. In the case of smaller lesions that are slowly progressing, compression stockings and watchful waiting can be considered. In more severe cases, local therapies, such as surgery and radiation, can be useful. Chemotherapy can also be considered. Blanching erythematous "brick-red" maculopapular rash on the head and neck (A) is the rash associated with measles. It would be accompanied by fever, cough, coryza, conjunctivitis, and Koplik spots. Coarse, sandpaper-like, erythematous, blanching rash that is diffuse (B) is the description of scarlet fever. This condition would follow an infection of group A Streptococcus. Pharyngitis would be a likely precursor to the rash. Vesicular lesions distributed along a dermatome (D) is the description of herpes zoster. Incidence of this condition increases in immunocompromised patients, however this condition is painful unlike Kaposi sarcoma.
A 35-year-old, obese woman presents to the clinic with an erythematous and purulent nodule in her axilla. She states she has had four similar episodes over the last two years. She is diagnosed with hidradenitis suppurativa. Which of the following is part of the first-line treatment for mild hidradenitis suppurativa? AAdalimumab BOral doxycycline CTopical clindamycin DWide excision
Correct Answer ( C ) Explanation: Hidradenitis suppurativa refers to a painful, chronic, suppurative process that most commonly involves skin in intertriginous areas and affects the apocrine sweat glands. It affects the folliculopilosebaceous units and is marked by the development of inflammatory nodules, pustules, sinus tracts, and scars that occur primarily in intertriginous areas. Hidradenitis suppurativa is neither contagious nor due to poor hygiene. It occurs most commonly in women, and primarily occurs on intertriginous skin. The axilla is the most common site of involvement. Other common sites include the inguinal area, inner thighs, perineal and perianal areas, mammary and inframammary skin, and buttocks. The classic presentation of hidradenitis suppurativa is recurrent, painful, and inflamed nodules. The nodules may rupture and discharge purulent, malodorous material. Persistent disease leads to epithelialized sinuses and characteristic "rope-like" scarring. The symptoms and subsequent psychological manifestations often have a profound impact on quality of life. The diagnosis of hidradenitis suppurativa is based on the characteristic clinical manifestations occurring on a chronic and recurrent basis. Skin biopsy is usually not needed for the diagnosis, and bacterial cultures are typically not necessary to guide treatment. Lifestyle interventions, such as smoking cessation and weight management, can lead to improvement in symptoms of hidradenitis suppurativa. Patients with hidradenitis suppurativa should be staged using the Hurley staging system to guide management. Patients with Hurley stage I hidradenitis suppurativa present with a single or multiple nodules or abscesses without associated sinus tracts or scarring. The treatment of individuals with Hurley stage I consists of daily topical clindamycin and topical antiseptic washes (chlorhexidine) used when showering. Additional treatments for this stage may include analgesics and intralesional corticosteroids. Patients with Hurley stage II present with recurrent inflamed nodules and abscesses with some sinus tracts and scarring. Treatment includes continuing preventive therapy (daily topical clindamycin) and further medical or surgical intervention. These patients may be treated with oral doxycycline, which is typically continued for several months. However, the sinus tracts of Hurley stage II hidradenitis suppurativa will not resolve with antibiotic therapy and often require surgical deroofing. Hurley stage III is characterized by diffuse involvement of the affected area with fluctuant nodules, interconnecting sinus tracts, and extensive scarring. Although oral antibiotics may be adequate in some of these patients, some require adalimumab. Surgical options for the management of hidradenitis suppurativa include punch debridement, unroofing of sinus tracts or abscesses, and wide excision. The choice of procedure depends on the severity of the lesion. Wide excision is typically used in patients with chronic or extensive disease and provides the best chance for permanent relief in patients who have failed to respond to medical interventions and have disease that is too advanced for surgical unroofing. Incision and drainage have a limited role in the treatment of hidradenitis suppurativa. This treatment method is mainly used in the setting of a tense abscess that requires immediate pain relief. However, lesions caused by hidradenitis suppurativa that are managed with incision and drainage often recur. Adalimumab (A) is a biologic treatment that can be used in patients with hidradenitis suppurativa who do not respond to oral antibiotics, oral retinoids, or hormonal therapies. Adalimumab is a tumor necrosis factor-alpha inhibitor, and it is administered subcutaneously. Adalimumab is typically used for Hurley stage III, which is defined by severe and refractory disease. Oral doxycycline (B) can be given over prolonged courses to prevent or reduce the frequency of new lesions. Oral doxycycline is often used for moderate hidradenitis suppurativa (Hurley stage II). Wide excision (D) is a surgical treatment for hidradenitis suppurativa. Wide excision is used for patients with chronic or extensive disease. It consists of wide excision of the entire affected area and provides the best chance for permanent relief.
An 18-year-old man presents to the office with a new-onset rash for two weeks. He has a family history of hyperthyroidism and diabetes mellitus. He is a senior on his high school wrestling team and reports a high-stress lifestyle. He reports no recent sick contacts but does report receiving a course of amoxicillin for a sore throat last week. On physical exam, 1-1.5 cm silvery, annular plaques on an erythematous base located around the umbilicus and unevenly dispersed on his torso are noted. Which aspect of his historical information is most concerning for psoriasis? AA family history of hyperthyroidism BA high-stress lifestyle CA recent streptococcal pharyngitis infection DInvolvement on the wrestling team
Correct Answer ( C ) Explanation: Psoriasis is a chronic inflammatory skin disease affecting less than 10% of the population. There are two peaks for the age of onset: one between the ages of 30-39 years of age and another between the ages of 50-69 years of age. Psoriasis affects men and women equally. The pathophysiology of psoriasis reveals a complex immune-mediated disease in which T lymphocytes play a central role. Risk factors for psoriasis include a family history of psoriasis; smoking; obesity; medications such as beta-blockers, lithium, and antimalarial drugs; alcohol; vitamin D deficiency; and stress. Infrequently, rare life-threatening presentations can occur that require intensive inpatient management. Psoriasis may present in a variety of forms, each of which has characteristic physical exam findings. The different types of psoriasis are chronic plaque psoriasis, guttate psoriasis, pustular psoriasis, inverse psoriasis, and nail psoriasis. Chronic plaque psoriasis is the most common. It presents as erythematous, sharply defined macules and plaques that have a silvery scale. Chronic plaque psoriasis is commonly found on the scalp, extensor elbows, knees, periumbilical abdomen, and gluteal cleft. The location of chronic plaque psoriasis can often differentiate it from other common skin conditions. Guttate psoriasis has plaques that are similar in appearance to plaque psoriasis, however, the plaques are typically no larger than 1 cm and appear on the trunk and extremities. Guttate psoriasis typically occurs in children or young adults and has a strong association between a recent streptococcal pharyngitis infection or another infection. Pustular psoriasis presents with acute onset of widespread erythema, scaling, and sheets of superficial pustules. This specific type of psoriasis is considered life-threatening, as renal, hepatic, or respiratory abnormalities and sepsis are potential complications. Inverse psoriasis is less common and often presents in the intertriginous areas, which is opposite (or the "inverse") of where the more common plaque psoriasis presents. Nail psoriasis is the second most common presentation of psoriasis and can occur prior to cutaneous eruption of the disease. The nails may exhibit pitting, or small circular indentations, across the entire nail. It is possible to diagnose psoriasis by history and physical exam alone, however, to confirm clinical suspicion, a skin biopsy may be performed. Numerous topical and systemic therapies are available for the treatment of psoriasis. Appropriate treatment is chosen on the basis of disease severity, patient preference, efficacy, and patient response. Topical therapies include emollients, potent topical steroids, topical vitamin D, and calcineurin inhibitors. Systemic therapies may have increased efficacy in some patients but also carry increased side effects. These therapies include methotrexate, acitretin, apremilast, and various biologic agents. Additionally, ultraviolet light is recognized as beneficial for the control of psoriatic skin lesions. Patients diagnosed with psoriasis should be assessed for signs or symptoms of psoriatic arthritis and other comorbidities. A family history of hyperthyroidism (A) has no correlation with a new history of psoriasis, although they both may have autoimmune characteristics. A high-stress lifestyle (B) may contribute to psoriatic flares but does not necessarily potentiate new-onset psoriasis. Involvement on the wrestling team (D) would not be a causative factor for psoriasis. Tinea corporis is the common skin dermatitis found in wrestlers.
A 28-year-old man presents to the clinic desiring treatment for his psoriasis. On exam, several erythematous plaques with silver scales are noted on the anterior and posterior aspects of his trunk. The total body surface affected is approximately 3%. The patient denies joint pain, fever, or fatigue. Which of the following is the best choice for first-line treatment of this patient's psoriasis? AOral adalimumab BOral methotrexate CTopical betamethasone dipropionate cream 0.05% DTopical tacrolimus ointment 0.1%
Correct Answer ( C ) Explanation: Psoriasis is a chronic skin disorder involving erythematous, pruritic plaques with a silvery scale that can appear on almost any skin surface, including the scalp and nails. Variations of the disease include guttate, plaque, inverse, and pustular manifestations. A systemic form of the disease also exists and can manifest as psoriatic arthritis, an inflammatory polyarthropathy. The exact etiology of psoriasis is unknown, but genetics, behavior, and environment all play a role. Risk factors for psoriasis include certain gene mutations, cigarette smoking, alcohol consumption, obesity, vitamin D deficiency, stress, bacterial or viral infections, and use of certain drugs, such as beta-blockers, antimalarials, and lithium. Diagnosis of psoriasis is made clinically but can be confirmed on biopsy if necessary. Treatment regimens for psoriasis depend on the disease manifestation, patient preferences, cost considerations, and comorbidities. For uncomplicated plaque psoriasis not involving the face or intertriginous areas and not involving more than 5-10% of the total body surface area, potent topical corticosteroids, such as betamethasone dipropionate, are the first-line treatment of choice. For patients with psoriasis that affects the face or intertriginous areas, where skin has a greater propensity to thin and atrophy, less potent topical steroids or topical tacrolimus ointment can be prescribed. Patients with involvement of the scalp may need special scalp preparations of topical corticosteroids, such as foams or shampoos. Psoriasis involving more than 5-10% of a patient's total body surface area should be treated with a systemic medication as topical medication becomes costly and impractical on large areas. Systemic medication should also be considered for patients with lesions on the hands, feet, or face that lead to decreased social or occupational function. Systemic medications for psoriasis include methotrexate, retinoids, and biologic agents. Ultraviolet light therapy can also be used either at home or in a clinic setting. Patients with psoriasis should be counseled to lose weight, stop smoking, decrease alcohol consumption, increase outdoor activities, and decrease stress levels. Oral adalimumab (A) is a biologic agent that has shown efficacy in the treatment of psoriasis. This medication is expensive and has many adverse side effects, including an increased risk of certain cancers and bacterial and fungal infections. Biologic agents should be reserved for patients with widespread or debilitating disease. Oral methotrexate (B) is a disease-modifying antirheumatic drug that is also used to treat psoriasis. The side effects of methotrexate include blood dyscrasias, hair loss, renal impairment, hepatic impairment, and gastrointestinal upset. Methotrexate treatment should be reserved for patients with psoriasis that affects more than 5-10% of their total body surface or for patients with lesions that affect their occupational or social functioning. Topical tacrolimus ointment 0.1% (D) is a calcineurin inhibitor that is useful in mild psoriasis and on psoriatic plaques involving the face or intertriginous areas as a corticosteroid-sparing agent.
Which of the following is a risk factor for developing stasis dermatitis? AHypertension BMale sex CObesity DSedentary lifestyle
Correct Answer ( C ) Explanation: Stasis dermatitis occurs in individuals with chronic venous insufficiency. The condition is characterized by chronic inflammatory changes in the skin of the lower extremities. It is often associated with varicose veins and dependent edema. Skin changes include hyperpigmentation, eczema, and fibrosis. Ulcerative skin lesions can occur as skin under pressure breaks down. Risk factors for developing stasis dermatitis include older age, female sex, obesity, and standing frequently. Conditions that place patients at risk include those that have the potential for creating edema such as congestive heart failure. Patients present with edema of the bilateral lower extremities with patches of erythematous or eczematous skin. The medial ankle is the most common area to find skin changes. Patients may complain of itching or have excoriations or lichenification from scratching. Occasionally, the skin may be weeping or have vesicles present. Diagnosis is clinical, although testing with Doppler ultrasound can identify venous insufficiency. Management of stasis dermatitis is accomplished through minimizing the effects of venous insufficiency through continuous compression and occasionally surgical procedures. Skin care involves the use of emollients and lotions to keep the skin from drying and to minimize itching. Management of acute stasis dermatitis may involve topical corticosteroids or wet dressings. Antibiotics may be warranted in patients with a concurrent cellulitis or impetiginized lesions. Hypertension (A) alone is not a risk factor for developing stasis dermatitis, however, hypertension can lead to increased venous pressure or congestive heart failure, which can cause stasis dermatitis over time. Therefore, hypertension is considered an aggravating factor but not a risk factor. Females, not males (B) have a greater risk for developing stasis dermatitis. A sedentary lifestyle (D) does not increase the risk of stasis dermatitis but may increase the risk of obesity. Occupations that require standing for long periods of time increase the risk.
A 68-year-old man presents to the clinic complaining of a lesion on his face that has slowly been growing over the past few years. He denies trauma, pain, or pruritis. On exam, a 6 mm well-demarcated erythematous nodule with a scaly appearance and an area of ulceration is noted on the left cheek. No pus, telangiectasias, or enlarged lymph nodes are noted in the area. Of the following skin disorders, which is the most likely diagnosis? AMalignant melanoma BMerkel cell carcinoma CNummular eczema DSquamous cell carcinoma
Correct Answer ( D ) Explanation: Cutaneous squamous cell carcinoma is an epithelial malignancy found in greater proportion on sun-exposed areas of the skin, although lesions may be found almost anywhere on the body. Risk factors for the development of squamous cell carcinoma include exposure to ultraviolet radiation, immunosuppression, ionizing radiation, chronic inflammation, and genetic variables. Using sunscreen and protective clothing and avoiding tanning beds can decrease a patient's risk for the development of squamous cell carcinoma. Cutaneous squamous cell carcinoma can present as erythematous papules, plaques, or nodules with different degrees of scaliness or ulceration. Squamous cell carcinoma in situ is often well-demarcated, while the lesions of invasive squamous cell carcinoma can be indurated, firm, hyperkeratotic, and at times, ulcerated. The level of local tumor invasion is often not apparent on physical exam and must be distinguished using biopsy. Definitive diagnosis of cutaneous squamous cell carcinoma is made with biopsy, and treatment is dependent on the tumor size, location, and histology, as well as level of local or distant spread. For squamous cell carcinoma in situ, excision with Mohs microsurgery is the treatment of choice. Patients with local lymph node enlargement on physical exam should undergo excision of the lesion along with lymph node dissection and adjuvant chemotherapy, radiation, or both. Malignant melanoma (A) is an aggressive form of skin cancer that is prone to metastasis. The lesions of melanoma are most often hyperpigmented macules. The lesions grow slowly but metastasize early, so prompt excision is warranted. Survival is well-correlated with tumor stage at the time of diagnosis. Merkel cell carcinoma (B) presents as a bluish-red, dome-shaped nodule that is most commonly found on sun-exposed areas of the body. Merkel cell carcinoma is also aggressive and characteristically arises quickly and painlessly. Most patients with Merkel cell carcinoma have lymph node involvement and metastases at the time of diagnosis, and the prognosis is poor. Nummular eczema (C) can present as a longstanding scaly, erythematous lesion in any area of the body. However, these lesions tend to be very pruritic, and multiple lesions are more common. Usually, the patient has a history of atopy. Topical steroid creams are the first-line treatment of choice.
A 25-year-old man presents to the clinic complaining of erythematous, pruritic, and tender bumps on his buttocks. He says that the bumps sometimes have pustules at the end. He states that he sweats a lot and does not have good personal hygiene. He denies recent exposure to a heated swimming pool or hot tub. Which of the following is the most likely diagnosis? AHidradenitis suppurativa BKeratosis pilaris CPseudomonas aeruginosa folliculitis DStaphylococcus aureus folliculitis
Correct Answer ( D ) Explanation: Folliculitis is inflammation of the superficial or deep portion of the hair follicle. Deep infection of the hair follicle can lead to the development of a furuncle (boil). Multiple furuncles can coalesce together to form carbuncles. The most common etiologic pathogen of furuncles and carbuncles is Staphylococcus aureus. Infectious etiologies usually cause folliculitis. Infectious etiologies may be bacterial, fungal, viral, or parasitic. The most common infectious pathogen of folliculitis is Staphylococcus aureus, but Pseudomonas aeruginosa is another possible cause. Risk factors for bacterial folliculitis include nasal carriage of Staphylococcus aureus, hyperhidrosis, scratching secondary to pruritic skin disease, prolonged application of topical corticosteroids, long-term oral antibiotic therapy for acne, shaving against the direction of hair growth, and exposure to hot tubs or heated swimming pools. Folliculitis classically presents with follicular pustules and follicular erythematous papules. Associated symptoms may include pruritus (more common) or pain. Folliculitis can occur on any hair-bearing area, however, the scalp, face, upper trunk, buttocks, legs, and axilla are particularly common. The diagnosis of folliculitis can often be made clinically based on the patient's history and clinical findings. Diagnostic studies that can help in atypical cases include Gram stain, bacterial culture, potassium hydroxide (KOH) preparation, and immunofluorescent testing. The management of folliculitis varies based on the suspected etiology and severity. Mild folliculitis due to Staphylococcus aureus may resolve without medical treatment. Patients with known or suspected Staphylococcus aureus folliculitis that persists and involves a limited area of skin should be treated with topical antibiotic therapy rather than oral therapy. Topical antibiotics that cover Staphylococcus aureus include mupirocin and clindamycin. Patients with more extensive Staphylococcus aureus folliculitis or recurrent folliculitis should be treated with oral antibiotic therapy. Recommended oral agents include cephalexin and dicloxacillin. However, if methicillin-resistant Staphylococcus aureus is suspected, then trimethoprim and sulfamethoxazole, clindamycin, or doxycycline should be used. Personal hygiene by washing with soap and water can help prevent folliculitis. Most lesions resolve without scarring, however, postinflammatory hyperpigmentation may occur. Hidradenitis suppurativa (A) is a recurrent chronic inflammatory skin disorder characterized by the development of inflamed nodules, abscesses, and sinus tracts. The most common locations are the axillary and inguinal areas. The location of the lesions and history of hyperhidrosis in this vignette suggest Staphylococcus aureus folliculitis more than hidradenitis suppurativa. Keratosis pilaris (B) is a common disorder of follicular keratinization that presents with asymptomatic papules on the upper arms, face, thighs, and buttocks. It is most common in children and young adults. The history of pruritus, tenderness, and pustules support Staphylococcus aureus folliculitis more than keratosis pilaris in this patient. Pseudomonas aeruginosa folliculitis (C) is also known as hot tub folliculitis. It is a gram-negative folliculitis that is associated with recent contact with contaminated water, usually due to inadequate chlorination of a hot tub or heated swimming pool. The classic presentation of hot tub folliculitis is pruritic follicular macules, papules, or pustules that typically occur eight to 48 hours after exposure to contaminated water. Lesions usually occur on the trunk or buttocks in the distribution of the wet bathing suit that contacted the contaminated water. The management of hot tub folliculitis is often supportive as the lesions are often self-limited and resolve within seven to 10 days with good skin hygiene. Oral ciprofloxacin can be used for severe cases or in immunocompromised patients. The lack of exposure to heated contaminated water in this vignette supports Staphylococcus aureus folliculitis rather than Pseudomonas aeruginosa folliculitis.
Which of the following findings specifically supports a diagnosis of common warts? ABurrows between papules BPapules in linear configurations CTarget-shaped lesion DThrombosed capillaries on dermoscopy
Correct Answer ( D ) Explanation: Human papillomavirus is a double-stranded DNA virus that infects keratinized skin and causes excess proliferation and retention of the stratum corneum, which leads to papular lesions. Human papillomavirus is transmitted by direct contact from one epithelial surface to another. Most infections resolve within 12 months. However, persistent infection with high-risk human papillomavirus types can increase the risk of precancerous or cancerous lesions. Cutaneous infection with human papillomavirus types 1 and 2 is associated with plantar or common hand warts. Mucocutaneous infection with human papillomavirus types 6, 11, 16, and 18 is associated with genital warts (types 6 and 11) and precancerous and cancerous lesions of the vulva, vagina, cervix, penis, anus, and oropharynx (types 16 and 18 most commonly). Anogenital human papillomavirus is the most common sexually transmitted infection. As with other sexually transmitted infections, individuals with multiple sex partners are at greater risk for human papillomavirus infection compared to those in a monogamous relationship. The vulva, vagina, or cervix is typically affected in women, and the penis is typically affected in men. Men or women who have receptive anal sex with men are at risk for anal human papillomavirus infection. Furthermore, oropharyngeal infection may occur in individuals who engage in oral sex. In addition, human papillomavirus infection is more common among human immunodeficiency virus (HIV) infected men and women than among men and women who are not infected with HIV. Common warts (verruca vulgaris) and plantar warts (verruca plantaris) present as hyperkeratotic papules. The warts may occur as solitary warts, in groups, or as a coalescing plaque. Red or brown dots, which represent thrombosed capillaries and are pathognomonic, may be seen with dermoscopy. The clinical appearance of anogenital warts varies. Lesions may be white, skin-colored, or erythematous and are typically soft. They are sometimes described as cauliflower-like lesions. The diagnosis of cutaneous warts (common warts and plantar warts) and condyloma acuminata is typically made clinically based on history and physical examination findings. Biopsy can be performed if the diagnosis is in question. Cutaneous warts (common warts and plantar warts) are typically treated with topical salicylic acid or liquid nitrogen cryotherapy. Topical salicylic acid exfoliates the affected epidermis and may stimulate local immunity. Cryotherapy is effective but many lesions require multiple treatments. The management of anogenital warts varies, and shared medical decision making between the patient and clinician is appropriate. It is acceptable to simply observe the lesions since about 30% will resolve spontaneously within four months. Treatment options can be divided into treatments applied by the patient and treatments administered by the clinician. Treatments applied by the patient include imiquimod, podophyllotoxin, and sinecatechins. Patients need to be able to identify and reach all lesions to administer their own treatment. Clinician-administered therapies include cryotherapy, surgical excision, and electrosurgery. There are human papillomavirus vaccines that help reduce the prevalence and burden of human papillomavirus infection and diseases associated with human papillomavirus. Human papillomavirus types 16 and 18 cause approximately 70% of all cervical cancers, nearly 90% of anal cancers, and a significant proportion of oropharyngeal cancers, vulvar cancers, vaginal cancers, and penile cancers. These vaccines help prevent both the most likely human papillomavirus types to cause cancer and benign anogenital warts. Routine immunization is recommended for children aged 11-12 years but can be administered starting at 9 years of age. Catch-up vaccination can be offered for girls and women aged 13 to 26 years who have not been previously vaccinated and for boys and men aged 13 to 21 years who have not been previously vaccinated. These vaccinations should ideally be given prior to the individual engaging in sexual activity. The vaccination is administered in two doses to any individual who is younger than 15 years old and is administered in three doses at 0 months, 1-2 months, and 6 months for individuals who are at least 15 years old. Human papillomavirus immunization does not clear current human papillomavirus infection, genital warts, or anogenital intraepithelial neoplasia. In addition, human papillomavirus vaccination status does not impact cervical cancer screening recommendations. However, these vaccines have demonstrated excellent efficacy in preventing human papillomavirus infection and therefore lowering the risk of cervical, vulvar, vaginal, penile, anal, and oropharyngeal cancer. Burrows between papules (A) are a pathognomonic finding of scabies. Scabies presents with papular and pruritic lesions. The pruritus is often severe and worse at night. There are often multiple small erythematous papules, which are often excoriated. Burrows are a pathognomonic finding but are often not visible due to excoriation and secondary infection. Scabies often occurs on the sides of the fingers, webs of the fingers, flexor aspects of the wrists, extensor surface of the elbows, anterior or posterior axillary folds, periumbilical skin, waist, male genitalia, extensor surface of knees, or lower buttocks. Papules in linear configurations (B) is classic for the rash seen with poison ivy dermatitis. The rash often occurs in linear streaks because the part of the skin that came in contact with the plant is often a linear streak. Intense pruritus and erythematous plaques are common findings in poison ivy. Target-shaped lesions (C) are characteristic of Lyme disease and erythema multiforme. Erythema multiforme often occurs secondary to herpes simplex virus. Lyme disease is due to an Ixodes tick bite and the cutaneous lesion is called erythema migrans.
A 40-year-old woman with hepatitis C virus presents to the clinic with a purple, papular, pruritic, and polygonal rash on her ankles and volar wrists. Which of the following additional findings is characteristic of the most likely diagnosis? AErythema migrans BInvolvement of the palms and soles CVesicular lesions on an erythematous base DWickham striae
Correct Answer ( D ) Explanation: Lichen planus is a mucocutaneous disorder that may affect the skin (cutaneous lichen planus), oral cavity (oral lichen planus), genitalia, scalp, nails, or esophagus. Lichen planus is uncommon, and cutaneous lichen planus is thought to occur in less than 1% of the population. It most commonly develops between the ages of 30 and 60 years, and there does not appear to be a sex or racial predilection. The etiology of lichen planus is unknown, but immune-mediated theories have been proposed. There has been consideration that lichen planus is associated with hepatitis C virus. Although the association is controversial, it is reasonable for clinicians to have a high index of suspicion for lichen planus in patients with hepatitis C virus. Some medications, angiotensin-converting enzyme (ACE) inhibitors, beta-blockers, chloroquine, hydroxychloroquine, and thiazide diuretics, have been associated with causing lichen planus. The classic manifestation is a papulosquamous eruption characterized by flat-topped and violaceous papules on the skin. It is often described as the four "Ps:" pruritic, purple (violaceous hue), polygonal, and papule or plaque. Individual papules are usually a few millimeters in diameter but may coalesce to form larger plaques. Fine white lines, called Wickham striae, may be visible on the surface of papules or plaques of cutaneous. The extremities, especially the ankles and volar surfaces of the wrists, are common sites for cutaneous lichen planus. Intense pruritus is associated with lichen planus, and presentations without pruritus are rare. Cutaneous lichen planus lesions often cause postinflammatory hyperpigmentation. Oral lichen planus may occur in conjunction with cutaneous involvement or independently. Genital lichen planus in men presents with violaceous papules on the glans of the penis, while in women, lesions typically occur on the vulva. The diagnosis of lichen planus can often be made clinically based on characteristic findings. Skin biopsy can confirm the diagnosis in cases in which the diagnosis is uncertain. Dermoscopy can help identify Wickham striae on cutaneous lesions. Furthermore, given the possible association with hepatitis C virus, testing for hepatitis C virus is recommended in all patients diagnosed with lichen planus. Cutaneous lichen planus is a self-limited disorder. Therefore, treatment is focused on managing pruritus and accelerating symptom resolution. Topical corticosteroids, such as betamethasone dipropionate, are commonly used as first-line treatment for localized cutaneous lichen planus. Intralesional corticosteroids may be needed in patients with particularly thick lesions. Systemic corticosteroid therapy, such as prednisone, or phototherapy are used in patients with more widespread disease. Oral retinoids, such as acitretin, are an alternative treatment for widespread lichen planus. Oral antihistamines, such as hydroxyzine, may be used for pruritus. Genital lichen planus is typically treated with topical corticosteroids or topical calcineurin inhibitors. Topical corticosteroids are also first-line therapy for oral lichen planus. Drug-induced lichen planus should always be considered so that the offending agent can be identified and removed. Erythema migrans (A) is the characteristic target-shaped skin lesion associated with Lyme disease. It occurs at the site of the tick bite within seven to 14 days. Erythema migrans lesions are not painful but may occasionally itch or burn. They are most commonly found near the axilla, inguinal region, popliteal fossa, or at the belt line. Central clearing is another classic finding of erythema migrans. Involvement of the palms and soles (B) occurs in Rocky Mountain spotted fever and secondary syphilis. Rocky Mountain spotted fever presents with systemic symptoms and headache followed by a generalized rash that begins between the third and fifth days of illness. The rash is typically an erythematous, blanchable rash with macules that become petechial over time. Secondary syphilis presents with systemic symptoms, diffuse lymphadenitis, and a generalized rash. The rash characteristically can involve the palms and soles. Vesicular lesions on an erythematous base (C) is characteristic of rashes caused by a herpes virus, such as varicella, herpes zoster, and herpes simplex virus.
A 35-year-old Caucasian woman presents with a variably pigmented macule with irregular borders on her right anterior thigh. The lesion is 8 millimeters in diameter, and she states it has been growing for the past month. What is the most likely diagnosis? AAcral lentiginous melanoma BLentigo maligna melanoma CNodular melanoma DSuperficial spreading melanoma
Correct Answer ( D ) Explanation: Melanoma is the most serious form of skin cancer, and in the United States, it is the sixth most common cancer in men and women. Risk factors for melanoma include personal or family history of melanoma, high sun exposure (history of sunburns), fair skin, and having more than 50 common nevi. There are four major subtypes of invasive cutaneous melanoma: superficial spreading (most common), nodular melanoma, lentigo maligna, and acral lentiginous. Most melanomas arise as superficial tumors that are confined to the epidermis, where they may remain for several years. This stage is known as the horizontal growth phase, and melanomas in this stage are nearly always curable by surgical resection. Melanomas that infiltrate into the dermis are in a vertical growth phase and have metastatic potential. Nodular melanomas have no horizontal growth phase and are in the vertical phase from inception. Breslow depth (the vertical thickness of the tumor) measured in millimeters is the best predictor of the probability of metastasis. Superficial spreading melanoma accounts for 70% of all melanomas. Evaluating for risk factors during the history is also important. For example, a personal or family history of melanoma and history of excessive sun exposure increase the risk of melanoma. The classic presentation is a variably pigmented macule or thin plaque with an irregular border. The size ranges from a few millimeters to several centimeters in diameter. Most of these tumors are diagnosed while they are thin (less than 1 millimeter) and highly curable. Superficial spreading melanoma can arise from any location but has a predilection for the back in men and women and the lower extremities in women. Two-thirds of melanomas arise de novo, while one-third arise from a preexisting nevus. Overall, early clinical predictors of possible melanoma include asymmetry, irregular borders, variegated color, diameter ≥ 6 millimeters, and a recent change in or development of a new lesion. This is referred to as the ABCDE mnemonic, which is explained further in the image below. In addition, a skin lesion that looks different from surrounding lesions is called an "ugly duckling" sign and is an important finding in patients with multiple nevi. Primary care clinicians who identify a skin lesion that is not clearly benign should have a relatively low threshold for referral to a dermatologist for dermoscopic examination and possible biopsy. For lesions that are considered borderline, it is often helpful to obtain dermoscopic imaging and then have the patient follow-up for repeat imaging in three months. Dermoscopy examination should be performed on all suspicious pigmented lesions. This technique is commonly used in dermatologic settings and requires training. In trained clinicians, dermoscopy improves both the sensitivity and specificity of the clinical diagnosis of melanoma. Furthermore, it improves confidence in the diagnosis of benign lesions and reduces the number of unnecessary biopsies. The diagnosis is definitively confirmed on histology with biopsy. Complete full-thickness excisional biopsy of suspicious lesions with 1- to 3-millimeter margins of normal skin is recommended, including subcutaneous fat. Shave biopsies should not be performed if melanoma is being considered. Once the diagnosis of melanoma is confirmed, surgical excision with an adequate margin of normal tissue is required. The recommended width of surrounding local normal tissue varies based on the thickness of the primary tumor. The recommended margin of normal tissue for melanomas ≤ 1 millimeter thick is 1 centimeter. The recommended margin is 1 or 2 centimeters for melanomas that are between 1 and 2 millimeters in thickness. Lymph node biopsy is recommended for melanomas with thickness ≥ 0.8 millimeters and in those with melanomas < 0.8 millimeters if ulceration is present. Tumor thickness is the single most important determinant of prognosis. Acral lentiginous melanomas (A) account for less than 5% of melanomas. However, these are the most common type of melanoma among dark-skin individuals. Acral lentiginous melanomas occur most frequently on the palmar or plantar surface or in the subungual areas (beneath the nail plate). They most commonly present as dark brown or black, irregularly pigmented macules or patches with raised areas, ulceration, bleeding, or large diameter. Acral lentiginous melanomas can occasionally present as amelanotic or hypomelanotic lesions, which makes the diagnosis more difficult and can mimic benign conditions. The ethnicity and location of the melanoma distinguished acral lentiginous melanoma from superficial spreading melanoma in this patient. Lentigo maligna melanoma (B) most commonly arises in chronically sun-damaged areas of the skin, such as the head and neck, in older individuals and presents with tan or brown macules. These lesions gradually enlarge over years and may develop asymmetric darker foci of pigmentation. These cases account for 10-15% of melanomas. The age of the patient and location of the lesion in the vignette make superficial spreading melanoma the more likely choice. Nodular melanoma (C) is the second most common type and accounts for between 15 and 30% of all melanomas. This melanoma may present as darkly pigmented pedunculated or polypoid papules or nodules. Sometimes nodular melanomas present with uniform color, amelanotic, symmetric borders, and relatively small diameter, which makes early detection difficult. Most nodular melanomas are diagnosed at more than 2 millimeters of thickness. The vignette describes the lesion as a macule, which makes superficial spreading melanoma more likely. Furthermore, superficial spreading melanoma is the most common melanoma.
A 24-year-old man with a history of atopic dermatitis is recently diagnosed with nummular eczema. Which of the following best describes the appearance of his skin on physical exam? APresence of a few erythematous, scaly, well-demarcated, rough plaques that are thickened with exaggerated skin lines BPresence of a few round, erythematous, scaly plaques with raised erythematous narrow borders and central clearing CPresence of multiple round-to-oval erythematous plaques that appear on the face and scalp DPresence of multiple "coin-shaped" erythematous plaques that are on the arms and legs
Correct Answer ( D ) Explanation: Nummular dermatitis, also known as nummular eczema, is an inflammatory condition of the skin that is characterized by the presence of well-demarcated round-to-oval "coin-shaped" erythematous plaques. The cause of nummular dermatitis is unknown but is likely multifactorial in etiology. Dermatitis is one of the most common dermatologic conditions, with nummular eczema considered a subcategory of the larger disease umbrella. It is often associated with dry skin, which is thought to cause the epidermal skin to leak, allowing environmental allergens to penetrate and cause an inflammatory response. It is also associated with the use of certain medications, such as interferon and ribavirin. It is seen more in men than women. Patients will have a positive history of eczema, atopic dermatitis, or dry and sensitive skin. It is a chronic condition that is often recurrent, often waxing and waning in the winter due to cold and dry climates. Patients will present with characteristic round-to-oval erythematous plaques that may be several centimeters in diameter. The plaques are commonly found on the extremities, especially the legs, although they may affect the trunk, hands, and feet but spare the face and scalp. Pruritus is the most common complaint, often being worse at night. It can spread from lesion to lesion causing multiple plaques and can coalesce to form bigger plaques. Excoriation can cause erosions and lead to complications, such as secondary infections, with the most common offending organism being Staphylococcus aureus. Diagnosis is based on clinical findings, although a skin biopsy can be used to differentiate between other dermatological conditions, such as tinea corporis and psoriasis. Treatment targets rehydrating and repairing the epidermal barrier, reducing inflammation, avoiding harsh chemicals (laundry detergents), and treating any secondary infections. Patients are advised to avoid hot showers and baths and take lukewarm or cool baths and showers instead preferably only once per day. Soap should be avoided and moisturizers should be applied to damp skin after baths and showers. High- or ultra-high potency topical corticosteroids are first-line therapy, and should be applied once or twice a day for two to four weeks or until lesions are fully resolved. Soaking the lesions first and then immediately applying the steroid ointment before the skin dries can enhance treatment. Occlusive dressings may also increase corticosteroid penetration in the skin for a more effective response. Topical immunomodulators (e.g., tacrolimus and pimecrolimus) also reduce inflammation. Phototherapy with ultraviolet B can be helpful in generalized or prolonged eruptions. Oral antibiotics, such as cephalexin and erythromycin, can be used in cases of secondary infection. Oral and topical antihistamines, such as hydroxyzine, can reduce itching. Some severe and refractory cases may require narrowband ultraviolet B phototherapy, systemic steroid treatment and systemic immunosuppressive medications, such as methotrexate. Isolated lesions may be treated with intralesional injections of triamcinolone. Patients should be advised to keep the skin well-hydrated with heavy moisturizers or petroleum jelly and to void any offending allergens and irritants. Room humidifiers can also be helpful, especially in the winter and summer months when the heater or air conditioner is being used. Presence of a few erythematous, scaly, well-demarcated, rough plaques that are thickened with exaggerated skin lines (A) describes lichen simplex chronicus that can develop as a complication of nummular eczema with chronic excoriation. Presence of a few round, erythematous, scaly plaques with raised erythematous narrow borders and central clearing (B) best describes tinea corporis, which is a differential diagnosis that should be ruled out when diagnosing nummular eczema. Presence of multiple round-to-oval plaques that appear on the face and scalp (C) is incorrect because nummular eczema does not affect the face and scalp.
A 65-year-old woman with a history of inadequately controlled diabetes mellitus and recurrent cellulitis presents to the clinic with concerns for yellow, brittle toenails. Her recent laboratory studies indicate normal liver function. Moderate onychomycosis is diagnosed at this time. Which of the following antifungals would be most indicated for this patient? ACiclopirox BEfinaconazole CTavaborole DTerbinafine
Correct Answer ( D ) Explanation: Onychomycosis is defined as a fungal infection of the nail. This condition is very common and is estimated to be the cause of disfigured nails in 50-60% of cases worldwide. It is more common in adults. Patients who are immunosuppressed are at higher risk of more severe presentation and superimposed bacterial infections presenting as concomitant cellulitis. Dermatophytes, particularly Trichophyton rubrum, are the main cause, but yeasts and nondermatophyte molds can also be culprits. Data on risk factors are very limited, but proposed risk factors include autoimmune conditions such as psoriasis or inflammatory bowel disease, hyperhidrosis, tinea pedis, conditions that potentially reduce lower extremity circulation, trauma, fitness, age, and smoking, among several others. History of the illness usually details a slowly progressing disfigurement or yellowing of the nails. Onychomycosis can present with pain in the toes or feet, splitting or disfigurement of the nail plates, discoloration usually with yellowing, and subungual hyperkeratosis. Physical exam should also include a general dermatologic evaluation to examine for other causes of nail dystrophy such as psoriasis of lichen planus. Not all cases require treatment, but higher-risk comorbidities necessitate treatment of the infection. Besides diabetes, other reasons that warrant treatment are history of cellulitis, discomfort or pain associated with the infection, immunosuppression, and cosmetic appearance if the patient desires treatment. Diagnosis is often made clinically, but it is important to consider other causes of nail dystrophy. Laboratory studies should be obtained according to the literature. A fungal culture, KOH preparation, or periodic acid-Schiff stain are helpful for confirming the diagnosis. Terbinafine via the oral route is a first-line agent indicated for patients presenting with mild to moderate onychomycosis. Onychomycosis caused by yeast (mainly Candida albicans) or nondermatophyte molds should be treated with oral itraconazole, though terbinafine has been shown to be effective as well. The patient can also be instructed on the use of petrolatum for softening and later clipping or debridement of the affected nails. All treatment regimens tend to have a relatively high risk of failure, and recurrence rates have been estimated to be 20-50%. If treatment failure occurs with first-line therapy, combination with a topical agent and a different oral agent can be attempted and referral should be made to a dermatologist. Ciclopirox (A), efinaconazole (B), and tavaborole (C) are all topical agents that are neither first line nor initially appropriate for a patient at baseline higher risk of systemic complications. Generally, topical therapies are poorly effective for onychomycosis because of poor penetration of the nail plate.
A 32-year-old woman presents to the clinic with a pruritic diffuse rash one hour after eating a crab cake. The lesions are well-circumscribed, raised, erythematous, and have a circular shape. Which of the following is the most likely diagnosis? AAllergic contact dermatitis BAtopic dermatitis CTinea corporis DUrticaria
Correct Answer ( D ) Explanation: Urticaria (hives) is a common disorder mediated by cutaneous mast cells in the superficial dermis. Mast cells and basophils release multiple mediators (including histamine) that cause itching and vasodilation. Vasodilation leads to localized swelling in the uppermost layers of the skin. The same process is responsible for angioedema (localized subcutaneous or submucosal swelling), however, angioedema is due to the release of mast cells deeper in the skin in the deep dermis and subcutaneous tissue. There are many causes of new-onset urticaria, however, in many cases, a cause cannot be identified. Common causes include infections; allergic reactions to medications, foods, or insect stings and bites; and reactions to medications that may cause nonallergic mast cell activation, such as narcotics, nonsteroidal anti-inflammatory drugs, muscle relaxers, and radiocontrast medium. Infectious causes may include viral, bacterial, or parasitic infections. Acute infections are a particularly common cause of urticaria in children. Urticaria as an allergic reaction to foods, medications, insect stings and bites, or latex is an immunoglobulin E-mediated allergic reaction. Urticaria caused by an allergic reaction typically occurs within minutes to two hours of exposure to the allergen. Patients who have an allergic reaction of urticaria are at risk of angioedema or anaphylaxis on subsequent exposure to that allergen. Urticaria can be categorized as acute or chronic. Urticaria is considered acute when it has been present for less than six weeks. Chronic urticaria is defined by recurrent urticaria with symptoms or signs most days of the week for six weeks or longer. Urticarial lesions are blanchable, circumscribed, raised, and erythematous plaques or wheals, which often have central pallor. The shape varies and may be round, oval, or serpiginous. Lesions range from less than 1 centimeter to several centimeters in diameter. Urticarial lesions are intensely itchy, and the pruritus may disrupt work, school, or sleep. Symptoms often seem most severe at night. The lesions of urticaria are transient, usually appearing and enlarging over the course of minutes to hours and then disappearing within 24 hours. Lesions often coalesce as they enlarge. Any area of the body may be affected, but areas that are compressed by clothing (under waistbands) or intertriginous areas (axillary or inguinal areas) may be affected more severely. Urticaria is diagnosed clinically based on the history and appearance of the lesions. Laboratory testing is not indicated in patients with urticaria without evidence of an underlying disorder. Serum testing for allergen-specific immunoglobulin E antibodies is appropriate for patients with a history of exposure to a possible allergic cause (e.g., medication, food, or insect sting). The exposure typically occurs within two hours of the development of urticaria. For example, crab-specific immunoglobulin E testing can be obtained in an individual who develops hives within 10 minutes of eating a crab cake. An allergist should perform these allergy tests. The initial treatment of new-onset urticaria (with or without angioedema) should focus on the short-term relief of pruritus and angioedema and avoidance of precipitating factors. Most cases of new-onset urticaria are self-limited and resolve spontaneously. H1 antihistamines are often used in the treatment of urticaria. This medication class can be divided into first-generation agents, such as diphenhydramine and hydroxyzine, and second-generation agents, such as loratadine and cetirizine. The newer second-generation H1 antihistamines are recommended. The benefits of second-generation H1 antihistamines are less frequent dosing, less sedation, and less anticholinergic adverse effects. In some individuals, H1 antihistamines clear the urticarial lesions, however, in other individuals, the H1 antihistamines simply flatten the lesions and reduce itching. H2 antihistamines, such as famotidine and ranitidine, have also shown some benefit in the treatment of patients presenting to the emergency department with acute urticaria. Glucocorticoids do not appear to be necessary for isolated urticaria. However, a brief course of glucocorticoids, such as a week or less, may be added to antihistamine therapy for patients with prominent angioedema or urticaria persisting beyond a few days. Patients who are suspected of having an allergic etiology causing new-onset urticaria, such as food or medication allergy, should be referred to an allergy specialist. Allergic contact dermatitis (A) presents with erythematous and indurated scaly plaques. Vesiculation and bullae are seen in severe cases. Edema can be prominent in areas with thin skin, such as the eyelid, genitals, and the lips. The lesion distribution is usually localized to the skin areas that come in contact with the allergen. The most common areas affected are the hands, face, and eyelids. Pruritus is the dominant symptom. Urticaria can be distinguished from allergic contact dermatitis in this vignette by the history of crab cake exposure and the diffuse involvement of the rash. Allergic contact dermatitis typically occurs four to 96 hours after exposure in individuals who are sensitized. Urticaria often begins within minutes or a couple hours. Atopic dermatitis (B) presents with dry skin and severe pruritus. However, the presentation varies in severity and the location of the lesions vary based on a patient's age. Most patients (85%) who are affected by atopic dermatitis will have symptom onset prior to 5 years of age. Atopic dermatitis also has an allergy-based etiology. The exposure to the crab cake immediately prior to the onset of symptoms and the description of classic urticarial lesions suggests urticaria more than atopic dermatitis. Tinea corporis (C) is a tinea dermatophyte infection occurring in sites other than the feet, hands, groin, or face. Trichophyton rubrum is the most common etiologic pathogen. The classic presentation of tinea corporis is a pruritic, circular or oval, erythematous, and scaling patch or plaque. Central clearing with an active and advancing peripheral border is also classic. It is transmitted by direct contact with infected humans or animals. The exposure of the individual in the vignette to the crab cake and lack of exposure to a source of tinea corporis makes urticaria the most likely diagnosis.
Which of the following is a classic finding associated with nonbullous impetigo? AAdherent golden crusts BChristmas tree distribution CRash involvement of the palms or soles DTarget lesion
Correct Answer ( A ) Explanation: Impetigo is a common superficial bacterial skin infection. Children between 2-5 years of age are the most common demographic affected. Staphylococcus aureus is the most common cause, but Streptococcus pyogenes is another common cause. Transmission of impetigo occurs by direct contact. Impetigo can be classified as primary or secondary. Primary impetigo occurs at sites of previously healthy skin, whereas secondary impetigo occurs at sites with preceding skin trauma, such as eczema or minor abrasions. There are three clinical variants of impetigo: nonbullous impetigo, bullous impetigo, and ecthyma. Nonbullous impetigo is most common and presents with characteristic golden-crusted lesions. The lesions progress over the course of one week chronologically from papules to vesicles surrounded with erythema to pustules, which enlarge and breakdown to form the classic adherent golden crusts. Bullous impetigo classically presents as flaccid bullae with clear yellow fluid. Bullous impetigo is caused by strains of Staphylococcus aureus that produce an exfoliative toxin. Bullous impetigo typically occurs on the trunk. Ecthyma manifests as punched-out appearing ulcers with raised and violaceous borders. Impetigo may rarely cause complications, such as poststreptococcal glomerulonephritis or rheumatic fever. Impetigo is diagnosed clinically. Gram stain or culture of pus or exudate may be obtained to identify the causative pathogens. Treatment of impetigo is important for reducing the spread of infection, expediting the resolution of symptoms, and improving cosmetic appearance. Antibiotics are used to treat impetigo. The route of administration varies based on the number of lesions. Topical antibiotics, such as topical mupirocin or retapamulin, are used to treat patients with a limited number of lesions. Patients with numerous lesions (involvement of more than one area) should be treated with oral antibiotics, such as cephalexin or dicloxacillin. Hand washing is important to reduce the spread of impetigo amongst children. Children with impetigo can return to school 24 hours after beginning an effective antimicrobial therapy. Christmas tree distribution (B) is seen in patients with pityriasis rosea. Pityriasis rosea is an acute exanthematous skin condition that begins with a herald patch, which is a larger lesion preceding other lesions. The lesions classically occur on the trunk in the oblique direction following the cleavage lines, described as a Christmas tree pattern. Rash involvement of the palms or soles (C) can be seen in hand, foot, and mouth disease; secondary syphilis; or Rocky Mountain spotted fever. Nonbullous impetigo typically occurs on the face or extremities. Target lesion (D) may be seen with erythema migrans of Lyme disease or with erythema multiforme.
A 30-year-old woman presents to the emergency department with well-circumscribed and erythematous raised lesions that appeared today one hour after starting amoxicillin. The lesions are itchy. She has no symptoms or signs of a systemic allergic reaction. Which of the following is the best initial treatment? ACetirizine BCetirizine and prednisone CEpinephrine DTopical triamcinolone acetonide
Correct Answer ( A ) Explanation: Urticaria is a common disorder marked by pruritus and swelling of the skin that may occur with or without angioedema (swelling of the deeper dermis and subcutaneous tissue). The pathogenesis of urticaria involves the activation of mast cells in the upper dermis leading to the release of histamine and other vasoactive mediators, which cause itching and swelling. The etiology of urticaria cannot always be identified but common causes include allergic reactions (to medications, foods, insect stings and bites, latex etc.), reactions that cause nonallergic mast cell activation, and infections (viral, bacterial, and parasitic). Urticaria caused by allergic reactions typically occurs within minutes to two hours of allergen exposure. Allergic reactions may be limited to the skin or part of a systemic reaction with possible anaphylaxis. Clinical manifestations that indicate a possible systemic allergic reaction include hypotension, syncope, wheezing, nausea, vomiting, and swelling of the airway. Urticaria is often classified according to its chronicity. Acute urticaria is defined by the presence of urticaria for less than six weeks. Urticaria is considered chronic when symptoms are present on most days of the week for six weeks or longer. Acute and chronic urticaria present with the same lesions and therefore are indistinguishable at initial presentation. Individuals with new-onset urticaria may have an identifiable trigger, such as medications, food ingestions, insect bites, or infections. However, in many cases, there is no identifiable trigger. Urticaria presents as circumscribed and erythematous raised lesions that often have central pallor. The shape of urticaria may be round, oval, or serpiginous, and the size of the lesions varies from less than 1 centimeter to several centimeters. Urticaria is associated with intense pruritus that often seems worse at night. Individual lesions are often transient, appearing and enlarging over the course of minutes to hours and then disappearing within 24 hours. Lesions often coalesce as they enlarge. Urticaria is diagnosed clinically based on the history of exposures and clinical manifestations. Serum tests for allergen-specific immunoglobulin E antibodies can be performed to confirm an allergy in patients with a suspected allergen trigger. The treatment of new-onset urticaria is focused on the short-term relief of pruritus. Two-thirds of cases of new-onset urticaria will be self-limited and resolve spontaneously. However, pharmacologic therapy can help speed up the resolution of symptoms. Second-generation H1 antihistamines, such as cetirizine, are typically used for the treatment of urticaria. There is no data suggesting that one second-generation antihistamine is more effective than another. First-generation H1 antihistamines, such as diphenhydramine or hydroxyzine, are often helpful at bedtime because of the sedating effect. H2 antihistamines are also often used in the treatment of acute urticaria. There is one study that supports H2 antihistamines, such as ranitidine or famotidine, as an efficacious treatment in patients presenting to the emergency department with acute urticaria. Glucocorticoids are typically not indicated in cases with acute urticaria without angioedema. However, in cases where urticaria persists for a couple of days, a brief course (less than one week) of systemic glucocorticoids, such as prednisone, can be used. Antihistamines should be continued until several days after the resolution of the urticaria. Chronic urticaria is diagnosed when symptoms persist on most days for more than six weeks. The lesions are identical to acute urticaria. No specific cause can be identified in 80-90% of patients with chronic urticaria. Laboratory testing should include a complete blood count with differential, erythrocyte sedimentation rate, C-reactive protein, and thyroid-stimulating hormone. However, these results are typically normal. Second-generation H1 antihistamines are the initial treatment in patients with chronic urticaria. However, 50% of patients will not achieve complete control of symptoms with H1 antihistamines at the initial dose. In patients who do not achieve complete control of symptoms, the treatment options include adding a different second-generation antihistamine, adding an H2 antihistamine, adding a leukotriene-receptor antagonist, or increasing the dose of the second generation H1 antihistamine. Systemic glucocorticoids are reserved for short-term control of refractory symptoms. Once symptoms are controlled, the treatment regimen should be continued for at least one to three months before attempting to taper the doses. Cetirizine and prednisone (B) is incorrect because prednisone is not indicated as part of the initial treatment of urticaria. It is used in patients with persistent urticaria who are not responding to antihistamines and in cases of angioedema. Epinephrine (C) is not indicated in patients who present solely with urticaria. Epinephrine is used in systemic allergic reactions, such as anaphylaxis. Topical triamcinolone acetonide (D) is a topical corticosteroid. Topical corticosteroids are not used in the treatment of urticaria.
A 65-year-old woman presents to her dermatologist with a complaint of a small growth on her face for the past six months. She has a history of tanning and using tanning beds at least once per week. On physical exam, there is a 1 cm, pink, pearly papule with a rolled border. A telangiectasia can be seen just below the surface of the lesion. Which of the following is the most likely diagnosis? AAcrochordon BBasal cell carcinoma CMelanocytic nevi DSeborrheic keratoses
Correct Answer ( B ) Explanation: Basal cell carcinoma is a skin cancer that arises from the epidermis. While it is not likely to metastasize, it can be locally invasive affecting not only the skin but also the surrounding structures, including the bone. Risk factors include fair skin, age over 50, geographic location near the equator, exposure to ultraviolet radiation, Caucasian descent, medications, and immunosuppression. The lesion usually presents in areas of solar radiation exposure, most commonly on the face. There are three clinical presentations: nodular, superficial, and morpheaform. Nodular presentation is the most common presentation. It is a translucent pink or flesh-colored papule with a rolled border and frequent telangiectasia. When ulcerated, these lesions are known as rodent ulcers. Superficial carcinomas appear more frequently in men and are seen on the trunk. These pink to light red lesions may be slightly scaly and have an atrophic center. They may be a small macule a few millimeters in diameter or become as large as a few centimeters. They are usually asymptomatic. Morpheaform carcinomas are smooth erythematous plaques or papules that may be firm or indurated. Diagnosis can be made clinically with punch or shave biopsy to confirm the diagnosis. Biopsy can be done with definitive treatment by excision or prior to definitive treatment. The benefit of performing the biopsy prior to treatment is minimizing the risk of undertreating an aggressive lesion. Preventing the condition requires minimizing exposure to ultraviolet radiation from the sun or tanning beds. Using sunscreen and avoiding tanning beds can also minimize the risk of developing skin cancers. Nonsteroidal anti-inflammatory use has also been associated with a decreased risk of developing basal cell carcinoma. Once a diagnosis is established, if the lesion is considered a low risk for recurrence based on clinical features and pathology, electrodesiccation and curettage is the recommended treatment. Mohs surgery, surgical excision, and radiation therapy are indicated treatments if there is a concern of recurrence. An acrochordon (A) is a common benign growth known as a skin tag. Skin tags present as pedunculated lesions on narrow stalks and are commonly seen in patients who are diabetic or pregnant. Melanocytic nevi (C) are commonly known as moles. They are a benign proliferation of melanocytes. They present as pigmented or pink macules, papules, or plaques. Seborrheic keratoses (D) are benign proliferations of keratinocytes that are identified by their stuck-on appearance. They are round, hyperpigmented lesions with a dull, verrucous surface.
Which physical exam finding differentiates erysipelas from herpes zoster? AAcute onset of symptoms with systemic manifestations BNondermatomal involvement CPositive Hutchinson sign DSkin erythema and edema
Correct Answer ( B ) Explanation: Erysipelas is a soft tissue bacterial infection that occurs most commonly in young children and older adults. Most cases of erysipelas infections are caused by beta-hemolytic streptococci. Less common causes of cellulitis include Haemophilus influenzae type B, Clostridia, Streptococcus pneumoniae, and Neisseria meningitidis. The pathogen typically begins at a site of trauma or break in the skin and spreads to the lymphatic system. Risk factors for erysipelas include skin barrier disruption due to trauma, pre-existing skin infection, skin inflammation, edema, obesity, diabetes, or immunosuppression. Erysipelas is rarely life-threatening, and most cases resolve after antibiotic therapy without complication or sequelae. Erysipelas typically presents as an area of skin erythema, edema, and warmth. There is often a clear demarcation between the involved and uninvolved tissue. There may also be a raised, advancing border or erythema with central clearing. Although erysipelas may resemble tinea corporis, there is substantial edema and warmth, which will help differentiate erysipelas from tinea corporis. Edema surrounding the hair follicles may lead to dimpling in the skin, creating a "peau d'orange" appearance. Involvement of the ear, called Milian ear sign, is a distinguishing feature for erysipelas and will differentiate erysipelas from cellulitis since this region does not contain deeper dermis tissue that is affected by cellulitis. Nondermatomal involvement is a reliable way to differentiate erysipelas from herpes zoster. Although erysipelas is nearly always unilateral, the area of involvement is not restricted by dermatomes. The lower extremities are the most common site of involvement. In erysipelas, there is typically an acute onset of symptoms with systemic manifestations including fever, chills, severe malaise, and headache, which may precede cutaneous symptoms by minutes to hours. The diagnosis of erysipelas is based on clinical manifestations. Laboratory testing is not required for patients with uncomplicated infection in the absence of comorbidities or complications. A skin biopsy may be appropriate if the clinical diagnosis requires confirmation. Since most cases of erysipelas are a result of beta-hemolytic streptococci, penicillin is first-line therapy. It may be administered intramuscularly or orally, and the treatment course should last five days. Oral antibiotic therapy is appropriate in patients with mild infection. Parenteral antibiotic therapy may be indicated in instances of suspected toxicity, rapid progression of erythema, worsening of symptoms after 48 hours of oral antibiotic therapy, or inability to tolerate oral therapy. For patients with a penicillin allergy, a first-generation cephalosporin may be considered. Additional considerations to antibiotic therapy include symptomatic treatment of fever and headaches, increased hydration, cold compress, and elevation of the affected limb to reduce edema, inflammation, and pain. Patients with recurrent erysipelas infections should be evaluated for predisposing skin lesions, such as tinea pedis, intertrigo, and stasis ulcers. Acute onset of symptoms with systemic manifestations (A) may be found in both erysipelas and herpes zoster. Systemic manifestations common in both diseases include fever, chills, severe malaise, and headache. A positive Hutchinson sign (C), defined as herpetic vesicles appearing on the tip of the nose, is a physical exam finding specific to herpes zoster infection. This physical exam finding raises concerns for the development of ophthalmic herpes zoster and requires an urgent ophthalmology evaluation. The rash of herpes zoster begins as erythematous papules that evolve into grouped vesicles. Hutchinson sign is not indicative of erysipelas. Skin erythema and edema (D) are common findings associated with both erysipelas and herpes zoster. Skin erythema and edema alone will not accurately differentiate erysipelas from herpes zoster.
A 40-year-old man presents to the clinic with a skin-colored, cauliflower-shaped lesion on his penis. It is painless. He is diagnosed with condyloma acuminata. Which of the following is the best treatment? AFluconazole BImiquimod CPenicillin DValacyclovir
Correct Answer ( B ) Explanation: Human papillomavirus is a DNA virus that infects the outer layer of the skin (stratum corneum) and forms papular lesions. The transmission of human papillomavirus occurs by direct contact of epithelial surfaces. Transmission may occur from one individual to another or from one site to another on the same individual (autoinoculation). The clinical manifestations of human papillomavirus include both benign cutaneous warts, such as common warts and genital warts, and an increased risk of certain cancers that are associated with high-risk human papillomavirus types. Specific human papillomavirus subtypes are associated with different clinical manifestations. Human papillomavirus types 1 and 2 frequently cause plantar warts and common warts. Human papillomavirus types 6 and 11 typically cause genital warts, and human papillomavirus types 16 and 18 cause precancerous and cancerous lesions of the vulva, vagina, cervix, penis, anus, and oropharynx. Cutaneous warts include common warts and plantar warts. Common warts (verruca vulgaris) present most often as skin-colored papules. Plantar warts (verruca plantaris) occur on the palmar or plantar aspect of the hands or feet. Cutaneous warts may be solitary or occur in groups. Anogenital warts caused by human papillomavirus (condyloma acuminata or genital warts) are the most common sexually transmitted infection. Risk factors for anogenital warts include new sexual partners, multiple sexual partners, and human immunodeficiency virus infection. Lesions most commonly occur on the vulva, vagina, or cervix in women and the penis in men. The shape of the lesions may be flat, dome-shaped, verrucous, or cauliflower-shaped. Genital warts may be white, skin-colored, pink or erythematous, hyperpigmented, or brown. The lesions are typically soft to palpation and range in size from 1 millimeter to several centimeters in diameter and are either asymptomatic (most common) or pruritic. Cutaneous warts and genital warts are diagnosed based on clinical appearance. Dermoscopy can support the diagnosis of cutaneous warts. Dermoscopy is performed using the dermatoscope to look at the lesions. The classic findings are thrombosed capillaries, which appear as homogenous red or black dots, and interruption of dermatoglyphics (skin lines). Cutaneous warts are often treated with salicylic acid or cryotherapy (liquid nitrogen). Topical salicylic acid exfoliates the affected epidermis and may stimulate local immunity. Cryotherapy is effective, but lesions often require multiple treatments. The management of anogenital warts varies, and shared medical decision making between the patient and clinician is appropriate. It is acceptable to simply observe the lesions since about 30% will resolve spontaneously within four months. Treatment options can be divided into treatments applied by the patient and treatments administered by the clinician. Treatments applied by the patient include imiquimod, podophyllotoxin, and sinecatechins. Patients need to be able to identify and reach all lesions to administer their own treatment. Vaccines are available to prevent infections and increased risk of cancer associated with human papillomavirus. Human papillomavirus types 16 and 18 cause approximately 70% of all cervical cancers, nearly 90% of anal cancers, and a significant proportion of oropharyngeal cancers, vulvar cancers, vaginal cancers, and penile cancers. Routine immunization is recommended for children aged 11-12 years but can be administered starting at 9 years of age. Catch-up vaccination can be offered for girls and women aged 13 to 26 years who have not been previously vaccinated and for boys and men aged 13 to 21 years who have not been previously vaccinated. Vaccinations are most effective when given prior to the individual engaging in sexual activity. The vaccination is administered in two doses (at least six months apart) to any individual who is younger than 15 years old and is administered in three doses at 0 months, 1-2 months, and 6 months for individuals who are at least 15 years old. Human papillomavirus immunization does not clear current human papillomavirus infection, genital warts, or anogenital intraepithelial neoplasia. However, these vaccines have demonstrated excellent efficacy in preventing human papillomavirus infection and, therefore, lowering the risk of cervical, vulvar, vaginal, penile, anal, and oropharyngeal cancer. Fluconazole (A) is used to treat candidiasis infections. Balanitis due to Candida infection is a common cause of pain or a rash on the penis. It commonly occurs in patients with uncontrolled diabetes mellitus and is more common in uncircumcised men. Penicillin (C) injections are used to treat syphilis. Primary syphilis presents with a chancre, which is a painless ulcerative lesion that frequently occurs on the penis or vagina. Valacyclovir (D) is an antiviral agent that is used to treat genital herpes simplex virus infections. Herpes simplex presents with painful erythematous and vesicular lesions. The presentation of the patient in this vignette is most consistent with condyloma acuminata.
A 20-year-old woman presents to her primary care provider with a complaint of an itchy rash. She states that she had a headache and sore throat about one week ago. She noticed an itchy spot on her chest a few days ago, but today the rash is all over her back and between her breasts. On physical exam, the initial patch is pink, oval-shaped, and about 5 cm in diameter with a central clearing surrounded by a scaly collar. The newer lesions are smaller and are seen in the cleavage lines and proximal extremities. A potassium hydroxide preparation is negative. Which of the following is the most likely diagnosis? AGuttate psoriasis BPityriasis rosea CSecondary syphilis DTinea versicolor
Correct Answer ( B ) Explanation: Pityriasis rosea is an acute dermatologic condition. This self-limited condition is thought to be associated with human herpesvirus 7 although other viral associations are possible. It is commonly seen in older children and younger adults. The clinical presentation may be asymptomatic or a viral prodrome may be present. Headache, malaise, and pharyngitis may present prior to the rash presentation. The initial presentation of the rash of pityriasis rosea is a "herald" patch. This solitary oval lesion is sharply demarcated and salmon or pink in color. It is usually between 2 and 5 cm in diameter. It will likely appear on the back, neck, or chest. The pruritic lesion becomes scaly and begins to clear centrally. Within a few days to two weeks, there will be an eruption of smaller, similar lesions. These lesions develop from the top down on proximal extremities and in cleavage lines of the skin, predominantly on the back. Because of their appearance on the cleavage lines, a pattern resembling a Christmas tree forms. The erythema and pruritus gradually fade as the area desquamates. This scaling may take four to six weeks or several months. Areas of hyperpigmentation may persist, especially in darker skin tones. While the condition is most common in young adults, in children the presentation following the herald patch may be atypical and affect the face, scalp, and distal extremities as opposed to the trunk. Diagnosis is made clinically based on history and presentation of the rash. Potassium hydroxide preparations, punch biopsies, and serologic testing may be advised to rule out other conditions. Treatment is generally limited to education and reassurance as the condition resolves within two to three months. Pruritus can be managed with short-term corticosteroid creams. Antiviral medications, such as acyclovir, and phototherapy may accelerate the healing process but are only recommended in more severe cases due to risks associated with the treatment. Women may be at increased risk of spontaneous abortion if they develop the condition while pregnant. Guttate psoriasis (A) presents as small, scaly plaques distributed on the trunk following a streptococcus infection. The presentation does not begin with a herald patch. Secondary syphilis (C) will present as a red-brown papulosquamous eruption that will also be seen on the palms and soles. While there is no herald patch, a chancre may precede the eruption. If the patient is sexually active an RPR may be ordered if diagnosis is not clear. Tinea versicolor (D) is a fungal infection that presents as hyperpigmented or hypopigmented macules on the neck or trunk. A potassium hydroxide preparation would be positive.
A 20-year-old man presents to the clinic with a fluctuant abscess at the superior aspect of the intergluteal cleft. He is diagnosed with a pilonidal abscess. Which of the following is a risk factor for this condition? AActive lifestyle BAfrican American race CIncreased hair density in the natal cleft region DSmoking cigarettes
Correct Answer ( C ) Explanation: Intergluteal pilonidal disease is an infection involving the skin and subcutaneous tissue at or near the upper part of the gluteal cleft of the buttocks. The cause of pilonidal disease is thought to be acquired rather than congenital. It is likely due to friction on the skin overlying the natal (gluteal) cleft. Damage to hair follicles creates a port of entry for hair and debris. Subsequent infection results in pilonidal abscess formation. The average age at presentation is 19 years of age in women and 21 years of age in men. Men are affected two to four times more often than women. Risk factors include elevated body mass index, local trauma or irritation to the superior aspect of the intergluteal cleft, prolonged sitting, deep natal cleft, increased hair density in the natal cleft region, and family history. The clinical presentation of pilonidal disease varies from asymptomatic pilonidal cavities to an acute infection or chronic inflammation with intermittent drainage. The symptoms of acute infection include sudden onset of pain in the intergluteal region. The pain is typically worse while sitting or performing activities that stretch the overlying natal cleft skin, such as bending and sitting up. Patients may also report intermittent swelling with possible mucoid, purulent, or bloody discharge. Systemic symptoms, such as fever and malaise, may occur in individuals with an undrained abscess. Chronic pilonidal disease presents with recurrent or persistent drainage and pain. There may be one or more areas of drainage, typically from sinus tracts. Pilonidal disease can be identified on physical examination by retracting the buttock cheeks enough to visualize the pores (pits) or sinuses within the valley of the natal cleft. Midline pores in the natal cleft support the diagnosis. Pilonidal disease is a clinical diagnosis that is made without laboratory testing or imaging. The differential diagnosis should also include perianal abscess, anorectal fistula, perianal complications of Crohn disease, buttock skin abscess, folliculitis, and hidradenitis suppurativa. Management of pilonidal disease should be based on the acuity of presentation and extent of disease. Asymptomatic disease should not be treated with surgical excision because of the morbidity associated with the procedure. In addition, it is not known whether removing gluteal cleft hair decreases the chances of asymptomatic individuals becoming symptomatic. Acute pilonidal abscess should be treated with incision and drainage. Antibiotics alone will not typically be successful in treating a significant pilonidal abscess. Antibiotics should be used in those with abscess and surrounding cellulitis or only cellulitis. There is no evidence to suggest that antibiotics benefit patients with an acute abscess without cellulitis. However, antibiotics should be considered in immunocompromised patients, patients at high risk for endocarditis, patients with methicillin-resistant Staphylococcus aureus, and in patients with concurrent systemic illness. Metronidazole plus cephalexin is a reasonable antibiotic regimen. Chronic or recurrent disease presents with either recurrent abscess or one or more persistently draining sinuses. Surgical treatment is required for chronic pilonidal disease. Active lifestyle (A), African American race (B), and smoking cigarettes (D) are not known risk factors for pilonidal disease.
A 27-year-old pregnant woman presents to the dermatologist complaining of worsening acne vulgaris. You decide to start her on medical therapy as her lesions are cystic and demonstrate a propensity to scar. Which of the following medications is absolutely contraindicated in this patient? AOral erythromycin BTopical azelaic acid CTopical salicylic acid DTopical tazarotene
Correct Answer ( D ) Explanation: Acne vulgaris is a dermatologic condition characterized by papules and pustules that predominantly affect the face, chest, and back. It is caused by plugged follicles, increased sebum production, presence of Propionibacterium acnes (also called Cutibacterium acnes), and inflammation. Acne vulgaris most commonly presents between ages 10 to 15 years old and lasts for five to 10 years. Familial inheritance may play a role in acne vulgaris with a 75% chance of development in children whose parents are both affected. Lesions can be classified as either inflammatory or noninflammatory with the former including papules, pustules, and nodules (cysts) and the latter including open comedones (blackheads) and closed comedones (whiteheads). Patients who pick papules may have eroded lesions and scarring, termed acne excoriée. Women with persistent acne and evidence of a hyperandrogenic state should be tested for androgen excess and associated underlying conditions, such as congenital adrenal hyperplasia and polycystic ovarian syndrome. Laboratory testing involves testosterone, follicle stimulating hormone, luteinizing hormone, and dehydroepiandrosterone sulfate. Infectious folliculitis can be ruled out with fungal and bacterial cultures of lesions. Patients should be instructed to wash their face twice daily with a mild soap and tepid water followed by a moisturizing lotion. Topical medical therapies are first-line treatment for acne vulgaris and take at least two months to demonstrate their full effect. Control of acne requires continual and prolonged compliance with treatment strategies. Prescription and over-the-counter medications contain sulfur, salicylic acid, resorcinol, and benzoyl peroxide. Topical retinoids (e.g., tretinoin, adapalene, tazarotene) are comedolytic medications that correct follicular plugging. Adapalene 0.1% gel is the only topical retinoid available without a prescription and does not cause photosensitivity, which is found in other medications of this class. Tazarotene is administered using short-contact therapy, that is, it is applied and subsequently washed off at increasing intervals. It carries a pregnancy category X warning and should not be prescribed to pregnant or nursing women. Irritation, flaking, and dryness of the skin are common adverse effects of topical retinoid formulations. These side effects can be minimized by administering the lowest efficacious dose. Topical antimicrobials are used to control inflamed pores and may be combined with night-time use of retinoids. Benzoyl peroxide, clindamycin, erythromycin, sulfacetamide, and dapsone are common medications used to decrease the presence of P. acnes. Topical erythromycin and clindamycin are pregnancy category B medications but are not effective as monotherapy and are often combined with benzoyl peroxide. Azelaic acid is used in the treatment of inflammatory and noninflammatory acne and carries a pregnancy category B warning. Oral antibiotics (e.g., azithromycin, clarithromycin, tetracycline) demonstrate better clinical results and a lower rate of relapse if initiated at higher doses and tapered once adequate acne control is achieved. Tetracycline, doxycycline, and minocycline are pregnancy category D medications and should not be prescribed to pregnant or nursing women for the treatment of acne vulgaris. Additionally, these medications are contraindicated in children younger than 12 years old due to the staining of permanent teeth. Oral antibiotic treatment requires weeks to demonstrate efficacy and months to achieve maximal benefit. Photodynamic therapy involves the application of a photosensitizing drug with exposure to a light source (laser or blue light). Patients with nodulocystic acne can be initially treated with the above regimens but should be administered oral isotretinoin if there is no significant response to other therapies after three months. This and other oral retinoids are pregnancy category X medications and use is contraindicated in pregnant patients. Oral prednisone can be prescribed for markedly severe acne cases. Oral contraceptives may provide benefit in female patients. Those unable to take an oral contraceptive or who have relapsed after a course of oral retinoid therapy can be administered spironolactone. This medication is contraindicated in pregnancy but can be coadministered with oral contraceptives. Pigment changes can be treated with hydroquinone, tretinoin, azelaic acid, chemical peels, and laser resurfacing. Patients with acne scarring should be referred to a dermatologic surgeon for treatments including excision, laser therapy, dermabrasion, chemical peels, and injection with fillers (e.g., collagen, fat). Patients may develop significant psychosocial complications secondary to acne vulgaris, such as depression, anxiety, and social withdrawal. Oral erythromycin (A) is an antibiotic medication used to decrease the bacterial burden of P. acnes and decrease inflammatory acne lesions. It carries a pregnancy category B warning, which is defined as no evidence of risk to the fetus in studies, but controlled studies in pregnant women are unavailable. It is administered as a 500 mg oral dose twice daily and may cause gastrointestinal distress. Topical azelaic acid (B) is used to treat both inflammatory types of acne as well as noninflammatory lesions. It carries a pregnancy category B warning, and is a preferred agent in pregnant women. Topical salicylic acid (C) is a comedolytic medication that is applied between one to three times daily. It often causes local skin irritation and dryness and carries a pregnancy category C warning. This category rating corresponds to a risk that cannot be ruled out as animal studies have revealed adverse effects on the fetus without controlled studies in human women. Medications in this class should only be given if the potential benefits justify the potential risk to the fetus.
Which of the following patient populations experiences an increased incidence of lichen planus? APatients taking selective serotonin reuptake inhibitors BPatients who abuse alcohol CPatients with diabetes mellitus DPatients with hepatitis C
Correct Answer ( D ) Explanation: Lichen planus is a skin disorder of unknown etiology. The lesions of lichen planus are usually small, multiple, and may affect the epidermis, the nails, or the mucous membranes. Lichen planus has been described as papulosquamous, purple, polygonal, and pruritic, although variations, such as hypertrophic, atrophic, and bullous, exist. Lichen planus is more common in adulthood and in those taking certain drugs, such as thiazide diuretics, angiotensin-converting enzyme inhibitors, methyldopa, chloroquine, and sulfonylureas. Lichen planus is also more common in patients with hepatitis C. Symptoms of cutaneous lichen planus are intense pruritus and violaceous skin lesions. Symptoms of oral lichen planus include pain with eating, loss of appetite, and oral mucosal plaques, thinning, or ulcerations. Esophageal lichen planus can lead to dysphagia or odynophagia. Lichen planus affecting the nails or scalp may lead to nail loss or alopecia, respectively. Genital lichen planus generally presents as a purple papule or plaque on the glans penis or vulva. On physical exam, the papules or plaques of lichen planus may exhibit fine white striations known as Wickham striae. Diagnosis of lichen planus is made clinically and confirmed with punch biopsy. First-line treatment for lichen planus is with topical corticosteroids or intralesional corticosteroids for thick lesions. Patients with lichen planus who are taking drugs that are known to be associated with lichen planus should be switched to other agents, if possible. Those with unknown hepatitis C status who are at risk should be screened for the disease. Treatment with selective serotonin reuptake inhibitors (A) has not been associated with a higher incidence of lichen planus. Antidepressants that have been associated with lichen planus include amitriptyline and imipramine. The list of medications that are associated with lichen planus is very long and should be consulted whenever a patient presents with a lesion that is consistent with lichen planus. Patients who abuse alcohol (B) do not exhibit a higher incidence of lichen planus than the general population. Diabetes mellitus (C) has not been associated with lichen planus. Many skin conditions are associated with diabetes mellitus, including tinea, candidiasis, bacterial infections, and diabetic dermopathy.
A 72-year-old woman presents for a routine visit and medication refill. During her visit, she mentions that her right big toe has been bothering her. It is painful for her to wear shoes and cut her nails. On exam, a thickened, yellowed toenail is noted, affecting the distal two-thirds of the nail bed of her right great toe. The surrounding skin otherwise appears normal. A potassium hydroxide preparation is taken immediately, and hyphae are noted. A fungal culture is sent, and oral terbinafine is prescribed. Which of the following is the mechanism of action for this treatment? ABinds to membrane sterols and increases membrane permeability BImpairs fungal mitoses by disrupting mitotic spindle formation in fungi CInhibits 14-alpha-demethylase, preventing conversion of lanosterol into ergosterol, which is needed to build fungal cell walls DInhibits the enzyme squalene epoxidase, preventing synthesis of ergosterol, which is needed to build fungal cell walls
Correct Answer ( D ) Explanation: Nail infections caused by fungi (onychomycoses) are most commonly caused by dermatophytes (tinea unguium), yeasts, and nondermatophyte molds. Toenail onychomycosis is usually caused by dermatophytes (most commonly Trichophyton rubrum), and fingernail onychomycosis is usually caused by yeast (most commonly Candida albicans). Nondermatophyte molds rarely cause fingernail onychomycosis but can affect toenails. Risk factors for onychomycosis include diabetes, genetic predisposition, immunodeficiency, increasing age, psoriasis, swimming, and tinea pedis. Onychomycosis should be considered in patients presenting with nail abnormalities such as discoloration, onycholysis (separation of the nail plate from the nail bed), splitting, and subungual hyperkeratosis. Other common diagnoses with similar presentation include nail psoriasis, onychogryphosis, and traumatic nail injury. Patients with onychomycosis may also present with nail bed or plate pain, which is acute or chronic and may be exacerbated by pressure from bed linens, footwear, or nail cutting. There are several different patterns of presentation, but distal lateral subungual onychomycosis is the most common subtype. It most commonly affects the great toe, beginning in a distal corner as a discoloration and gradually spreading to involve the entire nail plate as it extends towards the cuticle. Onychomycosis can also increase the risk of or present with cellulitis or other cutaneous fungal infections. Because other causes of nail abnormalities are common, diagnosis of onychomycosis should be confirmed with either a potassium hydroxide (KOH) preparation demonstrating hyphae or periodic acid-Schiff stain of nail plate clippings. Both of these tests should be accompanied by fungal culture to determine the causative agent. Treatment for onychomycosis depends on severity, causative organism, clinical subtype, treatment availability, side effects, and cost. Onychomycosis is difficult to treat despite the availability of multiple treatment options. The duration of treatment is often very long, and treatment failure and disease recurrence are common. Topical treatment has few to no side effects but is less effective and requires a longer duration of treatment. The first-line treatment for mild, moderate, or severe dermatophyte onychomycosis is oral terbinafine, an antifungal that acts by inhibiting the enzyme squalene epoxidase, which enables the production of ergosterol, a key component of fungal cell walls. Providers should be aware there have been cases of hepatic failure in patients on terbinafine, so baseline liver function tests should be obtained and monitored periodically while on treatment. For patients who cannot tolerate terbinafine, itraconazole is an alternative systemic treatment that acts by inhibiting a different enzyme that also undermines production of ergosterol. Fingernails require at least 6 weeks of treatment, and toenails require at least 12 weeks. First-line options for topical treatment include efinaconazole, amorolfine, tavaborole, and ciclopirox. There are currently insufficient data to compare their efficacies. Binds to membrane sterols and increases membrane permeability (A) describes the mechanism of action for antifungals such as amphotericin B and nystatin. Nystatin is most commonly used to treat candidiasis. Amphotericin B is typically chosen to treat serious fungal infections but is used with caution due to its nephrotoxicity and hepatotoxicity. Impairs fungal mitoses by disrupting mitotic spindle formation in fungi (B) describes the mechanism of action for griseofulvin, which is also an antifungal. It is a weaker antifungal but does accumulate in keratin. While it can be used to treat onychomycoses, a much longer duration of treatment, at least four to six months, is usually needed. Inhibits 14-alpha-demethylase, preventing conversion of lanosterol into ergosterol, which is needed to build fungal cell walls (C) describes the mechanism of action for triazole antifungals, such as itraconazole, the second-line treatment for mild to moderate dermatophyte onychomycosis when oral terbinafine is not tolerated or fails.
A 22-year-old man presents with patches of skin discoloration that he noticed about two weeks ago. He works as a lifeguard during the summer and noticed it after his first week of work. The affected skin is not painful or pruritic. Upon physical exam, you notice some hypopigmented, whitish, oval-shaped macules on the patient's abdomen. The patient is diagnosed with tinea versicolor. Which of the following is the best initial treatment for this condition? AAvoid irritants such as harsh soaps or fragrances, for one month BOral fluconazole once every six months COral fluconazole once per week for four weeks DTopical ketoconazole to be applied daily for two weeks
Correct Answer ( D ) Explanation: Tinea versicolor, or pityriasis versicolor, is a benign fungal infection that is characterized by the development of hypo- or hyperpigmented macules and patches that appear on the chest and back. It is relatively common, with some patients having chronic recurrence of the condition. The yeast genus Malassezia causes the infection, with Malassezia globosa, Malassezia sympodialis, and Malassezia furfur being the main species found in tinea versicolor. This organism can be found both on healthy skin as normal flora and as an opportunistic infection in some people. Most cases of tinea versicolor develop in otherwise healthy individuals with no immunologic deficiencies. Factors that predispose certain patients to develop this condition include genetic disposition (family history), warm and humid environments, immunosuppression, malnutrition, corticosteroid use, and Cushing disease. Certain bath oils and skin lotions may also increase the risk of developing this disease. Tinea versicolor is more common in persons between 15-24 years of age when the sebaceous glands are most active. It is uncommon before puberty or after age 65. Patients will present with oval-to-round patches or macules on the trunk, back, abdomen, and proximal extremities. The macules tend to coalesce, forming irregularly-shaped patches. In patients with fair skin, each lesion varies in color from white to reddish-brown. In darker skinned individuals, there can be varying degrees of hypo- or hyperpigmentation. The lesions will have a fine, dust-like, powdery scale overlying them that can be noticed with light scraping. Most patients are asymptomatic, but some may report mild pruritus. Patients often notice the lesions when those affected areas fail to tan during the summer and become noticeable. Diagnosis is made clinically based on presentation. A dermatoscope is a useful diagnostic tool for examining the lesions. Hypopigmented lesions will appear as well-demarcated areas with fine scales, and hyperpigmented lesions show fine scales overlying brown pigmentation. An ultraviolet black light (Wood lamp) can be used to examine the lesions, which will show a yellow-green fluorescence overlying them. Diagnosis can be confirmed by potassium hydroxide examination with or without staining, which will reveal characteristic short hyphae with spores. Since a fungus that is part of the normal flora is the cause of tinea versicolor, it is not considered contagious. It does not leave permanent scarring or pigmentary changes, and skin color resolves after one to two months of treatment. However, it does have a high recurrence rate, especially in patients with a positive family history. Topical agents, such as selenium sulfide, zinc-pyrithione, sodium sulfacetamide, ciclopirox olamine, tacrolimus, as well as azole (e.g., ketoconazole) and allylamine (e.g., terbinafine) antifungals, can also be used. Treatment should be applied from the neck to the knees even if the lesions are small. Depending on the agent, it should be applied once daily for 10 minutes (or overnight for resistant cases) and then washed off. This regimen should be repeated daily for a duration of two weeks. Weekly application of the agents can be used for several months thereafter to prevent recurrence. Oral antifungals are used for extensive and recurrent disease. Patients only prefer them due to convenience of use but since they have more adverse side effects (e.g., liver damage, harmful drug interactions) they should be used sparingly. Fluconazole is the safest oral agent, if an oral medication is required, and can be dosed weekly for two to four weeks. Oral fluconazole may also be dosed once every six months for patients who have high risk of recurrence. Patients should be educated that the pigmentary changes may take several months to fully resolve. Avoid irritants such as harsh soaps or fragrances, for one month (A) could be helpful if the patient is experiencing pruritus but will not treat the infection. Oral fluconazole once every six months (B) can be used to prevent recurrence of tinea versicolor in patients who are at risk for recurrence but is not an initial treatment. Oral fluconazole once per week for four weeks (C) is an effective treatment but is not the best initial treatment since oral antifungals have a higher risk of adverse side effects than topical agents. Also, this patient does not require oral treatment since his disease is not extensive.
A 20-year-old woman presents to the clinic with an acute itchy rash that began after eating nuts. On exam, there are raised erythematous lesions with central pallor. The lesions are well-circumscribed and transient. Which of the following is the most likely diagnosis? AAllergic contact dermatitis BAtopic dermatitis CInsect bite DUrticaria
Correct Answer ( D ) Explanation: Urticaria (hives) is a common skin disorder marked by swelling and pruritus. Urticaria may be accompanied by angioedema, which is defined as edema of the deeper dermis and subcutaneous tissues. The pathogenesis of urticaria involves the activation of mast cells in the upper dermis leading to the release of histamine and other vasoactive mediators that cause itching and swelling. The etiology of urticaria cannot always be identified. However, common causes include allergic reactions (immunoglobulin E mediated hypersensitivity reactions) to medications (penicillins, cephalosporins, morphine, codeine, dextromethorphan, sulfa drugs, diuretics, aspirin, nonsteroidal anti-inflammatory drugs, and local anesthetics), foods (milk, eggs, nuts, fish, shellfish, and tomatoes), and insect stings and bites; reactions to medications that cause nonallergic mast cell activations (e.g., narcotics); infections (viral, bacterial, or parasitic); pregnancy; latex; and environmental causes (e.g., heat, cold, exercise). Urticaria presents as well-circumscribed erythematous lesions with central pallor. Urticaria varies in morphology, including round, oval, and serpiginous. The size varies from less than 1 centimeter to several centimeters. Urticaria is associated with pruritus. Individual lesions are classically transient and often disappear within 24 hours. Urticarial lesions caused by an allergic reaction typically begin within minutes to hours. Urticaria may occur in isolation or as part of a systemic allergic reaction, which may include anaphylaxis. It can also occur on any location in the body, however, areas compressed by clothing and skin-to-skin areas tend to be affected more often. Urticaria can be classified as acute or chronic based on the duration of symptoms. Urticaria for less than six weeks is considered acute and is considered chronic if more than six weeks. However, both acute and chronic lesions have the same appearance. The diagnosis of urticaria is made clinically. Serum tests for allergen-specific immunoglobulin E antibodies can be performed to confirm an allergy in patients with a suspected allergen trigger. The treatment of new-onset urticaria is focused on the short-term relief of pruritus. Most cases of new-onset urticaria will resolve spontaneously. Pharmacologic treatment may help symptoms resolve faster. H1 antihistamines are used to treat urticaria. First-generation H1 antihistamines, such as diphenhydramine and hydroxyzine, are helpful at night, and second-generation H1 antihistamines, such as cetirizine and loratadine, are helpful during the day. H2 antihistamines, such as ranitidine and famotidine, are also often used in the treatment of urticaria. Glucocorticoids are not always indicated in the treatment of urticaria without angioedema. However, in cases where urticaria persists for a couple of days, a short course of glucocorticoids, such as prednisone, may be used. Allergic contact dermatitis (A) is an allergic dermatitis that occurs from direct skin exposure to a substance. Poison ivy dermatitis is a common example. Allergic contact dermatitis manifests as erythematous papular dermatitis that often occurs with areas of vesiculation. The skin lesions occur in the areas of direct skin contact with the underlying etiology. The history of the rash beginning after eating nuts and the central pallor make urticaria the more likely diagnosis in this vignette. Atopic dermatitis (B) manifests with erythematous patches and papules with scaling. It is intensely pruritic. The most common areas affected in adults are the antecubital and popliteal fossa. The well-circumscribed appearance, central pallor, and history of exposure to nuts make urticaria the more likely diagnosis. Insect bite (C) typically presents with individual lesions that persist for days. Urticarial lesions appear and then disappear within 24 hours.
Which of the following genera of bacteria is most commonly associated with the development of acne vulgaris? ACutibacterium BDemodex CStaphylococcus DStreptococcus
Correct Answer ( A ) Explanation: Acne vulgaris is a dermatologic disease primarily affecting adolescents and young adults. The skin condition is caused by the interaction of four factors: hyperkeratinization of hair follicles, increased sebum production, cutibacterium acnes (formerly known as Propionibacterium acnes) infection, and inflammation. Diet and genetics may also play a role. During puberty, increased androgen production leads to enlargement of sebum glands. Propionibacterium acnes is considered normal skin flora but grows disproportionately in the presence of sebum. This overgrowth of bacteria leads to inflammation and hyperkeratinization of hair follicles. Acne may be mild, involving only small comedones and papules on the chin, nose, and forehead, or may progress to a more widespread pattern with distribution over the entire face, neck, and upper torso. Acne lesions can become pustular or nodular and may lead to scarring. Diagnosis of acne vulgaris is made clinically. Treatment is tailored to the individual patient and may involve topical astringents, topical retinoids, systemic retinoids, or oral antibiotics. Oral retinoids are reserved for severe cases of acne vulgaris as they carry risks for adverse psychiatric effects and hepatic toxicity. Preventing acne is difficult in the adolescent years, but keeping the face clean is essential. Dietary changes and stress reduction have not shown much benefit in the prevention of acne. Certain medications, such as lithium, phenytoin, glucocorticoids, androgens, and vitamin B supplements, can worsen acne. Female patients with acne who show signs of androgen excess, such as hirsutism, should be screened for polycystic ovary syndrome. Demodex (B) follicularis is considered normal human flora but is found in higher-than-average concentrations in patients with rosacea. Staphylococcus (C) aureus is also part of the normal skin flora. Many bacterial skin infections are the result of Staphylococcus aureus, but the bacteria do not play a major role in the development of acne vulgaris. Streptococcus (D) is a gram-positive organism responsible for many human infections, including pharyngitis and pneumonia, but has not been implicated in acne vulgaris.
Which of the following skin lesions can be a premalignant manifestation of cutaneous squamous cell carcinoma? AActinic keratosis BPyogenic granuloma CSeborrheic keratosis DVerruca vulgaris
Correct Answer ( A ) Explanation: Actinic keratosis is a skin lesion caused by atypical epidermal proliferation of keratinocytes, which result from prolonged ultraviolet exposure. While few actinic keratoses eventually develop into squamous cell carcinoma, more than half of all squamous cell carcinomas begin as actinic keratoses. For this reason, actinic keratosis is considered a premalignant manifestation of cutaneous squamous cell carcinoma. Risk factors for the development of actinic keratosis include prolonged sun exposure, fair skin type, and advanced age. Actinic keratoses present as erythematous macules, papules, or plaques with scaly surfaces and irregular borders on sun-damaged skin. Lesions may be singular or multiple and may be small or large. The differential diagnosis of actinic keratosis includes squamous cell carcinoma, psoriasis, solar lentigo, basal cell carcinoma, or inflamed seborrheic keratosis. Diagnosis of actinic keratosis is clinical, and treatment involves application of liquid nitrogen, topical cytotoxic creams, or photodynamic therapy. If the lesion fails to respond to treatment or the diagnosis is suspect, a punch, shave, or excisional biopsy can provide a definitive diagnosis. Prevention of actinic keratosis involves avoiding excess ultraviolet light exposure and applying sunscreen. Pyogenic granuloma (B) is an erythematous, vascular papule that grows in response to tissue injury. The lesion is not a true granuloma and is not infectious. Pyogenic granulomas are benign and do not represent premalignancy. Seborrheic keratosis (C) is a skin lesion common in middle aged and older adults. The appearance is verrucous or scaly, often flesh-colored or hyperpigmented, and papular or plaque-like. Often, the most superficial layers of the lesion can be flaked off, only to regrow in weeks or months. The lesions are not malignant and only need to be removed if cosmetically bothersome. Verruca vulgaris (D) is a common wart caused by the human papillomavirus. The appearance of verruca varies but is usually papular with a variegated surface that is flesh-colored to slightly hyperpigmented. Verruca vulgaris lesions are not malignant and have not yet been shown to be related to squamous cell carcinoma. However, infection with certain strains of human papillomavirus increases the risk for certain cancers, such as cervical cancer and oropharyngeal cancer.
A healthy 30-year-old man with a history of vitiligo presents to his dermatologist with a complaint of burning areas on his scalp and beard and of losing hair in those areas over the past three weeks. He states that his father started losing his hair at about the same age. On physical exam, there are discrete oval-shaped areas of complete hair loss that are edged by short broken hairs. Hair pull test is positive. His fingernails show some pitting. Which of the following is the most likely diagnosis? AAlopecia areata BAnagen effluvium CTinea capitis DTrichotillomania
Correct Answer ( A ) Explanation: Alopecia areata is an autoimmune condition that leads to nonscarring hair loss. Risk factors for this condition include having a first-degree relative with the condition, having trisomy 21, and having other autoimmune disorders, such as thyroiditis and vitiligo. The hair follicles are targeted in the period of active growth, which leads to discrete circular areas of complete hair loss. These areas may merge to create irregular patterns of hair loss. Burning or pruritus may precede the loss of hair. The condition most commonly begins on the scalp. It may be limited in men to the area of the beard or may progress to hair loss of the entire body. White hair may be spared. Nail changes may also be present. Pitting, onycholysis, and trachyonychia are commonly seen nail changes. Depending on the severity of the condition, hair loss may persist indefinitely. Regrowth happens in many patients with remission times varying between recurrence. Diagnosis can be made clinically. Hair pull test will be positive with many hairs easily dislodged. Because of the association with other autoimmune diseases, screening should be performed for thyroiditis. Because this condition can relapse, management should include psychological therapy. There are treatments available that can help manage the condition, however, it is not mandatory to treat. For patchy hair loss, intralesional corticosteroids can be administered or topical steroids can be applied. For more severe cases, topical immunotherapy or systemic glucocorticoids may have some benefit. Cosmetic options are also available for loss of eyelashes or relapsing cases of alopecia areata. Anagen effluvium (B) is the acute loss of hair due to interruption of hair growth in the anagen phase. Hair loss is noticeable within a few weeks of the inciting event. Chemotherapy can cause this type of hair loss. Tinea capitis (C) is a fungal infection of the scalp that results in a scaling, pruritic areas of patchy hair loss. Inflammation and cervical lymphadenopathy may also be present. Trichotillomania (D) is a condition caused by the pulling and manipulation of the hair that often results in unusual patterns of hair loss.
A 30-year-old man presents to his primary care provider with a complaint of new-onset genital rash following recent unprotected intercourse with a new partner. He states that he has had fever, body pain, and painful urination for the past two days. He denies any previous sexually transmitted infections. On physical exam, multiple vesicles on an erythematous base are present in the genital area. Which of the following tests would be most specific and sensitive for diagnosing the suspected condition? APolymerase chain reaction CTzank smear DViral culture
Correct Answer ( A ) Explanation: Genital herpes virus infection is a common infection that can be caused by herpes simplex type 1 virus, HSV-1, or herpes simplex type 2 virus, HSV-2. Risk factors for infection are increasing age, increasing number of sexual partners, and sexual encounters with men. Women and non-Hispanic blacks are at increased risk for contracting the viruses. Both viruses can be present in the same area at the same time. There are three types of infection: primary, nonprimary first episode, and recurrent. An infection is considered primary if there are no preexisting antibodies to either HSV-1 or HSV-2. A non primary first episode occurs when either HSV-1 antibodies are present and a first episode of HSV-2 occurs or if HSV-2 antibodies are present and there is a first episode of HSV-1. Recurrent infection is the reactivation of either HSV-1 or HSV-2. Recurrent episodes are more likely with HSV-2. The infections, regardless of type, may be mild, subclinical, or asymptomatic. The classic presentation of the condition is multiple, painful vesicles on an erythematous base. Primary infections usually present about four days after exposure and may present with painful genital ulcerations and vesicles, fever, dysuria, myalgias, and headache. These symptoms usually resolve within 19 days. Complications can include end-organ disease and central nervous system infection. Urinary retention due to loss of sacral sensation can occur due to lumbosacral radiculomyelitis and viremia. Nonprimary first episodes and recurrent infections are generally less severe with shorter duration and are more likely to be asymptomatic. Recurrent infections may have prodromal symptoms, such as tingling or shooting pains. Once the clinical appearance of the eruption has cleared, asymptomatic viral shedding can occur for a long period of time. During this time, it is possible to unknowingly transmit the infection. Immunosuppressed patients may have more severe recurrence as well as prolonged viral shedding periods. Patients with HSV-2 are at increased risk for acquiring human immunodeficiency virus. Diagnosis of a clinical eruption should be confirmed with laboratory testing. The most specific and sensitive of these tests is the polymerase chain reaction, and it is the standard diagnostic method for diagnosis. Antiviral therapy is indicated to treat this infection. Adjunct therapy includes analgesia and sitz baths. An indwelling catheter may be indicated in the case of urinary retention. Chronic suppressive therapy may also be indicated. Serology (B) will be positive once antibodies to the virus have developed and will persist indefinitely after that time. It will not indicate whether the infection is an active infection or if it was a previous infection. It is also limited in the identification of the site of the infection. Tzank smear (C) has low sensitivity and specificity and is only helpful if positive. Viral culture (D) may be used if lesions are present. It however has a low sensitivity that decreases in value as the lesions begin to heal and is not used first line.
Which of the following factors is included in the ABCDE clinical prediction criteria to determine the risk of malignant melanoma? AAsymmetryCorrect Answer BBleeding CCircumference greater than 15 mm DDiameter greater than 4 mm
Correct Answer ( A ) Explanation: Malignant melanoma is a neoplastic skin disease. It is the fifth most common cancer in men and women in the United States with mortality rates second only to lung cancer. Risk factors for developing malignant melanoma include fair skin (types I and II), presence of atypical nevi, personal history of melanoma, family history of atypical nevi or melanoma, and history of blistering sunburn. Congenital nevi presence increases the risk of melanoma proportional to the size of the initial lesion. Early signs include an increase in size or change in color or shape, while itching is the most common early symptom. Lesions may progress to ulcerations and present with bleeding and tenderness. Characteristic indications of malignant melanoma include asymmetry, border irregularity, color variegation, and diameter ≥ 6 mm (ABCD). Evolution of the lesion is sometimes also included in this recognition acronym (ABCDE). Superficial spreading melanoma is the most common subtype of malignant melanoma and presents with flat, asymmetric lesions larger than 6 mm in diameter that spread on the skin surface over a period of years. Nodular melanoma is the second most common subtype and is most often found on the extremities. Lesions are raised with brown to black pigmentation and rapidly appearing and growing papules. Lentigo maligna is an in situ melanoma that may progress to invasive lentigo maligna melanoma. It is most common in elderly patients and develops over a period of years or decades on sun-exposed Caucasian skin. Lesions are brown, flat, and irregularly outlined with color variation within the lesion. Acral lentiginous melanoma is the most common form of melanoma in Asian and African American patients and is the least common form in Caucasian individuals. It primarily appears on the hands and feet, including the nails. Lesions are similar in presentation to those of lentigo maligna and appear and evolve over years. Clinicians should palpate regional lymph nodes to assess for enlargement with biopsy of suspicious nodes used to further evaluate extra-dermal spread of melanoma. Wood light examination may reveal irregular pigmentation extending beyond clinically evident lesion borders, especially in lentigo maligna. Suspicious skin lesions warrant biopsy with results indicating diagnosis, anatomic site, and whether biopsy margins are involved. Breslow level indicates the vertical thickness of the melanoma and is the single most important prognostic factor. Shave biopsy is not an appropriate technique as it may not demonstrate full Breslow depth of the lesion, and incisional or punch biopsy is preferred. Excisional biopsy should be utilized for lesions highly suspicious of malignant melanoma. Staging of melanoma ranges from stage IA with Breslow depth < 1.0 mm without ulceration to stage IV with the presence of distant metastases regardless of tumor thickness or lymph node status. Stage II lesions demonstrate a Breslow depth of 1.01 to > 4.0 mm in thickness with and without ulceration, depending on subclassifications within this stage. Lymph nodes with micrometastases denote stage III disease. Sentinel lymph node biopsy should be considered for patients with stage IB and stage II melanoma, with results predicting the risk of recurrence and mortality. Stage IIB, IIC, and III melanoma can be treated with adjuvant high-dose interferon therapy. First-degree relatives should be offered full-skin examination to screen for suspicious lesions and should be educated on their increased risk for malignant melanoma. Five-year survival rate decreases proportionally with increasing melanoma stage with stage IA disease having a 95% survival rate while that of stage IV melanoma is only 7-19%. Late diagnosis of melanoma corresponds to a worse prognosis, while female sex and younger age carry a more favorable prognosis. Patients and providers should periodically evaluate pigmented lesions with dermatology referral indicated for routine full body evaluation. Follow-up visits vary based on tumor thickness and number of years after diagnosis. After five years, all patients should be followed annually regardless of primary tumor thickness. Bleeding (B) is not a component in the ABCDE clinical prediction acronym used to determine a lesion's likelihood of malignancy. A dermatologist should evaluate any mole that is itching and bleeding. Circumference greater than 15 mm (C) is not a characteristic feature of malignant melanoma lesions. As many lesions are asymmetric, the diameter is used to determine potential for malignancy. Diameter greater than 4 mm (D) is not part of the clinical prediction acronym for malignant melanoma. Lesions with a diameter greater than 6 mm are more likely to be malignant.
Which of the following is the most important prognostic factor for patients with malignant melanoma? ABreslow level BDiameter CMultiple lesions DRapid lesion evolution
Correct Answer ( A ) Explanation: Malignant melanoma is the fifth most common cancer in the United States with mortality rates secondary only to those of lung cancer. Patients with fair skin (types I and II), presence of atypical nevi, personal history of melanoma, family history of atypical nevi or melanoma, and history of blistering sunburn have an increased risk of developing malignant melanoma. The presence of congenital nevi also increases the risk for malignant melanoma, with larger lesions purporting a higher risk. The most common early symptom of malignant melanoma is itching, and lesions tend to increase in size or change color or shape prior to progressing to ulceration, bleeding, and tenderness. Characteristic indications of malignant melanoma include asymmetry, border irregularity, color variegation, and diameter ≥ 6 mm (ABCD). Evolution of the lesion is sometimes included in this recognition acronym with rapid changes raising suspicion for malignancy (ABCDE). The most common type of malignant melanoma is superficial spreading melanoma. This condition spreads on the skin surface over a period of years and is characterized by flat, asymmetric lesions larger than 6 mm in diameter. Nodular melanoma presents with raised, brown to black pigmented lesions that rapidly appear and grow papules on the extremities. Lentigo maligna is an in situ melanoma that can become invasive. It is most common in elderly patients, especially Caucasian patients, with years of sun exposure. Lesions are brown, flat, and irregularly outlined with color variegation within the lesion. The most common form of melanoma in Asian and African American patients is acral lentiginous melanoma, which primarily affects the hands and feet, including the nails. Regional lymph nodes may be enlarged secondary to invasive disease and should be biopsied to further evaluate extra-dermal spread of melanoma. Wood light examination of lentigo maligna may reveal irregular pigmentation that extends beyond clinically evident lesion borders. Any suspicious skin lesion warrants biopsy. Result reports should include diagnosis, anatomic site, and whether biopsy margins are involved. The vertical depth of the lesion is reported as the Breslow level and is the single most important prognostic factor. Malignant melanoma is staged according to the primary tumor, regional lymph node, and distant metastasis (TNM) staging guideline created by the American Joint Committee on Cancer. Stage 0 melanoma is an intraepithelial, in situ lesion, while stages I and II lesions are characterized by local invasive cutaneous disease. Nodal disease indicates stage III melanoma with distant metastatic disease being termed stage IV melanoma. Referral to a dermatologist is indicated for any lesion that is not clearly benign. Treatment for malignant melanoma includes excision with confirmed noninvolved margins proportional to the depth of the primary tumor. Adjuvant treatments may be needed for lymph node involvement and include checkpoint inhibition immunotherapy involving agents such as nivolumab and pembrolizumab. Dabrafenib, trametinib, and vemurafenib are targeted therapies that can be used in select cases of stage III melanoma. First-degree relatives of patients with confirmed malignant melanoma should undergo full-skin evaluation to screen for suspicious lesions as these patients are at increased risk for developing malignant melanoma. The five-year survival rate worsens as the stage of melanoma progresses with stage IA disease carrying a 95% five-year survival rate and stage IV disease carrying a 7-19% survival rate. Early diagnosis may lead to a better prognosis, and patients should periodically be evaluated for new or changing lesions. Patients with confirmed malignant melanoma should have follow-up visits based on tumor thickness and the length of time that has passed since initial diagnosis. All patients should be followed annually after five years regardless of primary tumor thickness. Diameter (B) can be assessed in conjunction with other findings to determine the increased likelihood of malignancy for a lesion. A lesion with a diameter > 6 mm is more likely to be malignant and should be biopsied or further evaluated by a specialist. The presence of multiple lesions (C) does not worsen the prognosis of malignant melanoma. Those with more than 50 melanocytic nevi are at an increased risk for developing melanoma. Rapid lesion evolution (D) is another component of the clinical prediction acronym ABCDE that should be evaluated when determining the propensity of a suspicious lesion for cancer.
Which of the following distinguishes pemphigus vulgaris from other autoimmune blistering diseases? AFlaccid bullae BLocalized distribution of bullae CMucous membrane involvement DSystemic illness
Correct Answer ( A ) Explanation: Pemphigus is a group of four types of blistering disorders including pemphigus vulgaris, pemphigus foliaceus, immunoglobulin A (IgA) pemphigus, and paraneoplastic pemphigus. It is thought to be caused by autoantibodies targeting epithelial cell surface antigens or components of desmosomes. Risk factors for developing pemphigus vulgaris include a genetic predisposition, UV radiation, preceding viral infection, or certain drugs. Drugs that have been implicated in causing pemphigus vulgaris are captopril, penicillamine, rifampin, and nonsteroidal anti-inflammatory drugs. Pemphigus vulgaris is characterized by mucosal involvement, acantholytic blistering, and the presence of serum autoantibodies against desmoglein 3. This autoimmune disease is rare, occurring in adults between the ages of 40 and 60. Men are more commonly affected than women. Patients often initially present with mucosal lesions. Flaccid blisters will develop on cutaneous surfaces. Blisters will develop on normal-appearing skin when pressure is placed on the area (Nikolsky sign). A histopathology report from biopsy will show intraepithelial cleavage with acantholysis in the suprabasal region, basal keratinocytes along the basement membrane zone, and inflammatory infiltrate in the dermis. Prednisone is the preferred first-line treatment for initial management. Localized distribution of bullae (B) can be seen in other bullous diseases such as leukocytoclastic vasculitis, however, the distribution of the bullae associated with pemphigus vulgaris will be generalized. Mucous membrane involvement (C) is almost always associated with pemphigus vulgaris but does not distinguish the disease from other autoimmune blistering diseases such as bullous pemphigoid or linear IgA bullous dermatosis. Systemic illness (D) is typically seen in other disease processes such as Steven-Johnson syndrome or Staphylococcal scalded skin syndrome
A 52-year-old woman presents to her dermatologist with a complaint of persistent facial redness and flushing that affects her cheeks, nose, chin, and forehead. She states that if she has spicy foods or is in the sun, her skin becomes very dry and stings. On physical exam, there is facial erythema in the central region of her face. No pustules or phymatous changes are present. Which of the following pharmacologic agents would be most effective in managing her facial erythema? ATopical brimonidineCorrect Answer BTopical metronidazole CTopical permethrin DTopical retinoids
Correct Answer ( A ) Explanation: Rosacea is a common, chronic, inflammatory skin condition that primarily affects the center portion of the face most commonly seen in light pigmented skin. The populations most affected by the condition are women and those over the age of 30. The presentation of the condition is varied. Rosacea can be classified diagnostically into one of two phenotypes, either rosacea with phymatous changes, which includes tissue hypertrophy, rhinophyma, dilated follicles without comedones, and irregular nodular overgrowths, or those limited to centrofacial erythema with periodic flares. Major phenotypes that further define the condition include papules and pustules, flushing, telangiectasia, and ocular manifestations. Secondary phenotypes include dry appearance, burning or stinging, and edema. Diagnosis is made clinically based on the presence of either one diagnostic phenotype or two major phenotypes. The pathophysiology of the condition is not well understood, however, there are exacerbating factors that should be addressed to prevent flares or exacerbation of the condition. These factors include sunlight, alcohol, hot beverages, spicy foods, irritation from topical products, exercise, medications, menopause, and emotional triggers. Rosacea increases the risk of other autoimmune conditions, such as diabetes, celiac disease, Crohn disease, multiple sclerosis, and rheumatoid arthritis. First-line treatment is behavior modification, which includes daily skin care. Gentle skin cleansers, sunscreen, and moisturizers can help in the management of the condition. Second-line treatments include both laser treatment and medical management. Light treatment can be used for erythema and telangiectasia but it is not permanent. Pharmacological treatment is based on the presentation. The most effective treatment for patients with erythema is topical brimonidine. In mild to moderate cases with papules, pustules, and phymatous changes, topical metronidazole or topical ivermectin are first-line treatments. Topical azelaic acid has also been shown to help in the case of rosacea with papules, pustules, and erythema. Oral tetracyclines are first-line in treating moderate to severe rosacea involving papules and pustules when topical treatments fail to adequately treat the condition. The anti-inflammatory properties of the medication are beneficial in all aspects of the condition. In refractory cases, oral isotretinoin can be considered. Topical metronidazole (B) is the first-line treatment for papulopustular rosacea. While it may improve erythema associated with rosacea, it is not the most efficacious in the treatment of erythema. Topical permethrin (C) is an antiparasitic medication that is effective in treating Demodex folliculitis, which is similar in appearance to papulopustular rosacea. Its benefit in treating rosacea is unclear. Topical retinoids (D) can be used in treating papulopustular rosacea but do not appear to be effective in treating erythema associated with rosacea. Additionally, skin irritation is a side effect of this medication.
A 9-year-old boy is diagnosed with scabies. Which of the following patient education is accurate regarding the Sarcoptes scabiei mite? AHypersensitivity reaction to the mite causes the symptomsC BMites may be transmitted by flying from person to person CMites may survive off the body for weeks DSymptoms are caused by bacterial infections transmitted by the mites
Correct Answer ( A ) Explanation: Scabies is an infestation of the skin caused by the mite Sarcoptes scabiei. Scabies occurs across all ages and socioeconomic statuses. Transmission occurs by direct, particularly prolonged, skin-to-skin contact. The risk of scabies is increased in crowded settings, such as in prisons and college dormitories. The lifecycle of the scabies mite is important in understanding the pathogenesis of scabies. The female scabies mite burrows itself into the epidermis and lays eggs in the burrow daily for four to six weeks. Most individuals with clinical manifestations of scabies have 10-15 mites buried within the epidermis. The clinical manifestations are due to a delayed-type (type IV) hypersensitivity reaction to the mite, mite feces, and mite eggs. Symptoms begin within three to six weeks of a primary (first time) infestation and within a couple of days of subsequent infestations. Classic scabies presents with intense pruritus and a characteristic distribution of lesions. Common locations for scabies lesions include the sides and webs of the fingers, wrists, axillae, areolae, waist, genitalia, buttocks, and extensor surface of the knees. More than one area is typically affected. The back is typically free of involvement, and the head is spared except in infants. The pruritus is typically severe and worse at night. The typical scabies lesions are small erythematous papules that are often excoriated. Burrows are a nearly pathognomonic finding, which manifest as thin gray, red, or brown serpiginous lines. However, burrows are not always seen on exam. Multiple household members being affected is another important household clue. Scabies complications may include secondary bacterial infection. The diagnosis of scabies is confirmed by the detection of the scabies mites, eggs, or fecal pellets through microscopic examination, however, patients are often treated after a presumptive diagnosis is made based off history and exam findings. The management of scabies involves eradicating mites from the affected person, managing associated symptoms and complications, and reducing the risk of transmission and reinfestation. Scabies may be treated with topical or oral therapy. First-line agents include topical permethrin or oral ivermectin, however, topical permethrin is preferred in children, pregnant women, and lactating women. Skin lesions resolve within one week following successful treatment, but pruritus may linger for up to four weeks. Close contacts of infested individuals should be treated prophylactically, since symptoms may not occur until four to six weeks after exposure. Pruritus may improve with antihistamines, such as hydroxyzine. Treating environmental measures, such as clothing and linens, should focus on items that were in contact with the infested individuals within the preceding two or three days. These items may be dried on high heat or placed into a sealed bag for two days. Mites may be transmitted by flying from person to person (B) is incorrect because the mites do not fly. Mites may survive off the body for weeks (C) is incorrect. Scabies mites only survive for two or three days off of a human body. Therefore, when educating patients on environmental measures, only items the infested individual came into contact with in the past three days must be treated. Symptoms are caused by bacterial infections transmitted by the mites (D) is incorrect.
A 67-year-old man presents to the clinic with complaints of a new lesion present for two months. There is a 0.8 cm erythematous, tender papule with central hyperkeratosis present on the right lateral forehead. Which of the following physical exam findings differentiates squamous cell carcinoma from actinic keratosis? AAn infiltrated papule BErythema CNew onset DTenderness
Correct Answer ( A ) Explanation: Squamous cell carcinoma is the second most common cutaneous cancer after basal cell carcinoma and accounts for approximately 20% of nonmelanoma skin cancers. The incidence of squamous cell carcinoma increases with age and lower Fitzpatrick skin type. Risk factors for squamous cell carcinoma include ultraviolet light exposure, immunosuppression, chronic inflammation, tobacco use, poor diet, and human papillomavirus infection. The overall prognosis for primary cutaneous squamous cell carcinoma is very good. Recurrent or metastatic lesions result in poorer outcomes. Rates of metastasis of primary squamous cell carcinoma are 2-5% overall. Cutaneous squamous cell carcinoma presents with a variety of clinical manifestations, including papules, plaques, or nodules, and smooth, hyperkeratotic, or ulcerative lesions. Squamous cell carcinoma may appear on any skin surface but is most commonly found on sun-exposed areas. The head and neck, dorsum of the hands and forearms, legs, arms, shoulders, back, and chest are the most common locations for the development of squamous cell carcinoma. Cutaneous squamous cell carcinoma often presents as indurated or firm, hyperkeratotic, and flesh-toned, pink, or red papules, plaques, or nodules. Lesions are usually 0.5-1.5 cm in diameter, although some are much larger. Ulceration may or may not be present, and while lesions are often asymptomatic, they may be painful or pruritic. In the case presented in the vignette above, an infiltrated papule, one that has invaded the deeper dermis resulting in a fullness or palpable density under the skin, is more suspicious for squamous cell carcinoma than an actinic keratosis, which may present as a superficial, scaly macule or papule without infiltration. A skin biopsy is the preferred diagnostic modality of choice and will also assist with staging. Skin biopsies are often performed using local anesthesia and may be performed via shave, punch, or excisional technique. Selection of the appropriate treatment for cutaneous squamous cell carcinoma depends on the classification of the lesion as low or high risk and tumor location. For cutaneous squamous cell carcinoma at low risk for local recurrence, treatment options include surgical excision, including Mohs micrographic surgery, electrodesiccation and curettage, cryosurgery, and radiation therapy. All patients with a history of skin cancer should receive close follow-up for the early recognition and management of treatment-related complications, local or regional recurrences, and the development of new skin cancers. For patients with a recent history of cutaneous squamous cell carcinoma, the current recommended follow-up is every six months for two years after the development of a malignant lesion and then yearly thereafter. Erythema (B) is a common finding in many cutaneous neoplasms, including both squamous cell carcinoma and actinic keratosis. Erythema is a result of increased vascularity and does not necessarily indicate a malignant or precancerous process. A lesion of new onset (C) may be a sign of a precancerous lesion, such as actinic keratosis, or a malignant lesion, such as squamous cell carcinoma. New cutaneous lesions may also be entirely benign, such as a solar lentigo or seborrheic keratosis. Patients commonly complain of tenderness (D) with both actinic keratoses and squamous cell carcinomas.
A 28-year-old woman presents with concerns for skin changes to her hands, and now a new spot on her face. The spots are not itchy or painful. She recently gave birth to her first child and there were no complications with the pregnancy or delivery. She denies any recent illnesses. On exam she has several well-demarcated pale macules between 1-4 cm in diameter on both of her hands and one near her lips, concerning for vitiligo. Which of the following mechanisms is most likely responsible for pathogenesis of this disease? AAutoimmuneAnswer BGenetic CInfectious DNeoplastic
Correct Answer ( A ) Explanation: Vitiligo is the most common cause of dermatologic depigmentation. While the problems with this condition are largely cosmetic, they can also lead to emotional and psychological sequelae due to its disfiguring effects. Vitiligo is characterized by well-defined pale macules that reveal epidermal melanocyte destruction on biopsy. Patients are otherwise asymptomatic without clinical signs of inflammation. The areas of skin most commonly affected include the face, hands, and genitalia, though they can occur anywhere. The size of the lesions varies from a few millimeters to confluent macules completely covering large areas. Most patients develop vitiligo before 30 years of age; otherwise, vitiligo shows no preference for sex, race, ethnicity, or socioeconomic status. While the exact etiology and pathogenesis of vitiligo is unknown, the mechanism with the most supporting data and evidence is autoimmune. Alopecia areata, Graves disease, Hashimoto thyroiditis, pernicious anemia, psoriasis, rheumatoid arthritis, and type 1 diabetes mellitus are among the autoimmune diseases associated with vitiligo. Patients may also attribute episodes of skin trauma or emotional stress such as sunburn or pregnancy to disease onset. There is no direct correlation between autoimmune or any other etiologic explanation with vitiligo, leading to a multifactorial "convergence theory" to explain the diverse phenotypes and mechanisms leading to development of disease. Diagnosis of vitiligo is usually clinical, though a Wood lamp may be employed to confirm clearly demarcated borders in patients with naturally pale skin, and skin biopsy, while usually unnecessary, may be indicated if the diagnosis is questionable. Due to the strong association between vitiligo and autoimmune disease, patients with vitiligo may also be screened for autoimmune thyroid or other autoimmune diseases. There is no cure for vitiligo, but treatments such as topical steroids and topical calcineurin inhibitors like tacrolimus, UV, and laser therapy can slow the progression of the disease and even allow patients to experience some extent of repigmentation. Sunscreen should be used to prevent burns to depigmented areas, and make-up can improve the cosmetic appearance of lesions. Clinicians should bear in mind the stigma surrounding obvious skin lesions like those found in vitiligo and offer psychological support and referrals as needed. Genetic (B) causes may play a role in the development of vitiligo as family clustering of the disease has been noted. However, in twin studies, there has been relatively low concordance, which suggests there are more influential triggers in the pathogenesis of vitiligo. Infectious (C) etiologies, most commonly viral, have also been suggested triggers of developing vitiligo, however, there are insufficient data to support this hypothesis. Neoplastic (D) causes are not thought to contribute to the pathogenesis of vitiligo, however vitiligo-like patches may be found prior to a finding of cutaneous melanoma and are associated with a better prognosis in these melanoma patients.
A 27-year-old man with a history of intravenous drug use presents to the clinic with redness and swelling along the lateral nail fold of his index finger. There is no associated abscess. Which of the following antibiotics is the best treatment for this patient if methicillin-resistant Staphylococcus aureus needs to be covered? AOral cephalexin BOral clindamycin COral dicloxacillin DTopical mupirocin
Correct Answer ( B ) Explanation: Acute paronychia is an inflammatory process that involves the proximal and lateral nail folds and has been present for less than six weeks. It may occur with or without an abscess. The most common causative organisms are skin flora, such as Staphylococcus aureus and Streptococcus pyogenes, however, oral flora, such as Eikenella, may be the causative pathogen in individuals who bite their fingernails. Paronychia occurs when these organisms are inoculated in the periungual area due to mechanical or chemical trauma. The classic presentation is rapid onset of painful erythema and swelling of the proximal and lateral nail folds. There may be an associated superficial abscess. Paronychia may occur on the finger or toenails. Felon is a complication of paronychia where the infection spreads to the pulp space. Paronychia is a clinical diagnosis. The management includes antibiotics and warm soaks. Topical antibiotics, such as mupirocin and triple antibiotic ointment, may be used for mild inflammation. Moderate or severe inflammation often requires oral antibiotics. In most cases, cephalexin or dicloxacillin may be used, however, if methicillin-resistant Staphylococcus aureus is suspected, then clindamycin, doxycycline, or trimethoprim-sulfamethoxazole should be used. Patients with paronychia with an associated abscess require incision and drainage. Adjunctive antibiotics are indicated in immunocompromised patients following incision and drainage, however, in most patients, oral antibiotics are not indicated following incision and drainage. Culture and susceptibility results may be used to guide therapy when available. Oral cephalexin (A) is a first-generation cephalosporin. It provides coverage against methicillin-susceptible Staphylococcus aureus but not methicillin-resistant Staphylococcus aureus. Oral dicloxacillin (C) is an antistaphylococcal penicillin. It provides coverage against methicillin-susceptible Staphylococcus aureus but not methicillin-resistant Staphylococcus aureus. Topical mupirocin (D) is a topical antibiotic and as such wouldn't be appropriate in this patient with complicating risk-factors.
Which of the following is a risk factor for developing a cherry angioma? AGenetic predisposition BMiddle age CPregnancy DSun exposure
Correct Answer ( B ) Explanation: Cherry angiomas are vascular skin lesions that appear in middle age. They are most commonly found on the trunk and are small in size with an average diameter of 0.1-0.4 cm. Patients typically present with a lesion or multiple small lesions that bleed easily with any trauma. The lesion consists of capillaries, and about half of these have oncogenic mutations similar to those found in port wine stains. On physical exam, the lesion is typically nontender and will blanch with pressure. The diagnosis is made clinically, although these must be distinguished from malignant skin lesions such as amelanotic melanoma. Treatment is not always necessary, however, it consists of removal either through excision, electrocautery, or laser therapy. Skin lesions such as seborrheic keratoses have been associated with a genetic predisposition (A), however, there is no genetic association with cherry angiomas. Pregnancy (C) is associated with the development of other skin lesions, such as skin tags or acrochordon, but not associated with the development of cherry angiomas. Sun exposure (D) is associated with skin damage and skin cancers but is not linked to the development of cherry angiomas.
Which of the following treatment options is considered first line in the treatment of mild to moderate atopic dermatitis? ACrisaborole BHydrocortisone CPhototherapy DTacrolimus
Correct Answer ( B ) Explanation: Hydrocortisone and other topical low-potency steroids are considered first-line treatment options for atopic dermatitis. Atopic dermatitis, or eczema, is a chronic skin condition that affects children and adults. The pathogenesis of atopic dermatitis is complex and involves several possible causes, including dysfunctional transepidermal water loss, a reduced epidermal barrier repair process, and increased inflammatory response of the epidermal layer. There is a strong genetic component to this condition and a relationship with other immunoglobulin E (IgE)-moderated responses. Histologically, eczema displays spongiosis, acanthosis, and hyperkeratosis. Patients with eczema complain of dry skin and severe pruritus. There may be associated papules and vesicles. Over time, patients with chronic eczema may demonstrate lichenification in excoriated areas. Most commonly eczematous changes occur over the flexor surfaces of the body. In infants, there may be patches of eczema present on the face or scalp. The severity of the eczema is categorized based on one of several disease severity scales such as the SCORAD index or the Eczema Area and Severity Index (EASI). Treatment is based on severity of the condition. Mild to moderate disease can be treated with topical low-potency steroids and emollient creams, moderate disease can be treated with topical calcineurin inhibitors, and severe disease may require phototherapy or oral immunosuppressive therapies. Crisaborole (A) is a topical phosphodiesterase 4 (PDE4) inhibitor that is used with moderate disease, however, this is not the first-line treatment for mild to moderate disease. Phototherapy (C) is used with severe disease that does not respond to other forms of treatment. Phototherapy with psoralens plus ultraviolet A radiation (PUVA) is a treatment option for those patients. Tacrolimus (D) and other topical calcineurin inhibitors are used in mild to moderate disease, however, these are not typically first-line treatment options.
A 27-year-old woman presents to her provider with a rash over her entire torso, and some on her arms and legs. Her vital signs are normal. She reports mild itchiness that improves with taking diphenhydramine, however, she is anxious about why she has the rash and how to make it go away. She denies any recent changes to her hygiene, cosmetic products, or laundry products. She is healthy overall but did complete a 10-day course of sulfamethoxazole-trimethoprim for a urinary tract infection two days ago. On exam, she has a morbilliform rash and there are no hives, pustules, or petechiae. Her mucous membranes are not involved, and her face, palms, and soles are spared. Based on her history and exam, a reaction to the antibiotic is suspected as the cause of the rash, and she is given reassurance the rash should self-resolve in the next one to two weeks. Which of the following mechanisms is likely responsible for her reaction and rash? AAntigen exposure, causing release of vasoactive substances BAntigen exposure, sensitizing T cells, which then mediate tissue injury CAntigen or hapten binding to antibodies, leading to cell or tissue injury DFormation or deposition of antigen-antibody complexes in vessels or tissue
Correct Answer ( B ) Explanation: The most common type of cutaneous drug reaction is the exanthem. They are most commonly described as morbilliform, maculopapular, and mild in severity, however, they do also include rare but life-threatening reactions like Stevens-Johnson syndrome and toxic epidermal necrolysis. The most common type of exanthem is pruritic, diffuse, and symmetric, occurring five days to two weeks after initiation of an offending drug. However, in patients previously sensitized to a drug, it may begin as early as one to two days after starting a medication. The trunk and proximal extremities are most likely to be involved although the face, palms, and soles may also be affected. Mucous membranes, hair, and nails are not involved except in very severe forms. The majority of drug exanthems are due to type IV hypersensitivity reactions. These are delayed, T cell-mediated immune reactions in which T cells activate macrophages, eosinophils, neutrophils, and monocytes. The exact mechanisms behind which certain drugs elicit specific immune responses are not well understood but are thought to be responsible for the different phenotypes of drug reactions, sometimes described as the four subcategories of type IV reactions (IVa to IVd). In many cases, the drug itself acts as a hapten, a small molecule that stimulates antibody production by fastening to a carrier molecule. Penicillins and cephalosporin drug exanthems are caused by this mechanism. Other drug reactions require an additional step of biotransformation before reactive metabolites are formed; sulfonamides like trimethoprim-sulfamethoxazole is an example. Drugs that have been taken uninterrupted for a prolonged period of time are less likely to cause reactions than those taken previously and then later introduced. Comorbidities like viral infections or immunodeficiency are also thought to predispose individuals to develop drug reactions. Diagnosis of a drug exanthem is usually clinical, and relies heavily on an accurate history, including the chronology of drug exposures, onset of skin lesions, and their subsequent evolution, including other symptoms. For patients with an antibiotic drug reaction, the rash may even occur a few days after the treatment has been completed. In addition to the typical pruritic morbilliform exanthem on presentation, patients may also have a low-grade fever. Laboratory testing and biopsy are not usually indicated, though elevated acute-phase proteins and mild eosinophilia may be observed, and biopsies will reveal an interface dermatitis with scattered dyskeratotic keratinocytes along the dermoepidermal junction. The mainstay of management is discontinuing the offending drug, after which most eruptions will resolve within 7-14 days. Symptomatic treatment may be considered for uncomfortable patients, including topical corticosteroids and systemic antihistamines. Patients presenting with fever (> 38° C), blistering, facial edema, mucositis, or skin tenderness raise concern for more serious hypersensitivity reactions like drug rash with eosinophilia and systemic symptoms (DRESS), Stevens-Johnson syndrome (SJS) and toxic epidermal necrolysis (TEN), which require emergent management. Antigen exposure causes release of vasoactive substances (A) in type I hypersensitivity reactions. Immunoglobulin E is almost always involved and leads to the release of vasoactive amines like histamine, prostaglandins, and leukotrienes from mast cells and basophils. This is the type of response that commonly leads to anaphylaxis, asthma, and urticaria, including urticarial drug reactions. In contrast to type IV hypersensitivity reactions to drugs, type I reactions occur over minutes to hours. Antigen or hapten binding to antibodies, leading to cell or tissue injury (C) occurs in type II hypersensitivity reactions. Autoimmune hemolytic anemia, erythroblastosis fetalis, Goodpasture syndrome, and rheumatic fever are examples of disorders caused by this antigen-mediated cytotoxic process. Formation or deposition of antigen-antibody complexes in vessels or tissue (D) drives type III hypersensitivity reactions. These types of reactions include serum sickness, now most commonly caused by drugs that can be distinguished by the addition of fever, arthralgias, proteinuria, and lymphadenopathy in addition to urticaria. Autoimmune disorders like polyarteritis nodosa, systemic lupus erythematosus (SLE), rheumatoid arthritis, immune complex glomerulonephritis, and arthus reaction are also considered type III hypersensitivity reactions.
A 30-year-old woman presents to her dermatologist. She states she has recently been sunbathing, but her tan is not even. She has noticed some whitish patches under her arms and across her chest. Her only medication is an oral contraceptive. She recalls having a similar reaction to the sun as a child where only her face was affected. Her mother had a similar reaction to sun exposure. Which of the following would be the initial test for this condition? AAntinuclear antibody testing BPotassium hydroxide preparation CPunch biopsy DTzank smear
Correct Answer ( B ) Explanation: Tinea versicolor, also known as pityriasis versicolor, is a fungal skin condition caused by a lipid-dependent fungus from the genus Malassezia, most commonly M. globose, which is part of the normal flora of the skin. The condition presents as either hypopigmented or hyperpigmented plaques, patches, or macules that can be recurrent. These lesions may be erythematous and may have a fine scale. The lesions are generally asymptomatic, but mild pruritus may be present. The hypopigmented lesions are most commonly seen in the summer months and are noticeable as areas of sun exposure fail to tan. The fungus, which damages the melanocytes, produces azelaic acid that causes the failure to tan. Malassezia is seen in areas of sebum production, which explains the change in distribution with age. The distribution in children is often seen on the face, while in adults, the distribution is most commonly the trunk and proximal upper extremities. While the condition is not communicable, risk factors include a positive family history, immunocompromised state, and oral contraceptive therapy. External factors, such as hot weather, humidity, and use of oils, may contribute to the condition. Diagnosis is made clinically and confirmed using a potassium hydroxide preparation. Both yeast and hyphae will be present leading to the classic "spaghetti and meatball" appearance. Examination with Wood lamp may show a yellow to yellow-green fluorescence in some cases. Topical therapy with either selenium sulfide or zinc pyrithione is the first-line treatment. If topical antifungal therapy fails or exacerbations are persistent, the next step is topical azole antifungals, such as ketoconazole. Resistant or severe conditions may warrant the use of oral antifungals, such as fluconazole. Oral antifungals are not used in children. The success of the treatment is based on repeat potassium hydroxide preparation. Resolution of the pigment changes may take months following successful treatment. Patients who experience recurrence of the condition, particularly those who are immunocompromised, can prevent recurrence by preventive use of topical selenium sulfide or ketoconazole. Oral itraconazole can also be used as prevention. Antinuclear antibody testing (A) is used to confirm the diagnosis of cutaneous lupus erythematosus. This condition presents as annular plaques in a photo-distributed arrangement. Punch biopsy (C) would not be the initial test for suspected pityriasis versicolor. If the diagnosis is unclear, a punch biopsy would be considered. Tzank smear (D) along with direct fluorescent antibody preparations, viral cultures, and polymerase chain reaction studies would be used in the diagnosis of cases of herpes simplex virus. Herpes simplex virus is a common trigger of erythema multiforme, which appears as a regularly or irregularly shaped target lesion often in the genital area.
Which of the following historical findings would help distinguish a phototoxicity reaction from polymorphous light eruption? AAge of the patient BAppears hours after sun exposure CRecent doxycycline prescription DSun-exposed areas affected
Correct Answer ( C ) Explanation: A phototoxicity reaction occurs after ultraviolet irradiation of the skin in concurrence with an exposure to a drug that causes sun sensitivity. Doxycycline is a common antibiotic that can cause a phototoxic reaction. These reactions appear similar to a severe sunburn and can be associated with blisters or bullae. This typically begins minutes to hours after sun exposure and can occur in patients of any age. The diagnosis is clinical and based on the history of exposure. The reaction typically subsides once the drug is discontinued. Polymorphous light eruption also occurs minutes to hours after sun exposure on sun-exposed areas of the body (B, D). This can occur in patients of all ages, making the age of the patient (A) irrelevant when differentiating between polymorphous light eruption and a phototoxicity reaction.
A 21-year-old man presents to his dermatologist with the complaint of worsening acne. He was started on oral isotretinoin one week ago to treat his chronic acne as other medications were ineffective. On physical exam, there are multiple large nodules and friable plaques. The lesions have a hemorrhagic crust and there are a few areas of ulceration and erosion. Which of the following would be the mainstay treatment for this severe form of acne vulgaris? AIncrease isotretinoin dosage BOral antibiotics COral glucocorticoids DTopical retinoids
Correct Answer ( C ) Explanation: Acne vulgaris is a multifactorial, dermatological condition that, while it is not disabling, can have negative psychological effects on those affected by the condition. The four factors contributing to the development of acne vulgaris are follicular hyperproliferation and abnormal desquamation, increased sebum production, Cutibacterium (formerly Propionibacterium) acnes proliferation, and inflammation. Hormones can also play a role in triggering outbreaks, especially in women with hyperandrogenism. Treatment recommendations are made based on the clinical presentation and the factors triggering the outbreak. It is important to have the patient practice good skin and sleep hygiene and to avoid harsh chemicals. Acne presentation can be either comedonal, papulopustular, mixed, or nodular. Topical retinoids are considered the first-line treatment for all types of acne and can be used as monotherapy in treating comedonal acne. Topical antibiotics combined with benzoyl peroxide are used for patients with a C. acnes proliferation. They are used concurrently to reduce the risk of antibiotic-resistant bacteria. Oral antibiotics are a more aggressive therapy for treating both inflammation and bacterial colonization. Hormonal therapy can be useful in treating those with hyperandrogenism. Oral isotretinoin is used to decrease sebum production by decreasing the size of the sebaceous glands but is only used in severe, recalcitrant acne due to its side effects and teratogenicity. A severe form of acne vulgaris is acne fulminans. This condition is characterized by nodular acne with friable plaques, hemorrhagic crusts, erosions, and ulcerations. It can be triggered by recent initiation of isotretinoin use or can occur spontaneously. The mainstay of treatment for this condition is glucocorticoids or isotretinoin, providing it is not the trigger of the condition. If isotretinoin is the trigger, it should be discontinued immediately. Glucocorticoid monotherapy should be initiated for four weeks. At that point, either low dose isotretinoin or oral antibiotics can be started and slowly increased while tapering the glucocorticoid over the course of eight weeks as tolerated. Increasing the dosage of isotretinoin (A) would be contraindicated in this patient as the medication should be discontinued. Isotretinoin can be considered a mainstay of treatment for acne fulminans if it is not the trigger for the condition. Oral antibiotics (B) are an effective treatment of inflammatory, resistant acne. They are not considered firstline as there are side effects and antibiotic resistance is a concern. Topical retinoids (D) are first-line in treating most acne conditions and are effective in long-term maintenance of the condition.
A 29-year-old obese woman presents to the office with a new rash. She reports the lesions appeared under her breasts after she started exercising regularly, and they are becoming larger and more pruritic. On physical exam, you find bilateral erythematous, macerated patches of skin on both inframammary spaces. What is the best diagnostic test to confirm the suspected diagnosis? ACutaneous shave biopsy BDiascopy CPotassium hydroxide preparation DWood lamp examination
Correct Answer ( C ) Explanation: Candidiasis is a skin infection caused by candida. It affects all age groups and genders and most commonly presents as a result of the chronic presence of moisture within deep skin folds, such as under the breasts or in the groin. Risk factors for candidiasis include obesity, incontinence, poor hygiene, hyperhidrosis, and immunosuppression. Candidiasis results from inflammation within deep skin folds, which can be caused by humidity, friction, and infection with candida. Irritant and allergic contact dermatitis may also play a role in exacerbating intertriginous candidiasis. The prognosis is very good, and most patients improve without sequelae. Candidiasis may present anywhere on moist or mucocutaneous spaces. It most often presents in areas where skin is in direct contact with skin, such as the groin, axillary area, inframammary area, umbilicus, under the panniculus, and in the neck fold. Patients may complain of burning, tenderness, pruritus, or a malodorous discharge within affected areas. Fissuring and excoriations may also be present. Physical exam often reveals moist, red or reddish-brown, beefy, homogenous patches within skin folds. Erythematous satellite papules and pustules suggest candidal involvement. Candidiasis can be diagnosed based on the physical examination, and candidal involvement can be confirmed with a potassium hydroxide preparation of skin scrapings from a satellite pustule or skin fold. A skin biopsy is not usually indicated and should be reserved for instances where the diagnosis is uncertain. The first-line treatment for candidiasis is application of a topical azole antifungal cream such as topical ketoconazole, clotrimazole, miconazole, econazole. These topical preparations also have additional antibacterial and anti-inflammatory benefits. With appropriate management, candidiasis usually clears rapidly. Secondary infection and cellulitis are potential complications of persistent intertrigo. A cutaneous shave biopsy (A) will identify candidiasis, however, it is not the best option. For the patient in the vignette above, a shave biopsy would present the unnecessary risk of infection or scarring. Diascopy (B) is a diagnostic test that evaluates the blanchability of a suspicious lesion. A clear microscopic slide is applied over the lesion in question and firm pressure is applied. Vascular lesions will blanch with pressure, and nonvascular or hemorrhagic lesions will remain erythematous or pigmented. This test is not useful for diagnosing candidiasis. A Wood lamp examination (D) is a diagnostic test used to identify cutaneous pigmentation disorders. A Wood lamp is a handheld black light used to illuminate areas of the skin. Certain conditions will appear augmented under black light. Tinea corporis, pityriasis versicolor, and vitiligo will appear a brighter white. Erythrasma will appear coral-colored. For the patient presented in the vignette above, if a potassium hydroxide preparation is available, a Wood lamp examination is not the preferred diagnostic test.
A 45-year old woman with a history of atopy presents to the dermatologist due to an intensely pruritic rash on her hands. It has been coming and going every few weeks for the last six months. The pruritis seems to happen all of a sudden and lasts for a week or two, and allergy medicine, lotions, and hydrocortisone that control her eczema seem to have little to no effect on relieving the pruritis or rash. Previously, by the time she made an appointment to be seen, the rash had resolved. She shares pictures on her phone of an erythematous vesicular rash on her palms from the most recent eruption. On physical exam, both of her palms are dry with peeling patches marked with shallow fissures and she does not have any other active lesions. Which of the following is the most likely diagnosis? AAtopic hand dermatitis BDermatophytid reaction CDyshidrotic eczema DHerpetic infection
Correct Answer ( C ) Explanation: Dyshidrotic eczema is also known as acute palmoplantar eczema or pompholyx. The cause of dyshidrotic eczema is unknown, but its development is strongly associated with factors such as a history of atopic dermatitis, exposure to contact allergens or systemic allergens, dermatophyte infection at distant sites, intravenous immune globulin, hyperhidrosis, smoking, and exposure to ultraviolet radiation. This dermatitis primarily and symmetrically affects the palms and soles, including the lateral aspects of the fingers. For the majority of patients, only the hands are involved. Episodes usually begin with acute episodes of intense pruritus followed by a sudden eruption of vesicles, often described as "tapioca" or "sago pudding" lesions. In more severe cases, the vesicles may converge into large bullae, and the symptoms may interfere with daily activities and occupational duties. After this first vesicular stage that may last for weeks, the lesions dry out and resolve with desquamation and peeling. There is usually no clear trigger for flare-ups but they are commonly associated with warm weather, emotions, and physical stress. Recurrence of dyshidrotic eczema is common and may occur for months or years, and as a result chronic hand dermatitis may be seen with lichenified, fissured, and scaling patches or plaques. In severe cases, the nail bed may also be affected. Patients with dyshidrotic eczema are at increased risk of secondary infection, most commonly due to Staphylococcus aureus. Young adults are more likely to be affected, episodes typically decrease with age, and they may completely remit. Diagnosis is primarily made by history and clinical findings. Biopsy is not needed for diagnosis except in cases that respond poorly to treatment or when another diagnosis is under consideration; histological findings for acute eruptions of dyshidrotic eczema include intraepidermal spongiotic vesicles that do not involve the intraepidermal portion of the eccrine sweat duct. The management of dyshidrotic eczema is determined by its severity. In addition to identifying and avoiding triggers and adopting a regular skin care routine to reduce irritation, high potency topical corticosteroids are recommended for mild-to-moderate cases. Severe cases require oral rather than topical corticosteroids in addition to the other general measures. Atopic hand dermatitis (A) is most likely to involve the dorsal aspect of the hands. Patients will also have findings of eczematous lesions or lichenification in joint flexures like the elbows and knees, and report other atopic history like seasonal allergies or asthma in addition to atopic dermatitis. Dermatophytid reaction (B) is a secondary skin eruption that may be pruritic and papulovesicular like dyshidrotic eczema. History and a complete skin examination are usually able to point to a likely primary dermatophyte infection at a distant site like the scalp or the feet, and skin scrapings with a potassium hydroxide (KOH) preparation can confirm the diagnosis. Herpetic infection (D) is more associated with pain than pruritus. The lesions are usually unilateral and present as grouped vesicles on an erythematous base. The diagnosis can be made by sending fluid from deroofed lesions for viral culture, immunofluorescence staining, polymerase chain reaction, or Tzanck smear.
A 25-year-old man with a history of herpes simplex virus presents to his primary care provider with a complaint of a new rash. He states that he had a cold sore about 10 days ago. Three days ago, he noticed a small, itchy blister that is now larger and resembles a target. He states that he has multiple skin lesions and may even have one in his mouth. On physical exam, multiple target-shaped lesions that measure less than 3 cm in diameter are clustered around bilateral elbows and knees. There appears to be a similar lesion on his tongue as well as a resolving lesion on the vermillion border of his left lower lip. A sample of the lesion on his lower lip is positive for herpes simplex virus. Which of the following is the diagnosis of this condition? ABullous pemphigoid BErythema marginatum CErythema multiforme DStevens-Johnson syndrome
Correct Answer ( C ) Explanation: Erythema multiforme is an acute, self-limited, dermatologic condition that may be recurrent. The eruption is thought to be triggered by infection, most commonly the herpes simplex virus, although Mycoplasma pneumoniae infection may also be a trigger. The clinical rash will usually occur two to 17 days following an outbreak of herpes simplex virus. The lesions are cutaneous but may also be seen on mucous membranes. Round, erythematous papules are the initial presentation. The papules evolve into classic target lesions that are made up of three parts. The center of the target is a dusky, blister-like area surrounded by a pale ring, which is, in turn, surrounded by a ring of erythema. These lesions are less than 3 cm in diameter. They are distributed symmetrically to the extensor surfaces of the peripheral extremities. They may also be seen on the trunk, face, palms, and soles. Trauma or sunburn may also cause the lesions to appear in those areas, which is known as the Koebner phenomenon. Mucosal lesions may appear as bullae or erosions. The lesions evolve within three to five days and generally resolve spontaneously within two weeks. While they do not scar, the area may remain hyperpigmented following the eruption. There are less common variations of erythema multiforme. Recurrent erythema multiforme can recur as often as six times per year. The persistent erythema multiforme is less common and is related to viral conditions or irritable bowel syndrome and can persist as long as one year if left untreated. Diagnosis is made clinically based on history and physical exam. Biopsy can be performed should the diagnosis be unclear. As the condition is self-limited and generally asymptomatic, treatment is supportive. Identifying a trigger is important to prevent recurrence. Topical glucocorticoids or oral antihistamines can be used to manage pain or pruritus. Oral antiviral medications can be used for prevention in the care of recurrent erythema multiforme. Second-line therapies are available for patients who have recurrent episodes despite the use of continuous antiviral therapy. Bullous pemphigoid (A) is an autoimmune condition that presents as tense bullae. A target lesion would not commonly be seen in this condition. Erythema marginatum (B) is a sequela to a group A streptococcal infection. The rash presents as nonpruritic, erythematous plaques that migrate. Stevens-Johnson syndrome (D) may appear as an atypical target lesion. Unlike erythema multiforme, the most common trigger for Stevens-Johnson syndrome is a drug reaction.
A 28-year-old man presents to the clinic complaining of a new onset rash that started one week after initiating antiepileptic therapy. He complains of associated fever and joint pain that started with the appearance of his rash. Multiple skin lesions are present on the posterior elbows bilaterally and demonstrate three concentric rings with a dusky, central area surrounded by a pale ring of edema and an erythematous halo. The central maculopapular lesions vary in size from 1 to 3.5 cm in diameter. The patient also has mild mucosal involvement. Which of the following clinical findings indicates a diagnosis of erythema multiforme major versus erythema multiforme minor? AIncreased number of lesions BMucosal involvement CPresence of fever DTarget lesions greater than 3 cm in diameter
Correct Answer ( C ) Explanation: Erythema multiforme is an inflammatory dermatologic disease characterized by target-shaped skin lesions. It is most commonly associated with herpes simplex, Mycoplasma pneumoniae, and upper respiratory tract infections. Other causes include contact allergens, medications (e.g., antiepileptics, nonsteroidal anti-inflammatory drugs, sulfonamides, antibiotics), connective tissue diseases, X-ray therapy, pregnancy, and internal malignancies. While specific pathogenesis of the skin condition is unknown, it is suspected to result from a cytotoxic immune response directed against keratinocytes expressing foreign viral or drug antigens. The rash has a variety of presentations including target lesions, erythematous papules and macules, urticarial-like lesions, vesicles, and bullae. Target lesions initially present as dusky red, round macules and papules with pruritus and burning that evolve to a central erythematous area surrounded by a pale area of edematous skin and a sharp discrete ring of erythema past this pale area. The central maculopapular lesion is generally 1-3 cm in diameter. These lesions have a sudden onset and appear in a symmetric pattern on the palms, soles, backs of the hands and feet, and the extensor aspect of the forearms and legs. Lesions appear in crops and resolve within one to two weeks without scarring. The oral cavity should be examined for bullae and erosions. Erythema multiforme lesions may occur as a result of skin trauma (Koebner phenomenon). Erythema multiforme major is characterized by severe mucosal involvement, often with systemic symptoms (e.g., fever, arthralgias), while erythema multiforme minor presents without or with minimal mucosal involvement and without systemic symptoms. Patients may report symptoms secondary to underlying causes prior to skin manifestations, such as cough and fever with associated pneumonia. Viral culture and direct immunofluorescence can be used to rule out herpes simplex in suspicious vesicles and erosions. Skin biopsy reveals an interface reaction with necrotic keratinocytes and can help to confirm the diagnosis. Treatment is not required for erythema multiforme, which usually resolves within one month. Topical antibiotics can be administered for ruptured blisters and eroded skin to prevent secondary bacterial infection. Systemic corticosteroids (prednisone) can be used for one to three weeks for widespread erythema multiforme. Antiseptic mouthwash promotes healing of oral mucosal lesions. If the rash is associated with herpes simplex infection, oral acyclovir, valacyclovir, or famciclovir can be prescribed as continuous suppressive therapy to reduce the recurrence of disease. Ocular complications of erythema multiforme include keratitis, conjunctival scarring, and visual impairment. Recurrent cases of erythema multiforme that are resistant to systemic antiviral therapy can be treated using azathioprine, mycophenolate mofetil, or dapsone. An increased number of lesions (A) does not differentiate between erythema multiforme major and minor. Lesions most commonly begin at the extensor surfaces of the extremities and spread centripetally to involve the face, neck, palms, soles, flexural surfaces of the extremities, and the trunk. Mucosal involvement (B) is more common in erythema multiforme major, but limited mucosal involvement can be present in erythema multiforme minor. The patient in the vignette above has only mild mucosal involvement, which does not by itself classify the case as major. While most lesions of erythema multiforme are between 1-3 cm in diameter, target lesions greater than 3 cm in diameter (D) do not classify the rash as either major or minor.
A 10-year-old boy presents to a pediatric clinic for a rash consisting of dusky red macules surrounded by pale halos on the extensor surfaces of the arms and legs. His parents report he was recently treated with azithromycin for a fever with nonproductive cough. His vaccinations were updated in the last two months prior to presentation. Which of the following risk factors is the cause of this patient's condition given the most likely diagnosis? AAntibiotic therapy BInfection with herpes simplex virus CInfection with Mycoplasma pneumoniae DRecent vaccination
Correct Answer ( C ) Explanation: Erythema multiforme presents acutely with characteristic target-shaped skin lesions and is often confused with Stevens-Johnson syndrome. Stevens-Johnson syndrome has a similar cutaneous and mucosal distribution but is much more severe and often drug induced. Typically, erythema multiforme affects young adults between the ages of 20 and 40 years but can also occur in the pediatric population. Studies done with herpes simplex virus have shown that a key point in the pathway of clinical presentation is CD34+ cells containing the virus traveling to the epidermis, where they deposit DNA fragments into epidermal keratinocytes. This later results in the lysis of these infected keratinocytes via T cells. Infections, notably those with herpes simplex virus, are by far the most common cause of this condition. Infection with Mycoplasma pneumoniae is also a risk factor especially in children. Other risk factors include medications, malignancy, autoimmune conditions, immunizations, radiation exposure, sarcoidosis, and menstruation. Generally, erythema multiforme can be attributed to medications if it is an acute presentation after medication initiation with known culprits, but this represents only 10% of cases. Such medications include nonsteroidal anti-inflammatory drugs, sulfonamides, antiepileptics, and antibiotics. There are two classifications of erythema multiforme: minor and major. The clinical presentation often includes erosions or bullae of mucosa in the genital region, oral cavity, and eyes, but it can present just with red, dusky macular skin lesions. Erythema multiforme minor is without mucosal involvement or only mild mucosal involvement, while major presents with significant mucosal involvement. Systemic symptoms such as cough and fever may be found in cases associated with underlying Mycoplasma pneumoniae infection. The diagnosis is clinical, but skin biopsies can be obtained to aid the diagnosis. Biopsy findings include basal cell vacuolar degeneration, scattered necrotic keratinocytes, and lymphocyte exocytosis. Laboratory studies are not specific and include elevated ESR, liver enzymes, and white blood cell count. Treatment is usually symptomatic relief and consists of removing the offending agent if medication related. Treatment with antivirals in cases caused by herpes simplex have been shown to be ineffective. Other causes such as infection should be managed with appropriate antibiotics. For mild disease, topical steroids can be used. Severe skin and mucosal involvement that leads to debilitation or severe pain can be treated with prednisone. Ocular involvement necessitates immediate ophthalmologic consultation, and topical dexamethasone may be used as per judgment of opthamologist. The eruptions should clear within 2-17 days after presentation and most often do not result in any other complications. Recurrent erythema multiforme, which is six episodes or more per year, may occur in a small subset of patients but is exceedingly rare, given that the annual incidence of erythema multiforme is less than one percent. Antibiotic therapy (A) has been implicated in the development of erythema multiforme but is only responsible for 10% of cases. Infection with herpes simplex virus (B) is the most common cause of erythema multiforme overall. However, the patient in this vignette has a history of Mycoplasma pneumoniae, as evidenced by a recent history of fever and nonproductive cough. Therefore, Mycoplasma pneumoniae is the most likely cause in this patient. Recent vaccination (D) is also a possible factor but is quite rare.
Which of the following is a risk factor for developing erythema nodosum? ACutaneous polyarteritis nodosa BHuman immunodeficiency virus CStreptococcal infection DTick bite
Correct Answer ( C ) Explanation: Erythema nodosum is secondary to triggers that cause a delayed hypersensitivity reaction. The reaction is immune-mediated but the exact pathogenesis is not well understood. Triggers can be infection, drugs, or inflammatory conditions, however, most cases of erythema nodosum are idiopathic and have no clear cause. Women are affected more often than men, but otherwise erythema nodosum can occur in all ages and races. Patients present with tender, immobile, and erythematous nodular lesions on the bilateral anterior lower extremities, which are approximately 2-5 cm in diameter and may follow a prodrome of fever and malaise. Diagnosis is typically clinical, although biopsy with histologic examination can confirm the diagnosis. Further testing through lab work could be done to rule out underlying causes. Symptoms usually resolve without sequelae within eight weeks of the first presentation. Supportive treatment includes leg elevation and compression. Nonsteroidal anti-inflammatory medications may be used for analgesia. Systemic glucocorticoids can be used in severe cases. Cutaneous polyarteritis nodosa (A) is characterized by painful subcutaneous nodules on the on the lower extremities and can be complicated by livedo racemosa, necrosis, and ulceration. Human immunodeficiency virus (HIV) (B) can be associated with erythema multiforme but not erythema nodosum. Erythema multiforme is characterized by multiple small target-like lesions. There are many etiologies associated with erythema multiforme including drugs and infections. A tick bite (D) can be associated with erythema migrans in the form of a "bullseye" rash related to the transmission of Lyme disease.
A 60-year-old woman presents to the clinic with a new mole on her back. The patient reports the lesion has been growing. Physical exam reveals irregular borders and variegated color. Biopsy confirms the diagnosis of melanoma and shows that the lesion is 1.5 millimeters thick. Which of the following in the next best step? ACryotherapy BExcision with narrow margins CExcision with wide margins DRadiation therapy
Correct Answer ( C ) Explanation: Melanoma is the most serious form of skin cancer and is the sixth most common cancer. Risk factors include personal or family history of melanoma, high sun exposure (history of sunburns), fair skin, having more than 50 common nevi, and having clinically atypical nevi. Most melanomas begin as superficial tumors confined to the epidermis and remain this way for several years. This stage is known as the horizontal growth phase, and melanomas in this stage are nearly always curable by surgical resection. Melanomas that infiltrate into the dermis are in a vertical growth phase and have metastatic potential. Two-thirds of melanomas arise de novo, while one-third arise from a pre-existing nevus. Clinical predictors of possible melanoma include asymmetry, irregular borders, variegated color, diameter ≥ 6 millimeters, and a recent change in or development of a new lesion. These clinical predictors are referred to as the ABCDE mnemonic. In addition, a skin lesion that looks different from surrounding lesions is called an "ugly duckling" sign and should be considered suspicious. There are four major subtypes of cutaneous melanoma: superficial spreading (most common), nodular melanoma, lentigo maligna, and acral lentiginous. Superficial spreading melanoma accounts for 70% of all melanomas. The classic presentation is a variably pigmented macule or thin plaque with an irregular border. The size ranges from a few millimeters to several centimeters in diameter. Most of these tumors are diagnosed while they are thin (less than 1 millimeter) and highly curable. Superficial spreading melanoma can arise from any location but has a predilection for the back in men and women and the lower extremities in women. The ABCDE mnemonic is most applicable to superficial spreading melanomas and does not apply as well to the other subtypes. Nodular melanomas are the second most common type of melanoma accounting for 15-30% of melanomas. Nodular melanomas may present as darkly pigmented, pedunculated, or polypoid papules or nodules. However, nodular melanomas are frequently amelanotic or pink with symmetric borders and a relatively small diameter. Most nodular melanomas are more than 2 millimeters thick at the time of diagnosis. Melanomas may also occur within the nail unit. These melanomas are called subungual melanomas and usually present as a longitudinal brown or black nail plate band with or without nail dystrophy. Primary care clinicians who identify a skin lesion that is not clearly benign should have a relatively low threshold for referral to a dermatologist for dermoscopic examination and possible biopsy. Changes in a long-standing mole or a new persistent skin lesion (especially if growing and pigmented) are the most important criteria for referral. Other indications include any mole with three or more colors, loss of symmetry, and that is itching or bleeding, or a new pigmented band in a nail (especially if it is associated with nail plate damage). For lesions that are considered borderline, it is often helpful to obtain dermoscopic imaging and then have the patient follow-up for repeat imaging in three months. Dermoscopy examination should be performed on all suspicious pigmented lesions. This technique is commonly used in dermatologic settings and requires training. Dermoscopy improves both the sensitivity and specificity of the clinical diagnosis of melanoma when performed by trained clinicians. Furthermore, it improves confidence in the diagnosis of benign lesions and reduces the number of unnecessary biopsies. Melanoma is definitively diagnosed by histopathologic examination of a biopsy. Complete full-thickness excisional biopsy of suspicious lesions with 1- to 3-millimeter margins of normal skin including subcutaneous fat is recommended. Shave biopsies should not be performed if melanoma is in the differential. Once the diagnosis of melanoma is confirmed, surgical excision with an adequate margin of normal tissue is required. The recommended width of surrounding local normal tissue varies based on the thickness of the primary tumor. The recommended margin of normal tissue for melanomas ≤ 1 millimeter thick is 1 centimeter. The recommended margin is 1 or 2 centimeters for melanomas that are between 1 and 2 millimeters in thickness. Melanomas that are more than 2 millimeters in vertical thickness should have resection with a 2-centimeter margin of normal tissue. Lymph node biopsy is recommended for melanomas with a thickness ≥ 0.8 millimeters and in those with melanomas with a thickness < 0.8 millimeters if ulceration is present. Sometimes anatomy precludes an adequate resection margin. In these cases, adjuvant radiation is recommended postoperatively. Melanomas in unusual sites, such as subungual, plantar or palmar sites, or on mucous membranes are generally associated with a poorer prognosis than cutaneous melanomas. The same principles of wide local excision and radiation therapy apply to melanomas in unusual sites. Tumor thickness (vertical depth) is the single most important prognostic factor for patients with localized melanoma. The 10-year survival rate is 92% for patients with melanomas ≤ 1 thick and declines to 50% for patients with tumors more than 4 millimeters thick. Cryotherapy (A) is the treatment for benign skin lesions, such as cutaneous warts. It should not be used to treat melanoma. Excision with narrow margins (B) is incorrect. Narrow margin excision is associated with increased incidence of local recurrence. Radiation therapy (D) is used adjunctively in cases where adequate excision margins are not possible either due to anatomic location or disease extension.
A 35-year-old woman presents to the clinic with raised lesions on her elbows and knees that are erythematous, well-defined, and symmetric. The lesions have an overlying silvery scale. She states that she has a family history of similar lesions. Which of the following is the most likely diagnosis? AErythrodermic psoriasis BGuttate psoriasis CPlaque psoriasis DPustular psoriasis
Correct Answer ( C ) Explanation: Psoriasis is a common chronic inflammatory skin disease with a prevalence of 2-6%. There are various types of psoriasis, including chronic plaque psoriasis, guttate psoriasis, pustular psoriasis, erythrodermic psoriasis, inverse psoriasis, and nail psoriasis. Psoriasis is an immune-mediated condition. It has a bimodal peak regarding the average age of onset with one peak between 30 and 39 years of age and the second peak between 50 and 69 years of age. Risk factors include genetic predisposition (family history), smoking, obesity, and certain medications (beta-blockers and lithium). Chronic plaque psoriasis is the most common variant of psoriasis. It classically manifests with symmetric and well-defined erythematous plaques with an overlying silvery scale. Common locations include the scalp, extensor elbows, knees, gluteal cleft, ears, and umbilicus. The plaques are usually either asymptomatic or pruritic. The extent of involvement can range from localized disease to involvement of most of the body surface area. There is typically a thick and silvery scale present, however, recent bathing temporarily removes this scale. Inverse psoriasis refers to presentation of psoriasis in intertriginous areas, such as the axilla, inguinal area, intergluteal area, or inframammary region. Nail psoriasis is often noticed after cutaneous psoriasis but, in some cases, it may precede cutaneous psoriasis. In unclear cases, the presence of nail psoriasis can be an important finding to support the diagnosis of psoriasis. Nail psoriasis is particularly common in individuals with psoriatic arthritis, which may be due to the proximity of the nail matrix and the distal interphalangeal joint. Nail psoriasis often manifests with nail pitting scattered across the nail plate. Other findings may include leukonychia (white discoloration of the nail plate), spots on the lunula, and crumbling of the nail plate. Psoriasis is typically more severe in human immunodeficiency virus-infected patients manifesting with palmar and plantar surface involvement, nail disease, arthritis, and widespread cutaneous involvement. Psoriatic arthritis is a seronegative (rheumatoid factor negative) spondyloarthropathy that occurs in 7-48% of individuals with psoriasis. Symptoms may include joint pain, joint stiffness (especially in the morning), and back pain. Psoriatic arthritis may be symmetric or asymmetric and often involves the distal interphalangeal joints. There are multiple comorbidities associated with psoriasis. These include obesity, metabolic syndrome, hypertension, diabetes mellitus, atherosclerosis, hepatic disorder, and pulmonary disorders. It is unknown why these disorders occur more often in individuals with psoriasis. Stress is often an inciting or exacerbating factor for psoriasis. The diagnosis of psoriasis can be made clinically by history and physical examination in most cases. Classic, well-demarcated and symmetric plaques in typical locations with a coarse, silvery scale suggest a diagnosis of plaque psoriasis. Family history, nail involvement, and psoriatic arthritis are also supportive findings. Skin biopsy is occasionally performed in unclear cases. The preferred biopsy is a 4-millimeter punch biopsy from involved skin. The clinical course of psoriasis is not predictable. Plaque psoriasis is typically a chronic disease. The management of psoriasis varies depending on the type of psoriasis. Patients with limited plaque psoriasis are initially treated with topical corticosteroids, such as clobetasol propionate, and emollients. Topical tacrolimus or pimecrolimus may be used in facial or intertriginous areas as corticosteroid-sparing agents. Patients with moderate or severe plaque psoriasis should initially be treated with phototherapy in addition to topical corticosteroids. Patients who fail phototherapy should be treated with systemic agents, such as methotrexate, cyclosporine, or biologic agents (adalimumab, etanercept, or infliximab). A dermatologist should manage patients requiring systemic therapy for plaque psoriasis. Erythrodermic psoriasis (A) is an uncommon manifestation that may be acute or chronic. It is characterized by generalized erythema and scaling from head to toe. Complications of erythrodermic psoriasis include infection and electrolyte abnormalities. Patients with erythrodermic psoriasis are often admitted and require dermatology consultation. Guttate psoriasis (B) manifests with multiple small papules and plaques that are usually less than 1 centimeter in diameter. It is closely related to recent infections, particularly streptococcal pharyngitis. Guttate psoriasis typically occurs as an acute eruption in a child or young adult with no previous history of psoriasis. The trunk and proximal extremities are most commonly affected, and the lesions have the characteristic silvery scale seen in plaque psoriasis. Pustular psoriasis (D) is a form of psoriasis that can have life-threatening manifestations. It presents with acute onset of widespread and painful erythematous plaques and pustules along with systemic symptoms, including fever and malaise.
A 20-year-old woman presents to the clinic with erythematous pruritic papules. The papules are located on the webbed space of the fingers, wrists, and axilla. There are linear burrows between some of the papules, and she states that some of her family members have similar symptoms. What is the most likely diagnosis? AAllergic contact dermatitis BAtopic dermatitis CScabies DUrticaria
Correct Answer ( C ) Explanation: Scabies is an infestation of the skin by the mite Sarcoptes scabiei that results in an intensely pruritic eruption with a characteristic distribution. The scabies mite buries itself under the skin and leaves behind eggs and feces in the epidermal tunnels it creates. All demographics are affected, but scabies is more common in women and children. Risk factors include poverty; overcrowding; contact with hospitals, prisons, or refugee camps; and immunocompromised status. In most cases, transmission occurs from direct contact with an infected individual. Transmission commonly occurs from parents to children, especially from mother to infant. However, the mode of contact can be via sexual contact. Furthermore, scabies can be contracted by contact with contaminated clothing or sheets, although this is thought to be uncommon. The classic presentation of scabies is an intensely pruritic eruption with small (often excoriated) erythematous papules that frequently occur on the fingers (sides of the fingers and webbed spaces are common), wrists, axilla, areolae, waist, genitalia (scrotum, penile shaft, and glans), and buttocks. The back and head are often relatively free of involvement. The pruritus is due to a type IV hypersensitivity reaction to the mite, mite feces, and mite eggs. The itching is often worse at night, and it is common for multiple members of the household to be affected. Linear burrows are a pathognomonic finding but are often not evident on physical examination. The onset of symptoms from scabies may be delayed for several weeks after infestation. Therefore, individuals who have been in close personal contact with a patient with classic scabies may have active scabies even in the absence of symptoms. Complications of infestation include secondary staphylococcal infections, such as impetigo, cellulitis, and paronychia. The diagnosis is often suspected clinically. Microscopic examination can detect the scabies mite, eggs, or fecal pellets to confirm diagnosis. The important components to a clinical diagnosis of scabies include the itch, distribution of the lesions, and presence of other family members who are affected. Furthermore, the pathognomonic burrows are important when present. Treatment of scabies is recommended for symptom relief and to prevent transmission. Scabies can be treated with either topical or oral therapy. First-line agents include topical permethrin and oral ivermectin. Additional topical treatment options for scabies include benzyl benzoate, lindane, crotamiton, and malathion. Signs of successful treatment include resolution of active skin lesions and nocturnal pruritus one week after treatment. Nodules that persist after eradication can be treated with potent topical corticosteroids or intralesional corticosteroid injection. Treatment of individuals (even those who are asymptomatic) who have had prolonged skin-to-skin contact with patients with classic scabies is recommended. Crusted scabies is treated with a combination of topical permethrin and oral ivermectin. It is also recommended to clean things that the infested individual came in contact with within the preceding 48 to 72 hours. This is because scabies mites generally do not survive for more than this time period off of a human body. Environmental cleaning may include washing clothes and bedding or putting these items into a sealed bag for 48 to 72 hours. Allergic contact dermatitis (A) presents with erythematous and indurated scaly plaques. Vesiculation and bullae may be seen in severe cases. However, the linear burrows described in the vignette are pathognomonic for scabies. Atopic dermatitis (B) presents as an erythematous and pruritic rash. Dry skin and severe pruritus are the most common findings. The linear burrows and classic location for scabies described in the vignette support scabies as the most likely diagnosis. Urticaria (D) present as circumscribed, raised, and erythematous plaques with central pallor. The lesions often itch. The linear burrows and classic location for scabies described in the vignette support scabies as the most likely diagnosis.
A 20-year-old, breastfeeding woman presents to the clinic with an intensely pruritic, erythematous rash. The lesions are on the sides of her fingers, flexor surface of her wrists, axillary folds, waist, and on her lower buttocks. Several other household members are experiencing similar symptoms. On exam, linear burrows are seen between some of the lesions. What is the treatment of choice for this patient? ABenzyl benzoate BOral ivermectin CTopical permethrin DTopical sulfur
Correct Answer ( C ) Explanation: Scabies is an infestation of the skin caused by the mite Sarcoptes scabiei. It is a relatively common infestation that can occur in individuals of any age or socioeconomic status. Crowded settings, such as college dormitories, long-term care facilities, and prisons, increase the risk of scabies. Transmission of scabies usually occurs through direct and prolonged skin-to-skin contact, which may occur among family members or sexual partners. The life cycle of the scabies mite consists of the female mite burrowing into the epidermis and laying eggs in the burrow daily for four to six weeks. Most individuals with an initial episode of scabies have 10-15 mites buried within their epidermis. The clinical variants of scabies include classic scabies and crusted scabies. Scabies is marked by intense pruritus and a characteristic distribution of the lesions. Classic locations for scabies lesions include the sides and webs of the fingers, wrists, axillae, areolae, and genitalia. The pruritus is often intense and may be worse at night. The pruritus is caused by a delayed-type hypersensitivity reaction to the mite, mite feces, and mite eggs. Symptoms begin within three to six weeks of primary infestation or one to three days after subsequent infestations. The typical scabies lesions are small, erythematous papules that are often excoriated. Burrows manifest as 2-15-millimeter, thin, gray, red, or brown serpiginous lines. However, burrows are often not visible. Scabies typically involves more than one area, and common areas of scabies infection include the sides and webs of the fingers, flexor aspects of the wrists, extensor aspects of the elbows, anterior and posterior axillary folds, periareolar skin, periumbilical skin, waist, male genitalia, extensor surface of the knees, lower buttocks and adjacent thighs, and lateral and posterior aspects of the feet. The back is relatively free of involvement and the head is spared except in young children. Crusted scabies typically occurs in individuals with immunocompromising conditions, such as acquired immunodeficiency syndrome, and it typically presents with erythematous patches with prominent scale. Complications of scabies may include secondary staphylococcal or streptococcal infections, such as impetigo. The diagnosis of scabies is confirmed by the detection of the scabies mites, eggs, or fecal pellets through microscopic examination. However, a presumptive diagnosis can be made based on classic history and examination findings. Clinical clues include widespread itching that is worse at night, spares the head (except in infants and young children), occurs in characteristic distributions, and affects multiple household members. The itching may seem out of proportion to visible changes in the skin. Burrows strongly support the diagnosis when they are present. Common differential diagnoses include atopic dermatitis, contact dermatitis, nummular eczema, and arthropod bites. The management of scabies involves eradicating mites from the affected person, managing associated symptoms and complications, and reducing the risk of transmission and reinfestation. Classic scabies can be treated with either topical or oral therapy. Topical permethrin or oral ivermectin are considered first-line treatment for adults. Topical permethrin is preferred in children and pregnant or lactating women. However, oral ivermectin can be used in children who weigh more than 15 kilograms. Crusted scabies should be treated with a combination of topical permethrin and oral ivermectin. Resolution of skin lesions and nocturnal pruritus one week after treatment suggests that treatment was successful. However, pruritus may persist for up to four weeks. Nodules that persist after eradication of mites can be treated with potent topical corticosteroids or intralesional corticosteroid injections. Contacts of individuals with scabies who have had prolonged skin-to-skin contact should be treated prophylactically, because these individuals may be infested but not yet have symptoms. Environmental measures should also be taken to reduce the risk of transmission. The scabies mites cannot survive for more than two to three days away from human skin. Therefore, environmental measures are targeted at items and areas that were in contact with the infested individual in the preceding two or three days. These may include washing linens, clothing, and bedding on high heat, vacuuming all furniture, and placing items that cannot be washed into a sealed bag for two days. Pruritus may improve with antihistamines. A nonsedating antihistamine, such as cetirizine, can often be taken during the day, and a sedating antihistamine, such as diphenhydramine, can be taken at night. In addition, individuals with itching that persists after mite eradication can also be treated with topical corticosteroids, such as triamcinolone acetonide, or an oral glucocorticoid taper, such as prednisone. Benzyl benzoate (A) is considered a second-line treatment for pregnant women. Oral ivermectin (B) is first-line treatment in nonpregnant or lactating patients, however, data evaluating the risk in pregnant or lactating women is limited. Therefore, it is recommended to use topical permethrin in the treatment of scabies in pregnant or lactating women. Topical sulfur (D) is considered a second-line treatment for pregnant or lactating women.
A 20-year-old man presents to the clinic with an erythematous greasy-appearing rash on his central face. Which of the following is the best treatment for the suspected diagnosis? AKetoconazole shampoo BTopical clobetasol propionate ointment CTopical hydrocortisoneCorrect Answer DTopical triamcinolone ointment
Correct Answer ( C ) Explanation: Seborrheic dermatitis is a chronic and relapsing form of dermatitis. The prevalence of clinically significant seborrheic dermatitis is about 3%, but there are many more individuals with minor cases. The pathogenesis is incompletely understood, but sebaceous glands seem to play a role, as seborrheic dermatitis occurs most often in areas with a high concentration of sebaceous glands. One theory is that areas dense with sebaceous glands produce an environment where Malassezia yeast can grow, which causes the inflammatory reaction that manifests as seborrheic dermatitis. Seborrheic dermatitis presents as pinkish erythematous patches or plaques with a greasy appearance. There may be white-yellow scales. The most common areas of involvement include the scalp, external ears, center of the face, upper trunk, and intertriginous areas. Psoriasis is usually better demarcated than seborrheic dermatitis. Seborrheic dermatitis may cause itching. The typical clinical course is chronic with relapses. Seborrheic dermatitis is usually a clinical diagnosis. The management of seborrheic dermatitis requires repeat treatment and sometimes maintenance treatment due to the chronic and relapsing nature of the condition. Topical corticosteroids and topical antifungals are each used in the treatment. Topical ketoconazole or other azole antifungal agents work by decreasing the concentration of Malassezia and through their anti-inflammatory effect. Topical corticosteroids are often used due to their ability to reduce inflammation. Low-potency agents, such as hydrocortisone, should be used on the face. Patients with recurrent seborrheic dermatitis may require intermittent once-weekly treatment during asymptomatic periods to prevent relapse. Ketoconazole shampoo (A) is used in the treatment of seborrheic dermatitis involving facial hair or the scalp. The shampoo is typically used daily until remission and then once a week to prevent relapses. It may be used in combination with topical corticosteroids. Individuals with seborrheic dermatitis on the face may be treated with ketoconazole cream. Topical clobetasol propionate ointment (B) is a super-high-potency topical corticosteroid, therefore, it cannot be used on the face. Topical triamcinolone ointment (D) is a medium-high-potency topical corticosteroid, which means that it also cannot be used on the face.
A 50-year-old woman presents to the clinic complaining of a flaky, nonpruritic, hyperpigmented lesion on her upper arm. She states it has been present for six months, appears to be slowly growing, and occasionally partially flakes off then redevelops. Physical exam reveals a 1 cm, oval, hyperpigmented papule with a scaly, variegated surface and no surrounding erythema or edema. Which of the following is the most likely diagnosis? AActinic keratosis BBasal cell carcinoma CSeborrheic keratosis DSolar lentigo
Correct Answer ( C ) Explanation: Seborrheic keratosis is a skin lesion caused by the proliferation of immature keratinocytes that increases in incidence with age. The etiology is uncertain, although there appears to be a genetic predisposition in some individuals. The appearance of seborrheic keratosis is verrucous or scaly, flesh-colored or hyperpigmented, and papular or plaque-like. Often, the most superficial layers of the lesion are flaky and easily removable. Under dermoscopy, seborrheic keratoses demonstrate ridges and furrows and can be described as cerebriform. The borders are well-demarcated. Seborrheic keratoses are not malignant, slow growing, and only need to be removed if they are cosmetically bothersome. Cryotherapy can be used to remove thin lesions. Thicker lesions may require electrodesiccation or surgical excision with shave biopsy or curettage. The sudden appearance of multiple seborrheic keratoses along with skin tags and acanthosis nigricans can signify an underlying gastrointestinal or pulmonary malignancy. Actinic keratosis (A) is also a skin lesion resulting from an abnormal proliferation of keratinocytes that increases in incidence with age. However, these lesions are premalignant and have the potential to become cutaneous squamous cell carcinomas. Actinic keratoses develop on sun-exposed areas of the body and tend to have a more erythematous appearance. Their surface is often scaly and not easily removed. Basal cell carcinoma (B) appears as an erythematous, pearly nodule with superficial telangiectasias. The lesion is slow growing, more common in older individuals, and develops on sun-exposed areas of the skin. Some mature basal cell carcinomas can have central ulcerations. They are malignant lesions that require removal. Solar lentigo (D) is a flat, hyperpigmented lesion that develops after exposure to ultraviolet radiation. Solar lentigo is more common in fair-skinned, older individuals. The lesions are macular, not papular or pustular.
A 45-year-old woman with a history of active leukemia presents to her primary care provider following treatment for a urinary tract infection. She states she has been using sulfamethoxazole trimethoprim as prescribed for the past seven days. One day ago, she had a fever of 102.2°F and noticed that she was very sensitive to light and her mouth was burning, making it difficult to swallow. Today, she noticed an itchy rash developing on her face and what looks like blisters. On physical exam, her face has symmetrical, erythematous macules with ill-defined borders and bullae that spread when pressure is applied laterally to the lesion. The tissue sloughs with gentle pressure. On oral examination, there are hemorrhagic erosions that are covered with a greyish membrane. Which of the following is the most likely diagnosis? AChikungunya fever BErythema multiforme CStevens-Johnson syndrome DUrticaria multiforme
Correct Answer ( C ) Explanation: Stevens-Johnson syndrome and toxic epidermal necrolysis are a continuum of related mucocutaneous reactions whose diagnoses are dependent on the amount of body surface area that is affected by the condition. Stevens-Johnson syndrome affects less than 10% of the body surface area, and toxic epidermal necrolysis affects more than 30% with an overlap of the two conditions between 10-30%. These conditions are most commonly triggered by medications, and the most common triggers are allopurinol, sulfonamides, certain antiepileptic drugs, lamotrigine, and some nonsteroidal anti-inflammatory drugs. Malignancy, human immunodeficiency virus, and the presence of certain genetic markers increase the risk of this type of drug reaction. The drug reaction causes keratinocyte necrosis that leads to either partial or full thickness necrosis, which leads to the formation of subepidermal bullae. The reaction to the drug will occur within the first four days to the first four months of continuous use of the medication. A prodrome of fever and influenza-like symptoms lasting one to three days is followed by the development of cutaneous lesions and mucosal lesions, which affect oral, ocular, pharyngeal, esophageal, and urogenital mucosa. Mucosal lesions can cause photophobia, difficulty swallowing, stomatitis, mucositis, pharyngitis, and urethritis. Cutaneous lesions begin as erythematous macules with purpuric centers on the face and thorax. Scalp, palms, and soles are generally spared. These lesions evolve into bullae and vesicles that slough within a few days. Nikolsky sign, sloughing of uninvolved tissue due to pressure lateral to the lesion, and bullae spread sign, which is bullae extension due to pressure on the bullae, are common positive findings. Diagnosis is based on clinical findings. Skin biopsy is used to confirm diagnosis. Severity and prognosis are determined by estimating the body surface area that is involved. Because of the high risk for infection, bacterial and fungal cultures should be performed and repeated. Chest radiographs should be performed as there is a high risk of developing pneumonia and interstitial pneumonitis. Acute complications include hypovolemic shock, renal failure, bacteremia, and multi-organ dysfunction. Discontinuing the trigger medication is the first step in treatment. The patient should also be admitted to a burn unit for wound care, pain control, and supportive care. It is important for patients to avoid the trigger medication in the future to prevent recurrence. Complications should be treated as they arise. Chikungunya fever (A) presents atypically as a vesiculobullous eruption with superficial erosions in children and infants. It is a cutaneous condition that spares the mucosa. Erythema multiforme (B) is a target-shaped lesion that is linked to an infection of herpes simplex virus and has no epidermal detachment. Urticaria multiforme (D), also known as acute annular urticaria, appears as pruritic lesions on the face and trunk. The lesions are annular, erythematous plaques with either a central clearing or dusky blue center. The condition resolves within 24 hours.
Which of the following is a risk factor for melasma? AAdvanced age BLighter skin tones CMale sex DPregnancy
Correct Answer ( D ) Explanation: Melasma is an acquired hyperpigmentation of the skin that typically affects the sun-exposed areas of the face. Risk factors include female sex, reproductive age, darker skin tones, pregnancy, other hormonal exposure (oral contraceptive use or hormone-replacement therapy), ultraviolet radiation, and genetic predisposition. Melasma classically presents as irregularly-shaped hyperpigmented macules on the face. The color change is hyperpigmented but varies from light brown to dark brown. The hyperpigmentation is often symmetrically distributed. The diagnosis is made clinically based on history and clinical presentation. Drug-induced hyperpigmentation and postinflammatory hyperpigmentation should also be considered as possible etiologies of hyperpigmented skin on the face. The treatment of melasma is challenging because of its chronic and relapsing nature. Treatments include skin-lightening agents, chemical peels, and laser- and light-based therapy. Because no single therapy has proven to be beneficial for all patients with melasma, combinations of agents or modalities are often used, especially in recalcitrant cases. Patients should be informed that skin-lightening agents may require continued application for up to six months to work. The first-line treatment for patients with melasma may be a triple-combination cream containing hydroquinone 4%, tretinoin 0.05%, and fluocinolone acetonide 0.01%. The cream should be applied nightly for eight to 24 weeks and up to 12 months. Chemical peels or laser- and light-based therapies can be used in refractory cases. However, the risk of chemical peels or laser- and light-based therapies includes postinflammatory hyperpigmentation. Sun protection is an important aspect of the treatment and prevention of recurrence. Melasma is a chronic disease with common recurrence. In addition, intermittent application of single agents or triple combination creams can also be helpful in preventing recurrence. Melasma due to pregnancy typically improves within one year after delivery but may never completely resolve. Advanced age (A), lighter skin tones (B), and male sex (C) are not considered risk factors for melasma. In contrast, reproductive age, darker skin tones, and female sex are risk factors.
A 42-year-old, right-handed woman presents to your office with three days of pain and swelling of her right middle finger. On physical examination, she has swelling, erythema, and marked tenderness over the proximal nail fold. The nail itself appears normal. Which of the following is the most likely diagnosis? AFelon BHerpetic whitlow COnychomycosis DParonychia
Correct Answer ( D ) Explanation: Paronychia is an inflammation of the proximal and lateral nail folds of the digits. Acute paronychia, defined as being present for less than six weeks, is usually due to infection but can be a side effect of medications, including chemotherapy agents, retinoids, and antiretroviral drugs. Risk factors for paronychia include nail biting, thumb sucking, ingrown toenails, overly aggressive trimming of cuticles, and repeated immersion of the hands in water. Diabetes mellitus is also a risk factor. If the nail fold barrier is disrupted, bacteria can enter the tissues. The most common infecting organisms are skin flora, such as Staphylococcus aureus and Streptococcus species. Oral flora, including Eikenella corrodens, Fusobacterium, and Peptostreptococcus, may be involved with nail biting or thumb sucking. Initial treatment of acute infection is warm soaks and topical antibiotics. For more severe cases or those that are not responding to treatment, oral antibiotics and drainage of an abscess may be required. Drainage can be performed with or without local anesthesia by digital block using a number 11 scalpel or an injection needle inserted under the nail fold. In contrast to acute paronychia, chronic paronychia is due to local effects of irritants or allergens. Treatment is avoiding the irritant and using topical steroids. Chronic paronychia can become secondarily infected with bacteria or fungus. A felon (A) is a subcutaneous abscess of the pulp space, which is the septated fatty tissue on the volar surface of the fingertip. It presents with pain, erythema, and swelling over the pad of the digit. Treatment is antibiotics, and frequently surgical drainage is necessary. A herpetic whitlow (B) is a skin infection on the tip of a digit, most often the thumb or index finger, caused by herpes simplex virus. It presents with pain, erythema, and a vesicular rash, which may form draining pustules. Onychomycosis (C) is a fungal infection involving the nail plate that typically develops slowly and results in nail discoloration and deformity.
A woman presents to the clinic reporting a thickened patch of skin on the anterior surface of her lower leg, as seen in the picture above. She states this lesion has been present for over 5 years and the skin has progressively become tougher and more noticeably discolored. She reports habitually scratching this area due to her underlying atopic dermatitis. Which of the following is the best diagnosis? ALichen planus BLichen sclerosus CLichen simplex chronicus DLichen striatus
Explanation: Lichen simplex chronicus is a condition caused by habitual rubbing and scratching of the skin that results in a localized plaque of chronic eczematous inflammation. The most commonly affected areas include the back of the neck, wrists, ankles, and anogenital skin. Although more frequently encountered in adults, atopic children may also present with this condition. Additionally, women of reproductive age are more likely to present with this condition compared to post- or premenopausal women. Lichen simplex chronicus is characterized by a sharply demarcated, scaly, deeply violaceous or red plaque with prominent skin lines. Lichenification is thickening of the skin in response to repetitive trauma (e.g., scratching, rubbing). Patients with this condition can develop secondary infection, characterized by moist scaling, crust, pustules, and weeping. Laboratory testing may involve patch testing to rule out allergic contact dermatitis that can lead to or complicate lichen simplex chronicus. Potassium hydroxide scraping can be used to differentiate between this condition and tinea. The plaque of lichen simplex chronicus remains localized to the original area affected and does not enlarge with time. Even minor scratching or rubbing can perpetuate the cycle of lichenification and patients should be instructed to avoid touching the area entirely to ensure complete resolution. The area should be covered if scratching occurs during sleep. To reduce night-time itching, patients can be treated with an oral antihistamine (e.g., diphenhydramine, hydroxyzine) followed by a 20-minute cold tap-water wet dressing and subsequent application of a topical corticosteroid. Initial treatment involves the use of a 5-minute water soak followed by application of a medium-to-high potency topical corticosteroid ointment (e.g., mometasone furoate, triamcinolone acetonide). Low potency corticosteroids (e.g., alclometasone dipropionate, desonide) should be used to treat the genitoanal area or the fold behind the ear. Scalp lesions are treated with a group I or group II steroid gel (fluocinonide) or a solution (clobetasol) twice daily. Secondary infection should be treated with appropriate topical antibiotics (e.g., bacitracin, mupirocin). Superficial X-ray therapy (Grenz ray) is a useful treatment option but is limited in availability. Recurrent picking at the scalp can cause nodules that are highly treatment resistant and require monthly intralesional injections with triamcinolone acetonide. Stress and anxiety can induce scratching and perpetuate this condition and should be addressed to effectively treat and prevent recurrence of lichen simplex chronicus. Lichen planus (A) presents as pruritic, purple, polygonal plaques or papules. Wickham striae are fine white lines that may be present on the surface of lesions. Lichen planus most commonly occurs on the distal extremities. Lichen sclerosus (B) presents as thin, white, wrinkled skin that is most commonly localized to the labia minora or majora. It is often pruritic and can present in areas of epithelial hyperplasia from chronic rubbing, similar to the plaques of lichen simplex chronicus. Location of the plaque as well as the presence of lichenified skin and predisposing atopic dermatitis make lichen simplex chronicus the most likely diagnosis for the patient in the vignette above. Lichen striatus (D) is an asymptomatic condition that presents with unilateral pink, red, or skin-colored flat-topped papules arranged in a linear band that follows the lines of Blaschko. It is most common in children and is self-limited resolves within six to 12 months after initial onset. Lichen striatus has a characteristic V-shaped pattern over the spine and an S-shaped on the lateral and anterior aspect of the trunk.